由买买提看人间百态

boards

本页内容为未名空间相应帖子的节选和存档,一周内的贴子最多显示50字,超过一周显示500字 访问原贴
Parenting版 - 求建议,高中生如何提高学习效率?
相关主题
别鸡兔同笼了,来喝啤酒吧算不算有数学天分
学校早晚要教的东西,早早学会了又如何?大家怎么推孩子的弱项?
构建式数学在中国少年智力开发
关于刷还是不刷[转载] 为什么说usamo 简单
是不是任我儿子继续迷象棋二年级女儿的report
9岁, 如何报名考amc10觉得美帝数学进度慢的进来看一下
求科普: 奥数 vs. Math Olympiad vs. Math Count vs. Math circle vs. 超前学数学分数应用题
有明天考AMC 8的吗?我招,我是猪
相关话题的讨论汇总
话题: 孩子话题: 女儿话题: 学习话题: 老大话题: 法国
进入Parenting版参与讨论
1 2 下页 末页 (共2页)
g***3
发帖数: 471
1
各位父母好,我是在法国的妈妈,去年,我在本坛发帖,求教家里2大孩子选学问题,
得到大家许多有益的启发,
平时经常潜水,这里大孩子父母有很多可鉴的经验,常常给我很多帮助。先谢谢大家。
目前,我又有一个新的问题,希望拿来和大家讨论。
我女儿目前上高一,是在一个英美系里,这个高中比较特殊,是法国和德国联合办的,
有英美系,是为家里父母一方英美人士,或法国常驻英美国家,回归人士的孩子而设的
,是需要通过国家竞考择优入取的。
这个系统的课程,比一般法国高中生每周多5个小时的学习。还是联合国下属学校之一
,课外活动比较多。
女儿从小学习到初中,比较顺利,她也比较好学。
目前,我们发现,她在学习上化费太多的时间,但是效率不高,,尤其家里有一个高中
快毕业的哥哥,也常说她时间化得太长,不正常。女儿以前各科都比较均衡,理科更容
易些。初中毕业前期,学校组织考测学生各项能力评估,她强项在logique,抽象思维
和艺术创作等,,她语言能力也很强,,英文写作在班上名列前茅。
我们不是英美人士,我在法国生活多年,以前英文还可以,但是毕竟不长期在英美生活
,但我们经常去英国度假。我先生是理科毕业,喜欢数学。女儿在英文上的优势,还是
令我们比较欣慰。
她现在的问题是,每次遇到物理考试,就心慌,从这个学季开始,物理成绩下降,昨天
她有数学考试,考砸了,考试后马上给我短消息,她很泄气,很难过。我先生在国外出
差,平时都是他给孩子辅导。现在也帮不上忙。
昨晚上,女儿要我帮她复习化学,我看她写了好多小卡片,,分别用不同的颜色,不同
的文字花式,很好看,就是时间上不知不觉花花流,,她平时写文章,会花很多时间,
去查资料,这样时间上,各门课程,好象顾不及,
她朋友也很多,周末,经常和以前初中的同学聚会,现在的同学也交了几个知心的,她
常常感觉时间不够用,现在理科上成绩有点下降,法国这里来年9月开学,就需要分科
,选理科,还是商科,,她一直拿不定主意,
大家怎么看?
g***3
发帖数: 471
2
现在这里人气怎么没以前热闹了
d****g
发帖数: 7460
3
"昨晚上,女儿要我帮她复习化学,我看她写了好多小卡片,,分别用不同的颜色,不同
的文字花式,很好看,就是时间上不知不觉花花流"
--- 这个描述很仔细。很有警戒作用。。。学到了。
d****g
发帖数: 7460
4
真的吗?

【在 g***3 的大作中提到】
: 现在这里人气怎么没以前热闹了
g***3
发帖数: 471
5
是啊,以前那个贴子,都有好几十页,月光哪去了,,
还有呼唤PACMOM

【在 d****g 的大作中提到】
: 真的吗?
g***3
发帖数: 471
6
以你之见,是好,还是坏尼,
她今天提早回家,物理化学老师不在,所以本来有个考试,要等到放假后,再说。
我跟她聊,我发现,就是由于她在做学习小卡片时,字体花样多,不知不觉消化好多时
间,,
比如,明天,他们法语有个评论考试,,她现在在手机网上不知在看啥,我也不好多问,
如何帮她提高时间利用率尼,,

不同

【在 d****g 的大作中提到】
: "昨晚上,女儿要我帮她复习化学,我看她写了好多小卡片,,分别用不同的颜色,不同
: 的文字花式,很好看,就是时间上不知不觉花花流"
: --- 这个描述很仔细。很有警戒作用。。。学到了。

g***3
发帖数: 471
7
她吃了点心,现在就开始学习了,我问她,刚才看啥,她说是寻找学习方法。
d****g
发帖数: 7460
8
我觉得不好。。。
我娃小,三年级,做题喜欢自己在脑子里想,想半天,跟发呆一样。突然崩出来个答案
。要是对了,我很开心。要是错了,我很抓狂。
然后我就想起国内要求写步骤。顿觉国内老师也不是吃素的。那教学经验肯定是杠杠的
。所以开始要求娃写步骤。学习使用数学语言。好比令,LET,X,括号,属于,任
取,SIGMA,Just kidding.
然后就发现娃经常光写了,不想了。写耽误了理解。晕倒。
所以我的意思是思路要先发散,然后再整理?基建的时候不要忙乎收拾?少写多想?不
光要写,更还要想?
对你娃,先要求她不许做卡片。做卡片是卖卡片的人干的事儿。

问,

【在 g***3 的大作中提到】
: 以你之见,是好,还是坏尼,
: 她今天提早回家,物理化学老师不在,所以本来有个考试,要等到放假后,再说。
: 我跟她聊,我发现,就是由于她在做学习小卡片时,字体花样多,不知不觉消化好多时
: 间,,
: 比如,明天,他们法语有个评论考试,,她现在在手机网上不知在看啥,我也不好多问,
: 如何帮她提高时间利用率尼,,
:
: 不同

v***s
发帖数: 4031
9
我是工科毕业的,我有两个儿子,没有一个学工,学理。孩子学什么,要看他们的兴趣
,做父母要观察,根据他们的特点,引导他们学习的方向。初中阶段是培养孩子方法,
提高学习效率最好时机。
我的大孩子,从小喜欢阅读,学校里学得很扎实,初中的时候没有感觉到,在高中时,
学的很轻松,所有的升学标准考试都是没怎么复习,一次考过。大学也是学的没有什么
压力,去年提前毕业了。二儿子,从小喜欢玩,不喜欢学习,高中毕业之前的学习,在
我们看来是很挣扎的,他倒是开开心心毕业了。现在学的是偏文的专业,他数学不是很
好。大学的学习比高中阶段要好很多,可能是学的是他喜欢的。

【在 g***3 的大作中提到】
: 各位父母好,我是在法国的妈妈,去年,我在本坛发帖,求教家里2大孩子选学问题,
: 得到大家许多有益的启发,
: 平时经常潜水,这里大孩子父母有很多可鉴的经验,常常给我很多帮助。先谢谢大家。
: 目前,我又有一个新的问题,希望拿来和大家讨论。
: 我女儿目前上高一,是在一个英美系里,这个高中比较特殊,是法国和德国联合办的,
: 有英美系,是为家里父母一方英美人士,或法国常驻英美国家,回归人士的孩子而设的
: ,是需要通过国家竞考择优入取的。
: 这个系统的课程,比一般法国高中生每周多5个小时的学习。还是联合国下属学校之一
: ,课外活动比较多。
: 女儿从小学习到初中,比较顺利,她也比较好学。

g***3
发帖数: 471
10
你这个讲得很具体,女儿现在有点类似你说的,
但她以前没这么认真,以前初中课程,比较轻松,虽然,也是在欧洲班,课程比一般学
生多,但没发现她象现在那样。
现在她做的卡片,是便于她的记忆,都是写的公式,化学演变方程,元素代号等,
她思维应该比较抽象,相比哥哥,不具体,,哥哥以前物理是强项,,抽象数学是弱点
,但现在改到商科,学习应用数学,又发现是强项了,哥哥以前学习没有妹妹现在这么
努力的,,
女儿电脑也蛮上手的,很小就自学了一些图像软件,,,

【在 d****g 的大作中提到】
: 我觉得不好。。。
: 我娃小,三年级,做题喜欢自己在脑子里想,想半天,跟发呆一样。突然崩出来个答案
: 。要是对了,我很开心。要是错了,我很抓狂。
: 然后我就想起国内要求写步骤。顿觉国内老师也不是吃素的。那教学经验肯定是杠杠的
: 。所以开始要求娃写步骤。学习使用数学语言。好比令,LET,X,括号,属于,任
: 取,SIGMA,Just kidding.
: 然后就发现娃经常光写了,不想了。写耽误了理解。晕倒。
: 所以我的意思是思路要先发散,然后再整理?基建的时候不要忙乎收拾?少写多想?不
: 光要写,更还要想?
: 对你娃,先要求她不许做卡片。做卡片是卖卡片的人干的事儿。

相关主题
9岁, 如何报名考amc10算不算有数学天分
求科普: 奥数 vs. Math Olympiad vs. Math Count vs. Math circle vs. 超前学数学大家怎么推孩子的弱项?
有明天考AMC 8的吗?少年智力开发
进入Parenting版参与讨论
d****g
发帖数: 7460
11
"尤其家里有一个高中快毕业的哥哥,也常说她时间化得太长,不正常。"
这个也给你提个醒。从前我就是那个看不惯的哥哥,后来发现自己当时啥也不懂。每个
娃学习方法不同,家长不应该赞同一个,不赞同另一个,哪怕是哥哥的成绩好些。有时
候觉得是为了借鉴一下经验,但经验好不好用还是妹妹说的算。
个人看法哈。
d****g
发帖数: 7460
12
商?法?医?

【在 v***s 的大作中提到】
: 我是工科毕业的,我有两个儿子,没有一个学工,学理。孩子学什么,要看他们的兴趣
: ,做父母要观察,根据他们的特点,引导他们学习的方向。初中阶段是培养孩子方法,
: 提高学习效率最好时机。
: 我的大孩子,从小喜欢阅读,学校里学得很扎实,初中的时候没有感觉到,在高中时,
: 学的很轻松,所有的升学标准考试都是没怎么复习,一次考过。大学也是学的没有什么
: 压力,去年提前毕业了。二儿子,从小喜欢玩,不喜欢学习,高中毕业之前的学习,在
: 我们看来是很挣扎的,他倒是开开心心毕业了。现在学的是偏文的专业,他数学不是很
: 好。大学的学习比高中阶段要好很多,可能是学的是他喜欢的。

g***3
发帖数: 471
13
这个让我想起我家老大,我以前有关老大,也在本坛发过帖子,
他从小就喜欢玩,学习不扎实,但还是算中上的,高中进了一个较好的私立,从高2开
始,发现数学难,特别是在考试时,太折磨人心,情绪低落,高3,转到了商科,成绩
一下就赶上了,现在快要高中毕业,他以前的志向是商学院,法国最好的商学院招受来
自理科和商科最好的预科高中的学生,他现在是信心满怀了,
妹妹从小学习轻松,成绩一直是前3名,特别她的私立初中也是法国最好之一,我们从
来没有怀疑过,
她平时喜欢画画,喜欢看书,小时学过音乐,到初中最后一年,没有时间就停了,
现在是学校演讲会的成员,,
以前,他们学校还测试过,她平时在科学馆数学活动中,相比其他在场的人,也显出些
特长,
没有想到,现在出现这些问题,,,

【在 v***s 的大作中提到】
: 我是工科毕业的,我有两个儿子,没有一个学工,学理。孩子学什么,要看他们的兴趣
: ,做父母要观察,根据他们的特点,引导他们学习的方向。初中阶段是培养孩子方法,
: 提高学习效率最好时机。
: 我的大孩子,从小喜欢阅读,学校里学得很扎实,初中的时候没有感觉到,在高中时,
: 学的很轻松,所有的升学标准考试都是没怎么复习,一次考过。大学也是学的没有什么
: 压力,去年提前毕业了。二儿子,从小喜欢玩,不喜欢学习,高中毕业之前的学习,在
: 我们看来是很挣扎的,他倒是开开心心毕业了。现在学的是偏文的专业,他数学不是很
: 好。大学的学习比高中阶段要好很多,可能是学的是他喜欢的。

g***3
发帖数: 471
14
我家是,以前哥哥学习成绩没妹妹这么好,他自己也老是说妹妹厉害,
但现在哥哥高3,学习成绩一下就窜上了,他就说,自己以前高一时,没有化这么多时
间,他也为妹妹着急,他们的学校都是法国最好的高中之一了,哥哥现在志向很高,也
希望妹妹早做打算,

【在 d****g 的大作中提到】
: "尤其家里有一个高中快毕业的哥哥,也常说她时间化得太长,不正常。"
: 这个也给你提个醒。从前我就是那个看不惯的哥哥,后来发现自己当时啥也不懂。每个
: 娃学习方法不同,家长不应该赞同一个,不赞同另一个,哪怕是哥哥的成绩好些。有时
: 候觉得是为了借鉴一下经验,但经验好不好用还是妹妹说的算。
: 个人看法哈。

g***3
发帖数: 471
15
同问,他们现在大学学什么,,

【在 d****g 的大作中提到】
: 商?法?医?
g***3
发帖数: 471
16
记得她小学时,都没啥作业的,
包括现在的家里的老三,,现在是3年级,也没啥作业,回家就玩,现在在看电视,
学校要他直接就跳到5年级,,
真的,他也是很喜欢看书的,尤其科学画报,,

【在 d****g 的大作中提到】
: 我觉得不好。。。
: 我娃小,三年级,做题喜欢自己在脑子里想,想半天,跟发呆一样。突然崩出来个答案
: 。要是对了,我很开心。要是错了,我很抓狂。
: 然后我就想起国内要求写步骤。顿觉国内老师也不是吃素的。那教学经验肯定是杠杠的
: 。所以开始要求娃写步骤。学习使用数学语言。好比令,LET,X,括号,属于,任
: 取,SIGMA,Just kidding.
: 然后就发现娃经常光写了,不想了。写耽误了理解。晕倒。
: 所以我的意思是思路要先发散,然后再整理?基建的时候不要忙乎收拾?少写多想?不
: 光要写,更还要想?
: 对你娃,先要求她不许做卡片。做卡片是卖卡片的人干的事儿。

x***1
发帖数: 999
17
现在你家老大数学成绩好吗?成绩一下子窜上来,那些科目?
看了你以前的帖子,那时人山人海,好不热闹啊。

【在 g***3 的大作中提到】
: 我家是,以前哥哥学习成绩没妹妹这么好,他自己也老是说妹妹厉害,
: 但现在哥哥高3,学习成绩一下就窜上了,他就说,自己以前高一时,没有化这么多时
: 间,他也为妹妹着急,他们的学校都是法国最好的高中之一了,哥哥现在志向很高,也
: 希望妹妹早做打算,

v***s
发帖数: 4031
18
老大今年进医学院,老二一般的文科。

【在 d****g 的大作中提到】
: 商?法?医?
t******l
发帖数: 10908
19
这个冰冻三尺,非一日之寒。法国不知道,但美帝小学初中都是 promote better
participation 的,better participation get better GPA。这本身也没啥错,但有
了推妈性的父母容易 over-do,(也就是古人说的 过犹不及),特别是对于听话型的
娃影响更大一些。
但如果到了高中,也就是两次大脑灰质剪枝完毕之后。我觉得想改变也得循序渐进,强
扭的瓜不甜。
另一方面,可以在人生择业上选择 promote 细心的那种职位,避免那些需要一定程度
“乱中取胜” 能力的职位。当然生物千老还是算了,虽然那职位也 promote 细心,但
是坑爹程度有点高就是了。。。
x***1
发帖数: 999
20
女儿一直学习好,她肯定有自己的一套方法。而且这方法效率还不错。
高一效率不高,是不是潮水的decay理论在起作用,过一段时间会好吧。
女孩子到这个年龄,分心的事太多啦,和某个男孩子在谈对象?

【在 g***3 的大作中提到】
: 各位父母好,我是在法国的妈妈,去年,我在本坛发帖,求教家里2大孩子选学问题,
: 得到大家许多有益的启发,
: 平时经常潜水,这里大孩子父母有很多可鉴的经验,常常给我很多帮助。先谢谢大家。
: 目前,我又有一个新的问题,希望拿来和大家讨论。
: 我女儿目前上高一,是在一个英美系里,这个高中比较特殊,是法国和德国联合办的,
: 有英美系,是为家里父母一方英美人士,或法国常驻英美国家,回归人士的孩子而设的
: ,是需要通过国家竞考择优入取的。
: 这个系统的课程,比一般法国高中生每周多5个小时的学习。还是联合国下属学校之一
: ,课外活动比较多。
: 女儿从小学习到初中,比较顺利,她也比较好学。

相关主题
[转载] 为什么说usamo 简单分数应用题
二年级女儿的report我招,我是猪
觉得美帝数学进度慢的进来看一下请教:怎样说服孩儿他爸多关心孩子的学习?
进入Parenting版参与讨论
g***3
发帖数: 471
21
现在数学他是班上第一,第2的,成绩几乎满分,我们找到一个巴黎高商在读的一年级
学生,帮他辅导,他在私教老师辅导下,做了练习,高3第一季,数学成绩就上来了,
自己信心大增,第2季,成绩更好,,,其他科目是,经济,,,哲学,,历史,地理
,,英语,汉语,西班牙语,和体育,,
他这次总分名列全校商科100多学生中,第3名,我们都为他高兴,,
所以,我觉得心理因素,占了大头,他们学校学生都是从小就很厉害的,

【在 x***1 的大作中提到】
: 现在你家老大数学成绩好吗?成绩一下子窜上来,那些科目?
: 看了你以前的帖子,那时人山人海,好不热闹啊。

v***s
发帖数: 4031
22
女孩和男孩不一样,除非特别优秀,一般进入大学后,竞争力很强女孩不多,我们得接
受这事实。

【在 g***3 的大作中提到】
: 这个让我想起我家老大,我以前有关老大,也在本坛发过帖子,
: 他从小就喜欢玩,学习不扎实,但还是算中上的,高中进了一个较好的私立,从高2开
: 始,发现数学难,特别是在考试时,太折磨人心,情绪低落,高3,转到了商科,成绩
: 一下就赶上了,现在快要高中毕业,他以前的志向是商学院,法国最好的商学院招受来
: 自理科和商科最好的预科高中的学生,他现在是信心满怀了,
: 妹妹从小学习轻松,成绩一直是前3名,特别她的私立初中也是法国最好之一,我们从
: 来没有怀疑过,
: 她平时喜欢画画,喜欢看书,小时学过音乐,到初中最后一年,没有时间就停了,
: 现在是学校演讲会的成员,,
: 以前,他们学校还测试过,她平时在科学馆数学活动中,相比其他在场的人,也显出些

x***1
发帖数: 999
23
我咋一直认为医是顶尖理工科啊,医是理工皇冠上的明珠啊。

【在 v***s 的大作中提到】
: 老大今年进医学院,老二一般的文科。
t******l
发帖数: 10908
24
主要是女娃生娃已经为社会做贡献了,不需要参军为人类进步做炮灰。。。敲错了。。
。做贡献了。。。

【在 v***s 的大作中提到】
: 女孩和男孩不一样,除非特别优秀,一般进入大学后,竞争力很强女孩不多,我们得接
: 受这事实。

v***s
发帖数: 4031
25
没错,按老美来讲,医是科学,我老二也学的是科学。

【在 x***1 的大作中提到】
: 我咋一直认为医是顶尖理工科啊,医是理工皇冠上的明珠啊。
x***1
发帖数: 999
26
赞父母,你们还是费了心思的,找人辅导,数学一旦爱上了,就如胶似漆,爱屋及乌,
其他课程,犹如探囊取物,轻松得很。
我觉得不是心理因素,男孩子在走自己的路,上道了。
女孩子一般到一定年龄就变得心思太重,影响学习,但到了大学,女生几乎都名列前茅
,男生反而不想学了。

【在 g***3 的大作中提到】
: 现在数学他是班上第一,第2的,成绩几乎满分,我们找到一个巴黎高商在读的一年级
: 学生,帮他辅导,他在私教老师辅导下,做了练习,高3第一季,数学成绩就上来了,
: 自己信心大增,第2季,成绩更好,,,其他科目是,经济,,,哲学,,历史,地理
: ,,英语,汉语,西班牙语,和体育,,
: 他这次总分名列全校商科100多学生中,第3名,我们都为他高兴,,
: 所以,我觉得心理因素,占了大头,他们学校学生都是从小就很厉害的,

i**e
发帖数: 19242
27
初中阶段,孩子们一般会学习和认知适合自己的学习方法和学习习惯
我想当然地以为到了高中
事情多时间少
学习方法和习惯大致是一样的
只是做一些修改,比如事情不能做得那么精细,走20/80路线了
比如这个卡
可以保持,但是写在上面的东西必须精简突出重点
修饰性的部分(比如色彩,绘图,字体等等)就被忽略了
考试没考好,没关系,emotional reaction过后
要用心看看卷子,理解问题是出在了哪里,具体问题相应提高
初中升高中,第一年需要适应,也是挺正常的吧?
t******l
发帖数: 10908
28
好吧我来说一句 “乱中取胜” 型的数学/物理学习的效率问题。
第一:就是卡片在数学/物理里是 joke,直接放把火把卡片全烧了、或者挖个坑把卡片
全埋了。所有概念玩意儿(不包括定式棋谱)都应该是简洁到能记在脑子里的。记不住
的话出门左转文科不谢。
第二:刚开始学的新概念,可以考虑 12 hr 或者 24 hr repeat。一般不用笔纸,走路
时或躺沙发上聊聊。关键不在于手里有没有刀,关键在于心里有没有刀。
第三:做过的题目分析时,超过学校进度的,先可以跳过啰嗦题,反正啰嗦题也不容易
记住。先挑简洁有数学本质数学美的题目分析。我现在 AMC 10 的战术就是先打下
Bastogne,路上不重要的城市就绕过,将来打下中腹后回头再来收拾。万一打不下中腹
也就不浪费带宽,省下来干其他的。
第四:定式棋谱方面,除了考前突击,一般避免去记忆冗长傻鼻的定式,比如二次方程
的球根公式这种,代之以 complete a square 这种概念。
第五:剩下的定式棋谱和做过的题,一部分可以画成简明的图贴在墙上。数学是 space
-time 上 pattern 的艺术,1000 words 不如一张 sketch。。。当然这个是辅助,不
要看到别人墙上贴满了图,就去舍本逐末,把数学课变成绘画课。
第六:路过的给我包子伪币。谢谢。
p****m
发帖数: 860
29
被点名了呵呵。
瞎说几句,我觉得你女儿很棒了,学习方法啥的习惯了让她改也难,但可以给她些建议
,比方说卡片吧,我家两娃都用网上的quizlet,至少不用花时间做卡片了。
网上应该有很多现成的resource,问问她学校建议用什么。
效率我觉得都是逼出来的,真忙了自然效率就高了。像我今天上班就效率特低。。。

【在 g***3 的大作中提到】
: 是啊,以前那个贴子,都有好几十页,月光哪去了,,
: 还有呼唤PACMOM

g***3
发帖数: 471
30
她现在的科目不一般高一学生多5个小时,是英国文学,历史地理,有大量的写作,分
析评论性文章要做,,学的理科,据说是法国相对程度比较高的,比一般的难,他们考
试50分钟,量大,需要的是速度,思维能力强,这个学校很有名,培养了很多法国综合
理工的学生,和政治学院,高商的学生。
女儿初中最后一年,和一个男同学确立了关系,处了4个多月,后来放假,还有女儿换
了高中,
哥哥也对这个男生不太满意,帮女儿分析后,她停了,,没时间

【在 x***1 的大作中提到】
: 女儿一直学习好,她肯定有自己的一套方法。而且这方法效率还不错。
: 高一效率不高,是不是潮水的decay理论在起作用,过一段时间会好吧。
: 女孩子到这个年龄,分心的事太多啦,和某个男孩子在谈对象?

相关主题
娃又在推算他的质数公式。。。学校早晚要教的东西,早早学会了又如何?
大学录取率构建式数学在中国
别鸡兔同笼了,来喝啤酒吧关于刷还是不刷
进入Parenting版参与讨论
x***1
发帖数: 999
31
看看月光在你以前帖子里的回帖,铿锵有力,如五雷轰顶,振聋发聩。一般人都会急,
你没有。
他儿子数学比美国很多孩子要好
否则考不上欧洲的理科班的
那里面的题目,我敢说这里的家长绝大多数都做不出来,很难的
有些题目就连PhD都做不上来
所以他们家的问题应该不是智商,也不是无能,而是环境,这就没有太好的办法了
现在看来这对父母受过的教育程度和质量都是起点很高的,不然根本辅导不了孩子。
国外尤其欧洲的高中理科课很难的,没有你们以为的那么轻松自在,清华北大理科毕业
的很多留学生辅导不了这边的好高中的理科课程,都是国外念过高中或者本科的才行
我感觉这家已经开始体验到继续辅导的吃力了,所以从外面请家教。
还有:
男孩虽然可以粗旷一些,但粗旷也有不同种类的,也是拿来抗衡的
别人都有肌肉6块,长腿粗壮,经常在教室里把衣服拉上去,站讲台给其他女孩看自己
多么伟岸和雄壮,引来top 5的几个大美女的一阵赞叹和欢声笑语, 好不热闹!
可你儿子没长,他好意思不去Gym练么? 去练了,又不肯告诉你为啥要练,你呢,就以
为他是贪玩,就训他,数学题都不会做,还出去玩, 其实他也不想练的,可是他为了
争口气,夺回那几个美女的正眼瞧他一次,也要咬牙练下去啊
这是人性,是本色, 也是成长的代价, 古代不是都有两国的皇上为了争夺一个美女而
发动战争么? 在美国的教育系统里,也一样的
这就是为啥3岁的母亲就总是甜蜜的bso,因为3岁的小破孩儿根本没有这么多琐事要操
心要分心啊。。。到了13岁,就傻眼了。
德国的高中是最难念的,毕业了都相当于大学水平了。

【在 g***3 的大作中提到】
: 是啊,以前那个贴子,都有好几十页,月光哪去了,,
: 还有呼唤PACMOM

d****g
发帖数: 7460
32
初中升高中第一年真是跨度挺大。
我姐,考重点高中分挺高的。第一次物理,差点不及格。印象里是家里出大事了,我姐
躲屋子里哭。我爸妈找老师谈话。最后,我姐毅然决然的去学了文科。
等我上高中,第一次物理考试,我考了91。班上十名左右?我错愕,特别不理解,咋能
错那么多呢。不就一个F=ma吗。小填空题,判断题,一个一分,东一分西一分的丢,可
没少错。还好有我姐打预防针,要不我也学文了。

【在 i**e 的大作中提到】
: 初中阶段,孩子们一般会学习和认知适合自己的学习方法和学习习惯
: 我想当然地以为到了高中
: 事情多时间少
: 学习方法和习惯大致是一样的
: 只是做一些修改,比如事情不能做得那么精细,走20/80路线了
: 比如这个卡
: 可以保持,但是写在上面的东西必须精简突出重点
: 修饰性的部分(比如色彩,绘图,字体等等)就被忽略了
: 考试没考好,没关系,emotional reaction过后
: 要用心看看卷子,理解问题是出在了哪里,具体问题相应提高

t******l
发帖数: 10908
33
:还好有我姐打预防针,要不我也学文了。
哈哈哈哈哈哈哈哈哈

【在 d****g 的大作中提到】
: 初中升高中第一年真是跨度挺大。
: 我姐,考重点高中分挺高的。第一次物理,差点不及格。印象里是家里出大事了,我姐
: 躲屋子里哭。我爸妈找老师谈话。最后,我姐毅然决然的去学了文科。
: 等我上高中,第一次物理考试,我考了91。班上十名左右?我错愕,特别不理解,咋能
: 错那么多呢。不就一个F=ma吗。小填空题,判断题,一个一分,东一分西一分的丢,可
: 没少错。还好有我姐打预防针,要不我也学文了。

t******l
发帖数: 10908
34
所以关键是回帖的第一句话要牛鼻,后面的都是点缀无所谓了。。。

【在 x***1 的大作中提到】
: 看看月光在你以前帖子里的回帖,铿锵有力,如五雷轰顶,振聋发聩。一般人都会急,
: 你没有。
: 他儿子数学比美国很多孩子要好
: 否则考不上欧洲的理科班的
: 那里面的题目,我敢说这里的家长绝大多数都做不出来,很难的
: 有些题目就连PhD都做不上来
: 所以他们家的问题应该不是智商,也不是无能,而是环境,这就没有太好的办法了
: 现在看来这对父母受过的教育程度和质量都是起点很高的,不然根本辅导不了孩子。
: 国外尤其欧洲的高中理科课很难的,没有你们以为的那么轻松自在,清华北大理科毕业
: 的很多留学生辅导不了这边的好高中的理科课程,都是国外念过高中或者本科的才行

i**e
发帖数: 19242
35
会考试,也是一种能力:)
那你姐现在混得咋样?在祖国风生水起地吧?呵呵

【在 d****g 的大作中提到】
: 初中升高中第一年真是跨度挺大。
: 我姐,考重点高中分挺高的。第一次物理,差点不及格。印象里是家里出大事了,我姐
: 躲屋子里哭。我爸妈找老师谈话。最后,我姐毅然决然的去学了文科。
: 等我上高中,第一次物理考试,我考了91。班上十名左右?我错愕,特别不理解,咋能
: 错那么多呢。不就一个F=ma吗。小填空题,判断题,一个一分,东一分西一分的丢,可
: 没少错。还好有我姐打预防针,要不我也学文了。

g***3
发帖数: 471
36
是的,我现在发现了这一点,女儿从小天资比哥哥聪明,,
哥哥小时,不知不觉的,,
哥哥是到了高3,学到了自己喜欢的科目,遇到了志趣相投的同学,才奋起力争。

【在 v***s 的大作中提到】
: 女孩和男孩不一样,除非特别优秀,一般进入大学后,竞争力很强女孩不多,我们得接
: 受这事实。

d****g
发帖数: 7460
37
心有余悸。
每次做电梯,我都会心里默想,先超重,在正常,再失重。
每次下雨,我都研究雨滴痕迹在车窗上形成的角度。
:)

【在 d****g 的大作中提到】
: 初中升高中第一年真是跨度挺大。
: 我姐,考重点高中分挺高的。第一次物理,差点不及格。印象里是家里出大事了,我姐
: 躲屋子里哭。我爸妈找老师谈话。最后,我姐毅然决然的去学了文科。
: 等我上高中,第一次物理考试,我考了91。班上十名左右?我错愕,特别不理解,咋能
: 错那么多呢。不就一个F=ma吗。小填空题,判断题,一个一分,东一分西一分的丢,可
: 没少错。还好有我姐打预防针,要不我也学文了。

g***3
发帖数: 471
38
法国学医的,都是要从理科学上去的,,女儿喜欢小动物,测试说,她也适合当医生。

【在 x***1 的大作中提到】
: 我咋一直认为医是顶尖理工科啊,医是理工皇冠上的明珠啊。
i**e
发帖数: 19242
39
你说我个小学毕业的,语数均体教的
在这种高大上的楼里瞎掰活啥捏
等高人华山论剑,等高楼,火钳刘明:)

【在 d****g 的大作中提到】
: 初中升高中第一年真是跨度挺大。
: 我姐,考重点高中分挺高的。第一次物理,差点不及格。印象里是家里出大事了,我姐
: 躲屋子里哭。我爸妈找老师谈话。最后,我姐毅然决然的去学了文科。
: 等我上高中,第一次物理考试,我考了91。班上十名左右?我错愕,特别不理解,咋能
: 错那么多呢。不就一个F=ma吗。小填空题,判断题,一个一分,东一分西一分的丢,可
: 没少错。还好有我姐打预防针,要不我也学文了。

x***1
发帖数: 999
40
哈哈,

【在 t******l 的大作中提到】
: 所以关键是回帖的第一句话要牛鼻,后面的都是点缀无所谓了。。。
相关主题
关于刷还是不刷求科普: 奥数 vs. Math Olympiad vs. Math Count vs. Math circle vs. 超前学数学
是不是任我儿子继续迷象棋有明天考AMC 8的吗?
9岁, 如何报名考amc10算不算有数学天分
进入Parenting版参与讨论
g***3
发帖数: 471
41
儿子以前不喜欢他学校的数学老师,说她老套,不理解学生。

【在 x***1 的大作中提到】
: 赞父母,你们还是费了心思的,找人辅导,数学一旦爱上了,就如胶似漆,爱屋及乌,
: 其他课程,犹如探囊取物,轻松得很。
: 我觉得不是心理因素,男孩子在走自己的路,上道了。
: 女孩子一般到一定年龄就变得心思太重,影响学习,但到了大学,女生几乎都名列前茅
: ,男生反而不想学了。

d****g
发帖数: 7460
42
可不是吗!比我强多了。不过墙里墙外,已经成功的把我侄女撵出来了。

【在 i**e 的大作中提到】
: 会考试,也是一种能力:)
: 那你姐现在混得咋样?在祖国风生水起地吧?呵呵

x***1
发帖数: 999
43
还是兴趣吧,兴趣在减退,还在适应新学校?兴趣在转移?所谓效率,就看你用心不用
心,你专心了,效率自然就提高了。磨磨蹭蹭,不想做,非要让做,哪有效率啊。但不
能光看学习啊,兴趣转到其他方面,没准对孩子将来发展更有利啊,所以,观察孩子的
一些动向,看看心思在想啥?

【在 g***3 的大作中提到】
: 各位父母好,我是在法国的妈妈,去年,我在本坛发帖,求教家里2大孩子选学问题,
: 得到大家许多有益的启发,
: 平时经常潜水,这里大孩子父母有很多可鉴的经验,常常给我很多帮助。先谢谢大家。
: 目前,我又有一个新的问题,希望拿来和大家讨论。
: 我女儿目前上高一,是在一个英美系里,这个高中比较特殊,是法国和德国联合办的,
: 有英美系,是为家里父母一方英美人士,或法国常驻英美国家,回归人士的孩子而设的
: ,是需要通过国家竞考择优入取的。
: 这个系统的课程,比一般法国高中生每周多5个小时的学习。还是联合国下属学校之一
: ,课外活动比较多。
: 女儿从小学习到初中,比较顺利,她也比较好学。

g***3
发帖数: 471
44
你来啦,很高兴又见到你,
记得我老大小学3年级,为了他结交一个同学,关系太粘而在此坛发帖,得到你很多的
启发。
时间真快,现在老大快上大学预科了。
女儿是一个完美的孩子,她做的学习卡片,在我看来,简直是艺术品,,我想她化了时
间,
比如,她喜欢学习中文,自己也做小卡片,虽然法国这里不学笔划,她看中文字,就如
看画,她写的字,也很工整,她是一个很喜欢学习的人。考试没考好,对她打击蛮大的
,我心疼,,我想,今年来,哥哥回家,常说起同学的事,,对女儿压力也有点,她希
望自己有好成绩,,,
我们一直在帮她,前天,在家准备的还好好的,哪知道考试,就被第一个题难到了,而
且她说第一题,她在家碰到过,会,但考试时,就想不起来了,所以就一连窜,影响了
后面的题目,,,

【在 i**e 的大作中提到】
: 初中阶段,孩子们一般会学习和认知适合自己的学习方法和学习习惯
: 我想当然地以为到了高中
: 事情多时间少
: 学习方法和习惯大致是一样的
: 只是做一些修改,比如事情不能做得那么精细,走20/80路线了
: 比如这个卡
: 可以保持,但是写在上面的东西必须精简突出重点
: 修饰性的部分(比如色彩,绘图,字体等等)就被忽略了
: 考试没考好,没关系,emotional reaction过后
: 要用心看看卷子,理解问题是出在了哪里,具体问题相应提高

g***3
发帖数: 471
45
是是,真忙,效率就出来了,,,
家里,女儿象老公,,做事慢,,儿子象我,,同时能做几件事,
网上也找了,,找不到适合的练习题,
她做卡,其实就是自己记忆的过程,,写下来,能记得住,,但是她喜欢一边听音乐,,
哎,想起自己那时,也是一边听音乐,一边做功课,
女儿班上同学家长,很多是NASA项目的物理学家,,
学校物理老师也见了,说女儿课堂表现很好,也不懂为啥考试,就不太好,,,

【在 p****m 的大作中提到】
: 被点名了呵呵。
: 瞎说几句,我觉得你女儿很棒了,学习方法啥的习惯了让她改也难,但可以给她些建议
: ,比方说卡片吧,我家两娃都用网上的quizlet,至少不用花时间做卡片了。
: 网上应该有很多现成的resource,问问她学校建议用什么。
: 效率我觉得都是逼出来的,真忙了自然效率就高了。像我今天上班就效率特低。。。

g***3
发帖数: 471
46
她兴趣到是没降,
她的问题,就是太投入,为了写一篇文章,她会化几天的时间,,
虽然写出的文章,得到了老师的好评,但是,哥哥说,他2个小时就能应付,妹妹要花
几倍的时间,,

【在 x***1 的大作中提到】
: 还是兴趣吧,兴趣在减退,还在适应新学校?兴趣在转移?所谓效率,就看你用心不用
: 心,你专心了,效率自然就提高了。磨磨蹭蹭,不想做,非要让做,哪有效率啊。但不
: 能光看学习啊,兴趣转到其他方面,没准对孩子将来发展更有利啊,所以,观察孩子的
: 一些动向,看看心思在想啥?

g***3
发帖数: 471
47
我先做饭了。潮水的建议,要好好琢磨理解,等会回复
x***1
发帖数: 999
48
奇了怪咧,我每次去饭店吃饭,都要根据客流量算算这个饭店能不能盈利,联想到
Bernoulli distribution, 其均值方差是多少,依次决定给小费的多少。看到河流,要
算算河流量有多大,能供多少人吃喝,联想到Reynolds number。看到公共汽车,要算
算政府每辆车得补贴多少,包括汽油价格,票价,路程,司机的工资等等。每次下雨,
我研究的对象是雨滴的形成,联想到Raleigh instability, 有序排列等等。

【在 d****g 的大作中提到】
: 心有余悸。
: 每次做电梯,我都会心里默想,先超重,在正常,再失重。
: 每次下雨,我都研究雨滴痕迹在车窗上形成的角度。
: :)

d**********h
发帖数: 2795
49
理科生
人鲁迅就说一棵树,还有一棵树。。。

【在 x***1 的大作中提到】
: 奇了怪咧,我每次去饭店吃饭,都要根据客流量算算这个饭店能不能盈利,联想到
: Bernoulli distribution, 其均值方差是多少,依次决定给小费的多少。看到河流,要
: 算算河流量有多大,能供多少人吃喝,联想到Reynolds number。看到公共汽车,要算
: 算政府每辆车得补贴多少,包括汽油价格,票价,路程,司机的工资等等。每次下雨,
: 我研究的对象是雨滴的形成,联想到Raleigh instability, 有序排列等等。

h*****m
发帖数: 1034
50
收藏了。转账了。
转帐完成
转给用户:timefall,现金(伪币):10,收取手续费:0.10
但来挑点刺儿:不是所有孩子都能做到“快糙猛“的,或者说绝大部分孩子做不到。课
本里要求记住“冗长傻鼻的定式“对于大部分孩子不是没有道理的。
引用一段金庸老先生说的:
张无忌不知自己这么想,只因身负九阳神功之故,他所设想的招数虽能克敌制胜,却决
不是比殷、宋二人更妙更精,常人更万万无法做到。正如飞禽见地下狮虎搏斗,不免会
想:“何不高飞下扑,可制必胜?”殊不知狮虎在百兽之中虽然最为凶猛厉害,要高飞
下扑,却是力所不能。张无忌见识未够广搏,一时想不到其中的缘故。
我可不是在讽刺你“见识未够广博“,而且也给你封口费了,千万不要追杀我。。。

好吧我来说一句 “乱中取胜” 型的数学/物理学习的效率问题。
第一:就是卡片在数学/物理里是 joke,直接放把火把卡片全烧了、或者挖个坑把卡片
全埋了。所有概念玩意儿(不包括定式棋谱)都应该是简洁到能记在脑子里的。记不住
的话出门左转文科不谢。
第二:刚开始学的新概念,可以考虑 12 hr 或者 24 hr repeat。一般不用笔纸,走路
时或躺沙发上聊聊。关键不在于手里有没有刀,关键在于心里有没有刀。
第三:做过的题目分析时,超过学校进度的,先可以跳过啰嗦题,反正啰嗦题也不容易
记住。先挑简洁有数学本质数学美的题目分析。我现在 AMC 10 的战术就是先打下
Bastogne,路上不重要的城市就绕过,将来打下中腹后回头再来收拾。万一打不下中腹
也就不浪费带宽,省下来干其他的。
第四:定式棋谱方面,除了考前突击,一般避免去记忆冗长傻鼻的定式,比如二次方程
的球根公式这种,代之以 complete a square 这种概念。
第五:剩下的定式棋谱和做过的题,一部分可以画成简明的图贴在墙上。数学是 space
-time 上 pattern 的艺术,1000 words 不如一张 sketch。。。当然这个是辅助,不
要看到别人墙上贴满了图,就去舍本逐末,把数学课变成绘画课。
第六:路过的给我包子伪币。谢谢。

【在 t******l 的大作中提到】
: 好吧我来说一句 “乱中取胜” 型的数学/物理学习的效率问题。
: 第一:就是卡片在数学/物理里是 joke,直接放把火把卡片全烧了、或者挖个坑把卡片
: 全埋了。所有概念玩意儿(不包括定式棋谱)都应该是简洁到能记在脑子里的。记不住
: 的话出门左转文科不谢。
: 第二:刚开始学的新概念,可以考虑 12 hr 或者 24 hr repeat。一般不用笔纸,走路
: 时或躺沙发上聊聊。关键不在于手里有没有刀,关键在于心里有没有刀。
: 第三:做过的题目分析时,超过学校进度的,先可以跳过啰嗦题,反正啰嗦题也不容易
: 记住。先挑简洁有数学本质数学美的题目分析。我现在 AMC 10 的战术就是先打下
: Bastogne,路上不重要的城市就绕过,将来打下中腹后回头再来收拾。万一打不下中腹
: 也就不浪费带宽,省下来干其他的。

相关主题
大家怎么推孩子的弱项?二年级女儿的report
少年智力开发觉得美帝数学进度慢的进来看一下
[转载] 为什么说usamo 简单分数应用题
进入Parenting版参与讨论
g***3
发帖数: 471
51
职业习惯吧,

【在 x***1 的大作中提到】
: 奇了怪咧,我每次去饭店吃饭,都要根据客流量算算这个饭店能不能盈利,联想到
: Bernoulli distribution, 其均值方差是多少,依次决定给小费的多少。看到河流,要
: 算算河流量有多大,能供多少人吃喝,联想到Reynolds number。看到公共汽车,要算
: 算政府每辆车得补贴多少,包括汽油价格,票价,路程,司机的工资等等。每次下雨,
: 我研究的对象是雨滴的形成,联想到Raleigh instability, 有序排列等等。

g***3
发帖数: 471
52
这个列单,我要好好记下,琢磨,
数学,女儿至今,还没做过卡片,
第2点,觉得特别好,新学的概念,就是要在隔夜快速炒一炒,
第3点,他们学校进度已经比较快,我们家里就不额外加量了。不过你说的基本数学美
,这个讲得很到位,

【在 t******l 的大作中提到】
: 好吧我来说一句 “乱中取胜” 型的数学/物理学习的效率问题。
: 第一:就是卡片在数学/物理里是 joke,直接放把火把卡片全烧了、或者挖个坑把卡片
: 全埋了。所有概念玩意儿(不包括定式棋谱)都应该是简洁到能记在脑子里的。记不住
: 的话出门左转文科不谢。
: 第二:刚开始学的新概念,可以考虑 12 hr 或者 24 hr repeat。一般不用笔纸,走路
: 时或躺沙发上聊聊。关键不在于手里有没有刀,关键在于心里有没有刀。
: 第三:做过的题目分析时,超过学校进度的,先可以跳过啰嗦题,反正啰嗦题也不容易
: 记住。先挑简洁有数学本质数学美的题目分析。我现在 AMC 10 的战术就是先打下
: Bastogne,路上不重要的城市就绕过,将来打下中腹后回头再来收拾。万一打不下中腹
: 也就不浪费带宽,省下来干其他的。

t******l
发帖数: 10908
53
虽然大伙儿是在开玩笑,但是爱因斯坦确实也说了,中学数学/物理的真谛,是平时吃
饱了撑的没事瞎几把乱想那些 space-time-pattern 的无数浪漫的邂逅和偶遇,而不是
搞得像一个印刷厂的童工一样去做一套套无比美丽同时又无比傻鼻的 flash-cards。
当然考前突击一下的情况,另说。。。毕竟爱因斯坦也说了,要在 8 billion 猴子的
社会里生存,学点猴群专利局的猴言猴语也是其人生的一项技能。。。

【在 d****g 的大作中提到】
: 心有余悸。
: 每次做电梯,我都会心里默想,先超重,在正常,再失重。
: 每次下雨,我都研究雨滴痕迹在车窗上形成的角度。
: :)

d**********h
发帖数: 2795
54
求一张“精美卡片”的照片,有包子答谢
老生常谈,追求理科答案而舍弃细枝末节的艺术卡片,得也?失也?

【在 g***3 的大作中提到】
: 你来啦,很高兴又见到你,
: 记得我老大小学3年级,为了他结交一个同学,关系太粘而在此坛发帖,得到你很多的
: 启发。
: 时间真快,现在老大快上大学预科了。
: 女儿是一个完美的孩子,她做的学习卡片,在我看来,简直是艺术品,,我想她化了时
: 间,
: 比如,她喜欢学习中文,自己也做小卡片,虽然法国这里不学笔划,她看中文字,就如
: 看画,她写的字,也很工整,她是一个很喜欢学习的人。考试没考好,对她打击蛮大的
: ,我心疼,,我想,今年来,哥哥回家,常说起同学的事,,对女儿压力也有点,她希
: 望自己有好成绩,,,

d**********h
发帖数: 2795
55
潮水兄,别老拿老爱说事儿
大家不是爱因斯坦,没那个智力,没那个运气,也就别琢磨人家的“经验教训”了

【在 t******l 的大作中提到】
: 虽然大伙儿是在开玩笑,但是爱因斯坦确实也说了,中学数学/物理的真谛,是平时吃
: 饱了撑的没事瞎几把乱想那些 space-time-pattern 的无数浪漫的邂逅和偶遇,而不是
: 搞得像一个印刷厂的童工一样去做一套套无比美丽同时又无比傻鼻的 flash-cards。
: 当然考前突击一下的情况,另说。。。毕竟爱因斯坦也说了,要在 8 billion 猴子的
: 社会里生存,学点猴群专利局的猴言猴语也是其人生的一项技能。。。

t******l
发帖数: 10908
56
我只是开玩笑而已。。。我前面说了强扭的瓜不甜,每个人的办法不一样,没有建议人
要扔掉卡片的意思。。。
或者换一个角度,养娃的精髓在于 “抄”。。。但 “抄” 其实也是一门艺术,就好
比三星和小米都抄苹果,但咋抄也不是个容易的事儿。。。当然,苹果 CEO 宣布自己
出柜的那天,俺估计三星和小米的 CEO 一宿都在想 “特么苹果这招让我们怎么抄?”
。。。

【在 d**********h 的大作中提到】
: 潮水兄,别老拿老爱说事儿
: 大家不是爱因斯坦,没那个智力,没那个运气,也就别琢磨人家的“经验教训”了

g***3
发帖数: 471
57
我记得自己也是高中物理不行,高2选了经济类文科,
法国这里高一,是综合,高一第2季,就要开始选文理商高2了,
老大高一成绩还可以,全班第8名,所以我们建议他上理科,他自己想去商的,
到了高2后,就开始后悔,,一直想换,整个高2,心很灰,,
学校终于破格让他换到了高3商。。
所以,现在女儿,也是到了这步,女儿自己拿不定主意,
哥哥想让她去商科,以后考巴黎政治学院,,他现在班上3个同学通过了笔试竟考。

【在 d****g 的大作中提到】
: 初中升高中第一年真是跨度挺大。
: 我姐,考重点高中分挺高的。第一次物理,差点不及格。印象里是家里出大事了,我姐
: 躲屋子里哭。我爸妈找老师谈话。最后,我姐毅然决然的去学了文科。
: 等我上高中,第一次物理考试,我考了91。班上十名左右?我错愕,特别不理解,咋能
: 错那么多呢。不就一个F=ma吗。小填空题,判断题,一个一分,东一分西一分的丢,可
: 没少错。还好有我姐打预防针,要不我也学文了。

t******l
发帖数: 10908
58
属实。。。同时谢谢伪币。。。

【在 h*****m 的大作中提到】
: 收藏了。转账了。
: 转帐完成
: 转给用户:timefall,现金(伪币):10,收取手续费:0.10
: 但来挑点刺儿:不是所有孩子都能做到“快糙猛“的,或者说绝大部分孩子做不到。课
: 本里要求记住“冗长傻鼻的定式“对于大部分孩子不是没有道理的。
: 引用一段金庸老先生说的:
: 张无忌不知自己这么想,只因身负九阳神功之故,他所设想的招数虽能克敌制胜,却决
: 不是比殷、宋二人更妙更精,常人更万万无法做到。正如飞禽见地下狮虎搏斗,不免会
: 想:“何不高飞下扑,可制必胜?”殊不知狮虎在百兽之中虽然最为凶猛厉害,要高飞
: 下扑,却是力所不能。张无忌见识未够广搏,一时想不到其中的缘故。

g***3
发帖数: 471
59
这个确实,

【在 t******l 的大作中提到】
: 虽然大伙儿是在开玩笑,但是爱因斯坦确实也说了,中学数学/物理的真谛,是平时吃
: 饱了撑的没事瞎几把乱想那些 space-time-pattern 的无数浪漫的邂逅和偶遇,而不是
: 搞得像一个印刷厂的童工一样去做一套套无比美丽同时又无比傻鼻的 flash-cards。
: 当然考前突击一下的情况,另说。。。毕竟爱因斯坦也说了,要在 8 billion 猴子的
: 社会里生存,学点猴群专利局的猴言猴语也是其人生的一项技能。。。

g***3
发帖数: 471
60
我上不来照片啊,以前也试过,否则就给大家看看了,

【在 d**********h 的大作中提到】
: 求一张“精美卡片”的照片,有包子答谢
: 老生常谈,追求理科答案而舍弃细枝末节的艺术卡片,得也?失也?

相关主题
我招,我是猪大学录取率
请教:怎样说服孩儿他爸多关心孩子的学习?别鸡兔同笼了,来喝啤酒吧
娃又在推算他的质数公式。。。学校早晚要教的东西,早早学会了又如何?
进入Parenting版参与讨论
t******l
发帖数: 10908
61
当然另一方面,如果开着一架 F-22 隐形战机,但是后座的 F-16 中队转业过来武器控
制员,看见地面的一群群的雷达站就吓得开雷达干扰器、扔雷达干扰箔条。。。那前座
飞行员多半破口大骂后座武器控制员说,你丫干脆直接举块牌子曰:“F-22 在此,欢
迎来揍!” 。。。娘的你丫要是再敢扔雷达干扰箔条,信不信我一拉触发后座弹射椅
的红绳儿,你丫就跟雷达干扰箔条们一块儿下飞机算了!!

【在 t******l 的大作中提到】
: 属实。。。同时谢谢伪币。。。
f**********g
发帖数: 4709
62
人家做卡片的科目是化学啊
化学在高中(尤其高一的无机化学)的时候确实很多零散的知识,跟物理数学还是很不
一样的

【在 t******l 的大作中提到】
: 好吧我来说一句 “乱中取胜” 型的数学/物理学习的效率问题。
: 第一:就是卡片在数学/物理里是 joke,直接放把火把卡片全烧了、或者挖个坑把卡片
: 全埋了。所有概念玩意儿(不包括定式棋谱)都应该是简洁到能记在脑子里的。记不住
: 的话出门左转文科不谢。
: 第二:刚开始学的新概念,可以考虑 12 hr 或者 24 hr repeat。一般不用笔纸,走路
: 时或躺沙发上聊聊。关键不在于手里有没有刀,关键在于心里有没有刀。
: 第三:做过的题目分析时,超过学校进度的,先可以跳过啰嗦题,反正啰嗦题也不容易
: 记住。先挑简洁有数学本质数学美的题目分析。我现在 AMC 10 的战术就是先打下
: Bastogne,路上不重要的城市就绕过,将来打下中腹后回头再来收拾。万一打不下中腹
: 也就不浪费带宽,省下来干其他的。

t******l
发帖数: 10908
63
化学做卡片,我觉得就是读书那年突击、读完还给老师型的科目。随大流混过去就可以
了,反正只要混过去就不影响人生,多几分少几分问题不大。。。因为我觉得人的精力
有限,不值得在副科上花心思琢磨最佳学习策略,抄一个不出大错的策略就完事了。。
。。当然如果是生物方向、或者追藤方向、或者生物追藤方向,那另说。。。

【在 f**********g 的大作中提到】
: 人家做卡片的科目是化学啊
: 化学在高中(尤其高一的无机化学)的时候确实很多零散的知识,跟物理数学还是很不
: 一样的

t******l
发帖数: 10908
64
或者这个问题就好比问 F-22 遇到地面探照灯大队咋办?。。。回答一般就是绕过去就
完事了,别跟探照灯硬拗,多花点油钱可以回程找加油机报销。。。

【在 t******l 的大作中提到】
: 化学做卡片,我觉得就是读书那年突击、读完还给老师型的科目。随大流混过去就可以
: 了,反正只要混过去就不影响人生,多几分少几分问题不大。。。因为我觉得人的精力
: 有限,不值得在副科上花心思琢磨最佳学习策略,抄一个不出大错的策略就完事了。。
: 。。当然如果是生物方向、或者追藤方向、或者生物追藤方向,那另说。。。

f**********g
发帖数: 4709
65
女孩子小学初中比男孩子成绩好,并不表示就比男孩聪明,更认真听话一些而已。潮水
说的decay,也并不是decay,只不过真实水平展示出来而已。
其实你家儿子应该挺聪明的,以前不开窍而已。女儿说不上来,我个人感觉她的特长更
偏文科一些
话又说回来,怎么算聪明呢?有人做题考试小菜一碟,有人轻松写出锦绣文章,有人为
人处事八面玲珑……都是聪明,找到适合自己的道路就好。你们还是多提供精神支持,
让她自己去尝试,不要执着于一时的成绩

【在 g***3 的大作中提到】
: 是的,我现在发现了这一点,女儿从小天资比哥哥聪明,,
: 哥哥小时,不知不觉的,,
: 哥哥是到了高3,学到了自己喜欢的科目,遇到了志趣相投的同学,才奋起力争。

g***3
发帖数: 471
66
是的,女儿喜欢一边做卡片,一边记忆,她昨天做的是化学,化学她还行,还有历史地
理中文啥的。
我觉得她在做卡片时,不知不觉,又天性爱画,就美美的享受时间过去,还没觉得,,
但是高中阶段,又是遇到好多考试,没那么多时间可以给她享用了,

【在 f**********g 的大作中提到】
: 人家做卡片的科目是化学啊
: 化学在高中(尤其高一的无机化学)的时候确实很多零散的知识,跟物理数学还是很不
: 一样的

t******l
发帖数: 10908
67
:话又说回来,怎么算聪明呢?有人做题考试小菜一碟,有人轻松写出锦绣文章,
:有人为人处事八面玲珑……都是聪明,找到适合自己的道路就好。
属实

【在 f**********g 的大作中提到】
: 女孩子小学初中比男孩子成绩好,并不表示就比男孩聪明,更认真听话一些而已。潮水
: 说的decay,也并不是decay,只不过真实水平展示出来而已。
: 其实你家儿子应该挺聪明的,以前不开窍而已。女儿说不上来,我个人感觉她的特长更
: 偏文科一些
: 话又说回来,怎么算聪明呢?有人做题考试小菜一碟,有人轻松写出锦绣文章,有人为
: 人处事八面玲珑……都是聪明,找到适合自己的道路就好。你们还是多提供精神支持,
: 让她自己去尝试,不要执着于一时的成绩

t******l
发帖数: 10908
68
我觉得如果已经是高中了,那知道娃自己的长处短处,找个合适的人生方向,在社会上
成功 survival,比硬拗某一两门课程,重要得多得多。
或者换一个角度看,这也是阿发狗重视重腹、而不拘泥于某一个孤立的无忧角的战略的
另一种实现方式。

【在 g***3 的大作中提到】
: 是的,女儿喜欢一边做卡片,一边记忆,她昨天做的是化学,化学她还行,还有历史地
: 理中文啥的。
: 我觉得她在做卡片时,不知不觉,又天性爱画,就美美的享受时间过去,还没觉得,,
: 但是高中阶段,又是遇到好多考试,没那么多时间可以给她享用了,

g***3
发帖数: 471
69
是是,话是这么说,
但是高中以后每次考试分数,都是很重要的,是以后申请好学校的档案资料啊,
难到美国不需要,
女儿在英美系统,以后可以直接上美国英国大学,学校已经组织了好几次家长会,
就是讨论这些的,美国申请大学比法国早,高中平时所有分数是很重要的,不知到底真
假,

【在 t******l 的大作中提到】
: 化学做卡片,我觉得就是读书那年突击、读完还给老师型的科目。随大流混过去就可以
: 了,反正只要混过去就不影响人生,多几分少几分问题不大。。。因为我觉得人的精力
: 有限,不值得在副科上花心思琢磨最佳学习策略,抄一个不出大错的策略就完事了。。
: 。。当然如果是生物方向、或者追藤方向、或者生物追藤方向,那另说。。。

i**e
发帖数: 19242
70
呵呵,还有这事啊?很高兴以前的帖子能对你有所帮助和启发:)
你女儿要学会懂得取舍,凡事有权重
时间合理安排,在给定的时间内完成合理的工作,做事有“分寸”
以前一直都比较顺利
这个考试也算是一个让她成长的经历
“会考试”真的也是一种能力,学生期间锻炼一下挺好的

【在 g***3 的大作中提到】
: 你来啦,很高兴又见到你,
: 记得我老大小学3年级,为了他结交一个同学,关系太粘而在此坛发帖,得到你很多的
: 启发。
: 时间真快,现在老大快上大学预科了。
: 女儿是一个完美的孩子,她做的学习卡片,在我看来,简直是艺术品,,我想她化了时
: 间,
: 比如,她喜欢学习中文,自己也做小卡片,虽然法国这里不学笔划,她看中文字,就如
: 看画,她写的字,也很工整,她是一个很喜欢学习的人。考试没考好,对她打击蛮大的
: ,我心疼,,我想,今年来,哥哥回家,常说起同学的事,,对女儿压力也有点,她希
: 望自己有好成绩,,,

相关主题
学校早晚要教的东西,早早学会了又如何?是不是任我儿子继续迷象棋
构建式数学在中国9岁, 如何报名考amc10
关于刷还是不刷求科普: 奥数 vs. Math Olympiad vs. Math Count vs. Math circle vs. 超前学数学
进入Parenting版参与讨论
i**e
发帖数: 19242
71
还是那句话
抓重点,做必须做的,培养时间权重概念

【在 g***3 的大作中提到】
: 是的,女儿喜欢一边做卡片,一边记忆,她昨天做的是化学,化学她还行,还有历史地
: 理中文啥的。
: 我觉得她在做卡片时,不知不觉,又天性爱画,就美美的享受时间过去,还没觉得,,
: 但是高中阶段,又是遇到好多考试,没那么多时间可以给她享用了,

t******l
发帖数: 10908
72
美国更看重 GPA。
但是我觉得娃开开心心健康发展,比上一个所谓藤校或者准藤校型州立,重要的多。。
。我从小农角度很实在的考虑,万一推太狠推把娃推出精神问题,没人给我付医药费。
。。至于学校好差那么一点儿,对于最终人生幸福的影响基本没多少,最多也就辛苦多
挣两万块钱,然后统统交税养懒人。。。我觉得何苦为养社会上的懒人做炮灰。。。

【在 g***3 的大作中提到】
: 是是,话是这么说,
: 但是高中以后每次考试分数,都是很重要的,是以后申请好学校的档案资料啊,
: 难到美国不需要,
: 女儿在英美系统,以后可以直接上美国英国大学,学校已经组织了好几次家长会,
: 就是讨论这些的,美国申请大学比法国早,高中平时所有分数是很重要的,不知到底真
: 假,

g***3
发帖数: 471
73
说女儿聪明,我们也只是从比较而言,女儿从小学习比较开窍,,同样的学习,老大小
时,就没有那么领会,,现在小儿子,更是比姐姐领会得快,,而且喜欢看各种书,喜
欢提问,
老大小时,就不喜欢看书,喜欢运动,各种球类,他小时天生过敏,眼睛鼻子总是痒,
有哮喘,老大小时就喜欢交友,比较外向。
现在高3后,又特别喜欢哲学,更被哲学老师捧爱,说他会发现问题,会钻研问题,分
析问题,经常给他很高的评价,他现在很喜欢看书,经常被那些智慧的人感动,喜欢跟
我们讨论各种社会人文哲学问题。
女儿学习上好,外交也可以,独处没问题,常常一个人陶醉很多事,,2个儿子就不喜
欢独处。
做父母的,很想协助他们找到自己的兴趣点,以后可以在事业上发挥,,

【在 f**********g 的大作中提到】
: 女孩子小学初中比男孩子成绩好,并不表示就比男孩聪明,更认真听话一些而已。潮水
: 说的decay,也并不是decay,只不过真实水平展示出来而已。
: 其实你家儿子应该挺聪明的,以前不开窍而已。女儿说不上来,我个人感觉她的特长更
: 偏文科一些
: 话又说回来,怎么算聪明呢?有人做题考试小菜一碟,有人轻松写出锦绣文章,有人为
: 人处事八面玲珑……都是聪明,找到适合自己的道路就好。你们还是多提供精神支持,
: 让她自己去尝试,不要执着于一时的成绩

x***1
发帖数: 999
74
啥原因导致大多数女孩子到了高中就开始不行了?仍然听话啊,不够大度,内敛,爱读
书,记忆力好,男孩子爱骚扰,但文科生女孩子dominate, 逻辑思维能力欠缺,走神。
大人物女的的确屈指可数。倒是生了娃以后,女的上进心就是0.

【在 f**********g 的大作中提到】
: 女孩子小学初中比男孩子成绩好,并不表示就比男孩聪明,更认真听话一些而已。潮水
: 说的decay,也并不是decay,只不过真实水平展示出来而已。
: 其实你家儿子应该挺聪明的,以前不开窍而已。女儿说不上来,我个人感觉她的特长更
: 偏文科一些
: 话又说回来,怎么算聪明呢?有人做题考试小菜一碟,有人轻松写出锦绣文章,有人为
: 人处事八面玲珑……都是聪明,找到适合自己的道路就好。你们还是多提供精神支持,
: 让她自己去尝试,不要执着于一时的成绩

t******l
发帖数: 10908
75
“上进心” 这枚硬币的另一面也就是 “炮灰心”。。。所以上进心为 0 也不是啥坏
事,也可能是 survival 本能的一种表现。。。至于 male 的问题,male 本来就是
mother nature 设计为 expendable 的,炮灰掉也算是为物种进化做贡献。。。

【在 x***1 的大作中提到】
: 啥原因导致大多数女孩子到了高中就开始不行了?仍然听话啊,不够大度,内敛,爱读
: 书,记忆力好,男孩子爱骚扰,但文科生女孩子dominate, 逻辑思维能力欠缺,走神。
: 大人物女的的确屈指可数。倒是生了娃以后,女的上进心就是0.

g***3
发帖数: 471
76
女儿下个月就要16岁了,还不知道以后想干啥,,
但是她很想到英美读书,,她喜欢英美,我们来美国一次,孩子们很喜欢,尤其纽约,,
这个夏天,老大一得到准信,我们想来3个星期,到处走走,

【在 t******l 的大作中提到】
: 我觉得如果已经是高中了,那知道娃自己的长处短处,找个合适的人生方向,在社会上
: 成功 survival,比硬拗某一两门课程,重要得多得多。
: 或者换一个角度看,这也是阿发狗重视重腹、而不拘泥于某一个孤立的无忧角的战略的
: 另一种实现方式。

g***3
发帖数: 471
77
她现在是高一,还没分,所以每科成绩都很重要。

【在 i**e 的大作中提到】
: 呵呵,还有这事啊?很高兴以前的帖子能对你有所帮助和启发:)
: 你女儿要学会懂得取舍,凡事有权重
: 时间合理安排,在给定的时间内完成合理的工作,做事有“分寸”
: 以前一直都比较顺利
: 这个考试也算是一个让她成长的经历
: “会考试”真的也是一种能力,学生期间锻炼一下挺好的

g***3
发帖数: 471
78
是的,时间概念,时间管理,
她中文是通过远程学习,成绩计入学校档案,,我看她刚刚大概已花了2个多小时,写
字,让我用她的小卡片组句,让她读,,她很认真的,,
所以她面临选择,,高2选科问题,

【在 i**e 的大作中提到】
: 还是那句话
: 抓重点,做必须做的,培养时间权重概念

g***3
发帖数: 471
79
美国读大学,对于我们来说,是太贵了。
法国是一个很追求文凭的社会,尤其是第一流的精英学校,有一个好文凭,是一辈子的
事,甚至是流传到下面几代人的事,,

【在 t******l 的大作中提到】
: 美国更看重 GPA。
: 但是我觉得娃开开心心健康发展,比上一个所谓藤校或者准藤校型州立,重要的多。。
: 。我从小农角度很实在的考虑,万一推太狠推把娃推出精神问题,没人给我付医药费。
: 。。至于学校好差那么一点儿,对于最终人生幸福的影响基本没多少,最多也就辛苦多
: 挣两万块钱,然后统统交税养懒人。。。我觉得何苦为养社会上的懒人做炮灰。。。

g***3
发帖数: 471
80
这个上进心,就是自推动力了,,
老大是近一年,才开始爆发的,他去年就很灰心,甚至想到了最差的打算,,他现在对
妹妹的未来,也很认真,对小弟弟的未来,更是不能马虎,总是说我埋没了他的人才,
要让他去最好的学校,,
这个演变过程,我一直在观察,,
女儿是从小就不甘弱后,小时玩,如果输了,就要哭,,,现在小儿子,是会预测,如
果发现自己要输了,就不玩了,玩时,要想尽一切办法赢,,

【在 t******l 的大作中提到】
: “上进心” 这枚硬币的另一面也就是 “炮灰心”。。。所以上进心为 0 也不是啥坏
: 事,也可能是 survival 本能的一种表现。。。至于 male 的问题,male 本来就是
: mother nature 设计为 expendable 的,炮灰掉也算是为物种进化做贡献。。。

相关主题
有明天考AMC 8的吗?少年智力开发
算不算有数学天分[转载] 为什么说usamo 简单
大家怎么推孩子的弱项?二年级女儿的report
进入Parenting版参与讨论
d**********h
发帖数: 2795
81
放在附件里就可以了
web版和APP版都可以插入附件的
希望可以欣赏到艺术之乡法兰西原装艺术卡片 :)

【在 g***3 的大作中提到】
: 我上不来照片啊,以前也试过,否则就给大家看看了,
k**n
发帖数: 6198
82
天天看你拿阿法狗说事,你和他有仇阿:)
请帮他家算法的哲学思想总结一下,以广大人民群众喜闻乐见的普通话解释两下
不要乱说,有数学老师压阵的说

【在 t******l 的大作中提到】
: 我觉得如果已经是高中了,那知道娃自己的长处短处,找个合适的人生方向,在社会上
: 成功 survival,比硬拗某一两门课程,重要得多得多。
: 或者换一个角度看,这也是阿发狗重视重腹、而不拘泥于某一个孤立的无忧角的战略的
: 另一种实现方式。

k**n
发帖数: 6198
83
喔,看你这个解释,觉得你老大老美国发展一定很有后劲的说,这个社会他会赢

【在 g***3 的大作中提到】
: 这个上进心,就是自推动力了,,
: 老大是近一年,才开始爆发的,他去年就很灰心,甚至想到了最差的打算,,他现在对
: 妹妹的未来,也很认真,对小弟弟的未来,更是不能马虎,总是说我埋没了他的人才,
: 要让他去最好的学校,,
: 这个演变过程,我一直在观察,,
: 女儿是从小就不甘弱后,小时玩,如果输了,就要哭,,,现在小儿子,是会预测,如
: 果发现自己要输了,就不玩了,玩时,要想尽一切办法赢,,

f**********g
发帖数: 4709
84
首先你这个论点就有点儿问题。什么叫大多数女孩子到了高中就不行了。她们的学习能
力以及考试能力并没有改变,只不过小学初中考试简单,分辨不出来而已。
我举个例子简单量化一下,某个知识体系共100分格,多数男孩女孩的能力能达到80分
格,少数聪明的在90分格,极少数天才能handle满格。可是呢,小学初中时候只考了前
60分格,这时候的99分和90分其实并不能分辨一个人是不是聪明,是不是擅长这一科目
,可能那些天才还考不过大多数细心认真的女孩子。等到了高中考到90分格的时候,这
些能力不够的女孩子真心理解不了,思路就不在一条线上,所以花费大量时间精力也成
效甚微。这也是为什么高中要分科。学自己擅长的就好了。
其实每个人都有自己90分甚至无限接近满分的领域,哪怕是再不起眼的冷门偏门呢,在
现在这个社会,如果能发掘出来,也可以大放光彩

【在 x***1 的大作中提到】
: 啥原因导致大多数女孩子到了高中就开始不行了?仍然听话啊,不够大度,内敛,爱读
: 书,记忆力好,男孩子爱骚扰,但文科生女孩子dominate, 逻辑思维能力欠缺,走神。
: 大人物女的的确屈指可数。倒是生了娃以后,女的上进心就是0.

t******l
发帖数: 10908
85
阿发狗的哲学思想就是:“读好书,不求甚解。每有会意,便欣然忘记充电”。

:天天看你拿阿法狗说事,你和他有仇阿:)
g***3
发帖数: 471
86
好,我以后试试,我家那位出差,手机照片都是他给我输入电脑的,app太小,打字不
顺,
2个大孩子不能跟他们说,否则他们不肯的,,

【在 d**********h 的大作中提到】
: 放在附件里就可以了
: web版和APP版都可以插入附件的
: 希望可以欣赏到艺术之乡法兰西原装艺术卡片 :)

f**********g
发帖数: 4709
87
高中成绩重要,但并不是每一次考试每一科目的成绩都同样比重。
比如你女儿如果以后选了文科或者商科,那么高一时候的物理考试成绩就没那么重要。
申请材料里有很大可以解释发挥的余地。
又比如高一时候还在认识自己摸索方法的阶段,成绩不好,可是到了高二高三找到了适
合自己的道路,成绩一跃到了年级第一。以你女儿的文学水平,完全可以写出让人印象
深刻的essay。说不准比一直成绩好效果更好。
所以一两次考试成绩真心没有那么重要。不要给她太大压力。
另外既然喜欢纽约,学商科吧。

【在 g***3 的大作中提到】
: 是是,话是这么说,
: 但是高中以后每次考试分数,都是很重要的,是以后申请好学校的档案资料啊,
: 难到美国不需要,
: 女儿在英美系统,以后可以直接上美国英国大学,学校已经组织了好几次家长会,
: 就是讨论这些的,美国申请大学比法国早,高中平时所有分数是很重要的,不知到底真
: 假,

g***3
发帖数: 471
88
他现在对自己信心满满的,也很努力,他希望以后,妹妹弟弟能够在各个领域有起色,
然后他们很抱团,相互帮助,提携。

【在 k**n 的大作中提到】
: 喔,看你这个解释,觉得你老大老美国发展一定很有后劲的说,这个社会他会赢
g***3
发帖数: 471
89
解释很清楚,
但是男女擅长的领域相对来说,还是有所不同,,,这个是他们生理机构造成的,
做家长的,希望还是能够发现孩子的强项,

【在 f**********g 的大作中提到】
: 首先你这个论点就有点儿问题。什么叫大多数女孩子到了高中就不行了。她们的学习能
: 力以及考试能力并没有改变,只不过小学初中考试简单,分辨不出来而已。
: 我举个例子简单量化一下,某个知识体系共100分格,多数男孩女孩的能力能达到80分
: 格,少数聪明的在90分格,极少数天才能handle满格。可是呢,小学初中时候只考了前
: 60分格,这时候的99分和90分其实并不能分辨一个人是不是聪明,是不是擅长这一科目
: ,可能那些天才还考不过大多数细心认真的女孩子。等到了高中考到90分格的时候,这
: 些能力不够的女孩子真心理解不了,思路就不在一条线上,所以花费大量时间精力也成
: 效甚微。这也是为什么高中要分科。学自己擅长的就好了。
: 其实每个人都有自己90分甚至无限接近满分的领域,哪怕是再不起眼的冷门偏门呢,在
: 现在这个社会,如果能发掘出来,也可以大放光彩

x***1
发帖数: 999
90
高中试题难,就分辨出来啦,分辨出啥咧?对男孩子同样适用啊, 比如:等到了高中
考到90分格的时候,这些能力不够的男孩子真心理解不了,思路就不在一条线上,所以
花费大量时间精力也成效甚微。
你是说男女没有差别,算我理解错了。

【在 f**********g 的大作中提到】
: 首先你这个论点就有点儿问题。什么叫大多数女孩子到了高中就不行了。她们的学习能
: 力以及考试能力并没有改变,只不过小学初中考试简单,分辨不出来而已。
: 我举个例子简单量化一下,某个知识体系共100分格,多数男孩女孩的能力能达到80分
: 格,少数聪明的在90分格,极少数天才能handle满格。可是呢,小学初中时候只考了前
: 60分格,这时候的99分和90分其实并不能分辨一个人是不是聪明,是不是擅长这一科目
: ,可能那些天才还考不过大多数细心认真的女孩子。等到了高中考到90分格的时候,这
: 些能力不够的女孩子真心理解不了,思路就不在一条线上,所以花费大量时间精力也成
: 效甚微。这也是为什么高中要分科。学自己擅长的就好了。
: 其实每个人都有自己90分甚至无限接近满分的领域,哪怕是再不起眼的冷门偏门呢,在
: 现在这个社会,如果能发掘出来,也可以大放光彩

相关主题
觉得美帝数学进度慢的进来看一下请教:怎样说服孩儿他爸多关心孩子的学习?
分数应用题娃又在推算他的质数公式。。。
我招,我是猪大学录取率
进入Parenting版参与讨论
g***3
发帖数: 471
91
是是,她的essay总是得到老师很高的评价。她的历史地理老师是美国人,英国文学是
英国人。
以前她想去离家近的一个法国最好的预科高中,这个学校对高中每个成绩都很重视。
老大现在成绩是上了,但他去年成绩不够好,本来是可以去那个最最好的,
我们现在不给女儿压力,我就希望她开心,她是自己过不去,我总是说不要紧,越说,
她越觉得委屈,
我下周给她找了个专门提高学习方法的学习班,让她去试试。

【在 f**********g 的大作中提到】
: 高中成绩重要,但并不是每一次考试每一科目的成绩都同样比重。
: 比如你女儿如果以后选了文科或者商科,那么高一时候的物理考试成绩就没那么重要。
: 申请材料里有很大可以解释发挥的余地。
: 又比如高一时候还在认识自己摸索方法的阶段,成绩不好,可是到了高二高三找到了适
: 合自己的道路,成绩一跃到了年级第一。以你女儿的文学水平,完全可以写出让人印象
: 深刻的essay。说不准比一直成绩好效果更好。
: 所以一两次考试成绩真心没有那么重要。不要给她太大压力。
: 另外既然喜欢纽约,学商科吧。

x***1
发帖数: 999
92
纽约,脏乱差,阴森可怕。乞丐遍地,垃圾遍地,高楼林立,没有阳光,面目可憎,看
看就行了。美国真好的,是small town,干净,友好,舒服,设施齐全,房子便宜,也
不少挣,很多好的大学,除去超一流大学,都在这些small town.
time square, 应翻译为时代胡同。巴掌大的地方也叫广场。

,,

【在 g***3 的大作中提到】
: 女儿下个月就要16岁了,还不知道以后想干啥,,
: 但是她很想到英美读书,,她喜欢英美,我们来美国一次,孩子们很喜欢,尤其纽约,,
: 这个夏天,老大一得到准信,我们想来3个星期,到处走走,

g***3
发帖数: 471
93
我从我自己2个孩子身上谈谈体会啊,
我发现儿子到高中后,物理课上对那些光啊,度啊,量啊,电啊,理解比较快,女儿好
像就慢,
现在小儿子也是对这些,似乎天生就很喜欢,着迷,女儿就没有,,这个对他们来说似
乎就是区别,
这样的话,遇到考试,联想能力快的,就应答如流了,
但是女儿初中物理也是近乎满分的,大概是难易程度不同,,,

【在 x***1 的大作中提到】
: 高中试题难,就分辨出来啦,分辨出啥咧?对男孩子同样适用啊, 比如:等到了高中
: 考到90分格的时候,这些能力不够的男孩子真心理解不了,思路就不在一条线上,所以
: 花费大量时间精力也成效甚微。
: 你是说男女没有差别,算我理解错了。

f**********g
发帖数: 4709
94
嗯,我漏了一句,我个人认为,男女生智力分布差别不大。
我说的那一部分女孩子,就是原本能力不够因为考试简单才成绩靠前的那部分,其实是
false positive。同样分布区间的男孩子,都在打架斗殴逃课溜号呢,所以一直本色演
出,就没有这个先行再突然不行了的表现了。还有一部分false negative的男生,在高
中才test出来。其实聪明(只是说擅长学习和考试这一方面)的女生,都是从小学一路
轻松很行到大学。

【在 x***1 的大作中提到】
: 高中试题难,就分辨出来啦,分辨出啥咧?对男孩子同样适用啊, 比如:等到了高中
: 考到90分格的时候,这些能力不够的男孩子真心理解不了,思路就不在一条线上,所以
: 花费大量时间精力也成效甚微。
: 你是说男女没有差别,算我理解错了。

g***3
发帖数: 471
95
他们知道,每个地方,总有不平衡,差异大的地方,,
在美国那次美好的感觉,就是对人的印象,遇到的人,都是精神饱满,衣冠整洁,青年
男女非常有礼貌,也乐意助人,,我们在manhattan比较中心的地方,住了一个星期,
,每天在大街上走,观察那些匆匆的人群,孩子们很喜欢,,,他们喜欢运动,,喜欢
穿着干净利索,英俊高大的人群,喜欢当地的各色早餐,,喜欢领略一百多年前,就已
建成的这种现代气色,当然,肯定有象英国一样很胖的人,但是Manhattan不多,孩子
们觉得是一个世界上集会了最出色的人的地方。。。。
美国很大,需要时间去发现,,,

【在 x***1 的大作中提到】
: 纽约,脏乱差,阴森可怕。乞丐遍地,垃圾遍地,高楼林立,没有阳光,面目可憎,看
: 看就行了。美国真好的,是small town,干净,友好,舒服,设施齐全,房子便宜,也
: 不少挣,很多好的大学,除去超一流大学,都在这些small town.
: time square, 应翻译为时代胡同。巴掌大的地方也叫广场。
:
: ,,

g***3
发帖数: 471
96
谢谢大家参与,我现在基本有点底了,
一是,让女儿下周试试那个学习方法的课外班,
2是,还是帮她提醒时间概念,改善时间管理,
3是,再具体观察,最后做决定,来年是否去商科,
好在他们学校很灵活,如果发现孩子高2理科成绩不理想,可以中途就换去商科,
下线,睡了,
再次谢谢大家了。
f**********g
发帖数: 4709
97
要去small town的话,人家法国德国的就挺好,干嘛要千里迢迢的跑到美国来啊
十几岁精力体力智力旺盛的年轻人,到大城市拼搏一番挺好的,等到结婚养娃了再去
small town

【在 x***1 的大作中提到】
: 纽约,脏乱差,阴森可怕。乞丐遍地,垃圾遍地,高楼林立,没有阳光,面目可憎,看
: 看就行了。美国真好的,是small town,干净,友好,舒服,设施齐全,房子便宜,也
: 不少挣,很多好的大学,除去超一流大学,都在这些small town.
: time square, 应翻译为时代胡同。巴掌大的地方也叫广场。
:
: ,,

f**********g
发帖数: 4709
98
你这个说不要紧,不能是泛泛的表面的“没关系,你一直都很聪明,这次考不好下次就
好了”或者“不管怎么样妈妈都相信你支持你”这种,起不了任何安慰作用。要有建设
性意见的安慰,帮娃一起回顾分析试验一下,
为什么以前行现在不行了?
为什么平时行考试的时候不行?
是可以很轻松的行还是要很辛苦才行?
是只有现在行还是以后都行?(找两本高三甚至大学的专业书来翻一翻)
你周围也都是很厉害的人,忽略炫耀嫉妒不好意思等负面情绪,跟大家就具体问题心平
气和的探讨请教一下

【在 g***3 的大作中提到】
: 是是,她的essay总是得到老师很高的评价。她的历史地理老师是美国人,英国文学是
: 英国人。
: 以前她想去离家近的一个法国最好的预科高中,这个学校对高中每个成绩都很重视。
: 老大现在成绩是上了,但他去年成绩不够好,本来是可以去那个最最好的,
: 我们现在不给女儿压力,我就希望她开心,她是自己过不去,我总是说不要紧,越说,
: 她越觉得委屈,
: 我下周给她找了个专门提高学习方法的学习班,让她去试试。

i**e
发帖数: 19242
99
楼主开的楼
一直都挺欢乐的:)
i**e
发帖数: 19242
100
我个人更喜欢欧州的风格
我喜欢步行
喜欢街边的咖啡馆功克力店花店
喜欢街头艺人
喜欢小街小巷
喜欢一层店面楼上是住家的布局
喜欢阳台上的花花草草:)
还喜欢可以随便出入的教堂
欧州,在氛围上更接近中国,有人气

【在 g***3 的大作中提到】
: 他们知道,每个地方,总有不平衡,差异大的地方,,
: 在美国那次美好的感觉,就是对人的印象,遇到的人,都是精神饱满,衣冠整洁,青年
: 男女非常有礼貌,也乐意助人,,我们在manhattan比较中心的地方,住了一个星期,
: ,每天在大街上走,观察那些匆匆的人群,孩子们很喜欢,,,他们喜欢运动,,喜欢
: 穿着干净利索,英俊高大的人群,喜欢当地的各色早餐,,喜欢领略一百多年前,就已
: 建成的这种现代气色,当然,肯定有象英国一样很胖的人,但是Manhattan不多,孩子
: 们觉得是一个世界上集会了最出色的人的地方。。。。
: 美国很大,需要时间去发现,,,

相关主题
别鸡兔同笼了,来喝啤酒吧关于刷还是不刷
学校早晚要教的东西,早早学会了又如何?是不是任我儿子继续迷象棋
构建式数学在中国9岁, 如何报名考amc10
进入Parenting版参与讨论
v***s
发帖数: 4031
101
高中GPA高的女生很多,在我们这公校,女生全A的比男生多。那些全A的女生,但并不
说明她们全都是学得特别好,特别扎实。我大孩子在高中时,有两个全A的女生经常打
电话问问题,我孩子有一门课是B, 最后这两女孩子进的是藤校,我孩子进的是州大,
其中有个女孩进了大学还经常打电话问关于有机化学的修课。大学快毕业了,本来两人
都是要申请医学院的,其中一个放弃了,另一个,申请了,没有面试。

【在 f**********g 的大作中提到】
: 嗯,我漏了一句,我个人认为,男女生智力分布差别不大。
: 我说的那一部分女孩子,就是原本能力不够因为考试简单才成绩靠前的那部分,其实是
: false positive。同样分布区间的男孩子,都在打架斗殴逃课溜号呢,所以一直本色演
: 出,就没有这个先行再突然不行了的表现了。还有一部分false negative的男生,在高
: 中才test出来。其实聪明(只是说擅长学习和考试这一方面)的女生,都是从小学一路
: 轻松很行到大学。

f**********g
发帖数: 4709
102
那只能说你们高中的考试水平不够难
另外,你确定那个女孩不是看上了你儿子?/run

【在 v***s 的大作中提到】
: 高中GPA高的女生很多,在我们这公校,女生全A的比男生多。那些全A的女生,但并不
: 说明她们全都是学得特别好,特别扎实。我大孩子在高中时,有两个全A的女生经常打
: 电话问问题,我孩子有一门课是B, 最后这两女孩子进的是藤校,我孩子进的是州大,
: 其中有个女孩进了大学还经常打电话问关于有机化学的修课。大学快毕业了,本来两人
: 都是要申请医学院的,其中一个放弃了,另一个,申请了,没有面试。

v***s
发帖数: 4031
103
作为公校,学校的学的考的是按州的要求,但SAT,ACT,和AP的考试是比附近的公校要高
得多。

【在 f**********g 的大作中提到】
: 那只能说你们高中的考试水平不够难
: 另外,你确定那个女孩不是看上了你儿子?/run

i**e
发帖数: 19242
104
显然
太笨了
TA儿子没看上 //火箭

【在 f**********g 的大作中提到】
: 那只能说你们高中的考试水平不够难
: 另外,你确定那个女孩不是看上了你儿子?/run

x***1
发帖数: 999
105
人家法国德国的就挺好,为啥不允许人家怜新厌旧,千里迢迢跑到美国 small town。
“十几岁。。”,还在读书吧,拼啥博?弃学啦,学比尔盖茨?
别说打错了,是几十岁,没用啊,即使几十岁,为啥不能到small town拼搏。small
town 没拼搏?
"等到结婚养娃了再去", quit工作,到small town 喝西北风?
不好意思,纯属抬杠。

【在 f**********g 的大作中提到】
: 要去small town的话,人家法国德国的就挺好,干嘛要千里迢迢的跑到美国来啊
: 十几岁精力体力智力旺盛的年轻人,到大城市拼搏一番挺好的,等到结婚养娃了再去
: small town

x***1
发帖数: 999
106
我个人更喜欢美国 small town的风格
我喜欢步行
喜欢街边的咖啡馆功克力店花店
喜欢街头艺人
喜欢小街小巷
喜欢一层店面楼上是住家的布局
喜欢阳台上的花花草草:)
还喜欢可以随便出入的教堂
美国emall town,在氛围上更接近中国,有人气
也一样押韵哦。

【在 i**e 的大作中提到】
: 我个人更喜欢欧州的风格
: 我喜欢步行
: 喜欢街边的咖啡馆功克力店花店
: 喜欢街头艺人
: 喜欢小街小巷
: 喜欢一层店面楼上是住家的布局
: 喜欢阳台上的花花草草:)
: 还喜欢可以随便出入的教堂
: 欧州,在氛围上更接近中国,有人气

t******l
发帖数: 10908
107
归根到底也就是猴子从一棵树跳到附近的另一颗树,吱吱吱吱地表示很高兴。
BTW:你们说我应该买一块 Western Digital MyBook 2TB 呢?还是 Seagate Backup
Plus 2.001TB?谢谢。// run

【在 x***1 的大作中提到】
: 我个人更喜欢美国 small town的风格
: 我喜欢步行
: 喜欢街边的咖啡馆功克力店花店
: 喜欢街头艺人
: 喜欢小街小巷
: 喜欢一层店面楼上是住家的布局
: 喜欢阳台上的花花草草:)
: 还喜欢可以随便出入的教堂
: 美国emall town,在氛围上更接近中国,有人气
: 也一样押韵哦。

i**e
发帖数: 19242
108
not worth my time and energy to share my feelings about Europe
lesson learned:)
t******l
发帖数: 10908
109
GPA 可能还是女生更高一些,说实话这边美帝公立 GPA 评分系统也是彻底被猴子们搞
残了。
但是竞赛还是男生狠一些,这边六年级 AMC 8 靠前的分数都是男生为多。那些
invitational math event 一眼望过去女生更是寥寥无几。

【在 f**********g 的大作中提到】
: 那只能说你们高中的考试水平不够难
: 另外,你确定那个女孩不是看上了你儿子?/run

t******l
发帖数: 10908
110
属实。Once nerd,always nerd,don't count it。

【在 i**e 的大作中提到】
: not worth my time and energy to share my feelings about Europe
: lesson learned:)

相关主题
9岁, 如何报名考amc10算不算有数学天分
求科普: 奥数 vs. Math Olympiad vs. Math Count vs. Math circle vs. 超前学数学大家怎么推孩子的弱项?
有明天考AMC 8的吗?少年智力开发
进入Parenting版参与讨论
t******l
发帖数: 10908
111
属实
b***d
发帖数: 2695
112
我一直为这个很困惑, 从智商而言,并不见得是女生比男生差
但是为什嘛math competition基本都是男生?
百思钟得其解:
1. 女孩子比男孩子早成熟很多,这个年龄的女孩子各种drama,花很多心思在
非学习之外的事情上,比如穿着,比如social,比如drama,比如生理变化;
男娃还是傻乎乎得一门心思在数数上
2. 男孩子一根筋比较多, 比较focus。
所以在teen之后,各种drama都尘埃落定,比如大学,比如工作中,优秀女生
可能又开始突显。

【在 t******l 的大作中提到】
: GPA 可能还是女生更高一些,说实话这边美帝公立 GPA 评分系统也是彻底被猴子们搞
: 残了。
: 但是竞赛还是男生狠一些,这边六年级 AMC 8 靠前的分数都是男生为多。那些
: invitational math event 一眼望过去女生更是寥寥无几。

b***d
发帖数: 2695
113
THAT'S NOT 属实 AT ALL!

【在 t******l 的大作中提到】
: 属实。Once nerd,always nerd,don't count it。
t******l
发帖数: 10908
114
我的看法不太一样,我觉得在智商相等的情况下,根据大脑灰质总量守恒定理,越听话
的 GPA 越好,但竞赛会越差。
或者说如果你把大脑灰质都剪枝成当硬盘使了,那显然没有足够的硅片面积来搞超级数
学协处理器。
当然娃的心思也是一个因素,但我觉得不是 1st order 因素。因为再走心也是五十步
和一百步的差别,总不是一大堆 de facto 零蛋(de facto 零蛋的意思是六年级只做
对了 AMC 8 的送分题。AMC 10 另说,AMC 10 的送分题对六年级也不是那么容易的)。

【在 b***d 的大作中提到】
: 我一直为这个很困惑, 从智商而言,并不见得是女生比男生差
: 但是为什嘛math competition基本都是男生?
: 百思钟得其解:
: 1. 女孩子比男孩子早成熟很多,这个年龄的女孩子各种drama,花很多心思在
: 非学习之外的事情上,比如穿着,比如social,比如drama,比如生理变化;
: 男娃还是傻乎乎得一门心思在数数上
: 2. 男孩子一根筋比较多, 比较focus。
: 所以在teen之后,各种drama都尘埃落定,比如大学,比如工作中,优秀女生
: 可能又开始突显。

c***x
发帖数: 1826
115

你好啊!
很多老ID都回来了,这个版现在在线500人。还是izze有远见,一切顺其自然!
好像记得就智商分布来看,男女均值接近,但是方差男的比女的大。然后这个社会又有
很多机制,导致初始的微小差异被不断扩大,最后从收入等成就看,男的方差比女的就
更大。于是最后不管是expendable还是炮灰,都是男的。
好像潮水以前还给过一个生物进化学上的解释,人类祖先们,男的狩猎为主,需要长时
间集中注意力;女的在家养孩子嗑瓜子聊天,所以更善于语言和社交。
当然,统计规律和个体差异的关系,这个潮水和我争论过也不是一次两次了。
实在不行的话,最后再来一碗鸡汤: This is a man's world, but it will be
nothing without a woman or girl :-)

【在 b***d 的大作中提到】
: 我一直为这个很困惑, 从智商而言,并不见得是女生比男生差
: 但是为什嘛math competition基本都是男生?
: 百思钟得其解:
: 1. 女孩子比男孩子早成熟很多,这个年龄的女孩子各种drama,花很多心思在
: 非学习之外的事情上,比如穿着,比如social,比如drama,比如生理变化;
: 男娃还是傻乎乎得一门心思在数数上
: 2. 男孩子一根筋比较多, 比较focus。
: 所以在teen之后,各种drama都尘埃落定,比如大学,比如工作中,优秀女生
: 可能又开始突显。

t******l
发帖数: 10908
116
我觉得是现代各个社会都是 title inflation。严格而言物理属于 mathematical
science,也就是说 Algebra I / AMC 10 之前不存在物理,有的也就是相当于生理卫
生常识。
当然初中物理竞赛另说,那帮娃 AMC 10 估计也不是个事儿。

【在 g***3 的大作中提到】
: 我从我自己2个孩子身上谈谈体会啊,
: 我发现儿子到高中后,物理课上对那些光啊,度啊,量啊,电啊,理解比较快,女儿好
: 像就慢,
: 现在小儿子也是对这些,似乎天生就很喜欢,着迷,女儿就没有,,这个对他们来说似
: 乎就是区别,
: 这样的话,遇到考试,联想能力快的,就应答如流了,
: 但是女儿初中物理也是近乎满分的,大概是难易程度不同,,,

c***x
发帖数: 1826
117

下次去饭店吃饭,去河边散步,去乘公共汽车看下雨,记得叫上我,我补一下数学知识
补完数学课,我再去补文学课 :-)

【在 x***1 的大作中提到】
: 奇了怪咧,我每次去饭店吃饭,都要根据客流量算算这个饭店能不能盈利,联想到
: Bernoulli distribution, 其均值方差是多少,依次决定给小费的多少。看到河流,要
: 算算河流量有多大,能供多少人吃喝,联想到Reynolds number。看到公共汽车,要算
: 算政府每辆车得补贴多少,包括汽油价格,票价,路程,司机的工资等等。每次下雨,
: 我研究的对象是雨滴的形成,联想到Raleigh instability, 有序排列等等。

t******l
发帖数: 10908
118
或者说,牛顿之前无物理。。。当然,教生理卫生常识的亚里士多德很悲愤。。。

【在 t******l 的大作中提到】
: 我觉得是现代各个社会都是 title inflation。严格而言物理属于 mathematical
: science,也就是说 Algebra I / AMC 10 之前不存在物理,有的也就是相当于生理卫
: 生常识。
: 当然初中物理竞赛另说,那帮娃 AMC 10 估计也不是个事儿。

c***x
发帖数: 1826
119

终于有一个人和我一样,想看看“精美卡片”。。。

【在 d**********h 的大作中提到】
: 求一张“精美卡片”的照片,有包子答谢
: 老生常谈,追求理科答案而舍弃细枝末节的艺术卡片,得也?失也?

c***x
发帖数: 1826
120

说的很好啊。
父母纠结的可能是,如果孩子的raw talent里最擅长的方向,正好不是当下社会中回报
率最高的那些行业职业最需要的raw talent,应该怎么做的问题。
这是一个有一般意义的问题。

【在 f**********g 的大作中提到】
: 女孩子小学初中比男孩子成绩好,并不表示就比男孩聪明,更认真听话一些而已。潮水
: 说的decay,也并不是decay,只不过真实水平展示出来而已。
: 其实你家儿子应该挺聪明的,以前不开窍而已。女儿说不上来,我个人感觉她的特长更
: 偏文科一些
: 话又说回来,怎么算聪明呢?有人做题考试小菜一碟,有人轻松写出锦绣文章,有人为
: 人处事八面玲珑……都是聪明,找到适合自己的道路就好。你们还是多提供精神支持,
: 让她自己去尝试,不要执着于一时的成绩

相关主题
[转载] 为什么说usamo 简单分数应用题
二年级女儿的report我招,我是猪
觉得美帝数学进度慢的进来看一下请教:怎样说服孩儿他爸多关心孩子的学习?
进入Parenting版参与讨论
b***d
发帖数: 2695
121
智商相等的情况下,竞赛就是个体力活
任何事情,不focus,都做不好
assume智商相等,时间量一样多
谁花的时间多,谁投入,谁就会赢
女孩子会花很多时间和心思在别的事情上,
尤其在初中阶段,很多时候由不得自己
比如女娃,很emotional,一旦有被left out的感觉,
估计几天都没心思做数学题
自然竞赛的投入时间,精力,和集中度比男娃低
你不是女娃,你不懂

)。

【在 t******l 的大作中提到】
: 我的看法不太一样,我觉得在智商相等的情况下,根据大脑灰质总量守恒定理,越听话
: 的 GPA 越好,但竞赛会越差。
: 或者说如果你把大脑灰质都剪枝成当硬盘使了,那显然没有足够的硅片面积来搞超级数
: 学协处理器。
: 当然娃的心思也是一个因素,但我觉得不是 1st order 因素。因为再走心也是五十步
: 和一百步的差别,总不是一大堆 de facto 零蛋(de facto 零蛋的意思是六年级只做
: 对了 AMC 8 的送分题。AMC 10 另说,AMC 10 的送分题对六年级也不是那么容易的)。

c***x
发帖数: 1826
122

你看看这个写的怎么样?
http://www.mitbbs.com/article/LeisureTime/2020363_3.html

【在 k**n 的大作中提到】
: 天天看你拿阿法狗说事,你和他有仇阿:)
: 请帮他家算法的哲学思想总结一下,以广大人民群众喜闻乐见的普通话解释两下
: 不要乱说,有数学老师压阵的说

b***d
发帖数: 2695
123
是啊,看见几个老ID都发言了,我就蹦跶出来了 :)
貌似你是那谁的马甲 :)

【在 c***x 的大作中提到】
:
: 你看看这个写的怎么样?
: http://www.mitbbs.com/article/LeisureTime/2020363_3.html

t******l
发帖数: 10908
124
男生会打游戏。。。我家女娃一边打游戏一边看 Disney 情感 Junk TV,也不一定好多
少。。。
另一方面,我承认 6 年级 AMC 8 拿 15 分以上可能是需要很 focus。。。但你说的不
能解释拿 5 分 de facto 零蛋吧(其实选择题乱选前 20 题撞概率也有 4 分)。。。

【在 b***d 的大作中提到】
: 智商相等的情况下,竞赛就是个体力活
: 任何事情,不focus,都做不好
: assume智商相等,时间量一样多
: 谁花的时间多,谁投入,谁就会赢
: 女孩子会花很多时间和心思在别的事情上,
: 尤其在初中阶段,很多时候由不得自己
: 比如女娃,很emotional,一旦有被left out的感觉,
: 估计几天都没心思做数学题
: 自然竞赛的投入时间,精力,和集中度比男娃低
: 你不是女娃,你不懂

c***x
发帖数: 1826
125

楼主,我看下来,如果我能有你家这么三个孩子,会幸福到天天想写诗,想去远方。
当然,理想是能力的增函数,这个我完全理解,尊重;这个版的网友们也很给力。

【在 g***3 的大作中提到】
: 谢谢大家参与,我现在基本有点底了,
: 一是,让女儿下周试试那个学习方法的课外班,
: 2是,还是帮她提醒时间概念,改善时间管理,
: 3是,再具体观察,最后做决定,来年是否去商科,
: 好在他们学校很灵活,如果发现孩子高2理科成绩不理想,可以中途就换去商科,
: 下线,睡了,
: 再次谢谢大家了。

c***x
发帖数: 1826
126

再赞你一下。
当然,诗也需要是建设性的,远方也需要是理性的。

【在 f**********g 的大作中提到】
: 你这个说不要紧,不能是泛泛的表面的“没关系,你一直都很聪明,这次考不好下次就
: 好了”或者“不管怎么样妈妈都相信你支持你”这种,起不了任何安慰作用。要有建设
: 性意见的安慰,帮娃一起回顾分析试验一下,
: 为什么以前行现在不行了?
: 为什么平时行考试的时候不行?
: 是可以很轻松的行还是要很辛苦才行?
: 是只有现在行还是以后都行?(找两本高三甚至大学的专业书来翻一翻)
: 你周围也都是很厉害的人,忽略炫耀嫉妒不好意思等负面情绪,跟大家就具体问题心平
: 气和的探讨请教一下

t******l
发帖数: 10908
127
我作为民科表示这帖子写得不错,谢谢。

【在 c***x 的大作中提到】
:
: 再赞你一下。
: 当然,诗也需要是建设性的,远方也需要是理性的。

c***x
发帖数: 1826
128

都好,都喜欢,可以吗?
热爱美好的一切,一切的美好。

【在 x***1 的大作中提到】
: 我个人更喜欢美国 small town的风格
: 我喜欢步行
: 喜欢街边的咖啡馆功克力店花店
: 喜欢街头艺人
: 喜欢小街小巷
: 喜欢一层店面楼上是住家的布局
: 喜欢阳台上的花花草草:)
: 还喜欢可以随便出入的教堂
: 美国emall town,在氛围上更接近中国,有人气
: 也一样押韵哦。

t******l
发帖数: 10908
129
刚才 typo 了,改了过来。。。

【在 t******l 的大作中提到】
: 我作为民科表示这帖子写得不错,谢谢。
c***x
发帖数: 1826
130

难得你不Challenge我,我不Challenge你,不太习惯的说 :-)

【在 t******l 的大作中提到】
: 我作为民科表示这帖子写得不错,谢谢。
相关主题
娃又在推算他的质数公式。。。学校早晚要教的东西,早早学会了又如何?
大学录取率构建式数学在中国
别鸡兔同笼了,来喝啤酒吧关于刷还是不刷
进入Parenting版参与讨论
b***d
发帖数: 2695
131
你看看,你不是女娃,你就无法理解我说的
女娃主要表现在情绪波动很大上,
男娃打游戏,只是浪费时间,而不会影响他的情绪,也不会影响很多天
女娃看电视里的drama和生活里drama的影响是不一样的,可能会很多天
都没法focus。
为伊消得人憔悴衣带渐宽终不悔说的都是女的吧?!
还有,你有没有5分零蛋female vs male的stats啊?
btw,我觉得你是赤果果得racist //rocket run

【在 t******l 的大作中提到】
: 男生会打游戏。。。我家女娃一边打游戏一边看 Disney 情感 Junk TV,也不一定好多
: 少。。。
: 另一方面,我承认 6 年级 AMC 8 拿 15 分以上可能是需要很 focus。。。但你说的不
: 能解释拿 5 分 de facto 零蛋吧(其实选择题乱选前 20 题撞概率也有 4 分)。。。

t******l
发帖数: 10908
132
这么看来我娃看 Disney Junk TV 实际上相当于在打疫苗?也许这玩意儿真的是该疏不
该堵?
看来推娃还是要随其自然,也许 God 已经放了很多大人看不见的锦囊在路上让娃自己
捡起来。
应该让武器控制员看看这贴。。。

【在 b***d 的大作中提到】
: 你看看,你不是女娃,你就无法理解我说的
: 女娃主要表现在情绪波动很大上,
: 男娃打游戏,只是浪费时间,而不会影响他的情绪,也不会影响很多天
: 女娃看电视里的drama和生活里drama的影响是不一样的,可能会很多天
: 都没法focus。
: 为伊消得人憔悴衣带渐宽终不悔说的都是女的吧?!
: 还有,你有没有5分零蛋female vs male的stats啊?
: btw,我觉得你是赤果果得racist //rocket run

t******l
发帖数: 10908
133
所有参加的不分年级,得 5 分的 female 28.51% of total female,male 20.49% of
total male。

【在 b***d 的大作中提到】
: 你看看,你不是女娃,你就无法理解我说的
: 女娃主要表现在情绪波动很大上,
: 男娃打游戏,只是浪费时间,而不会影响他的情绪,也不会影响很多天
: 女娃看电视里的drama和生活里drama的影响是不一样的,可能会很多天
: 都没法focus。
: 为伊消得人憔悴衣带渐宽终不悔说的都是女的吧?!
: 还有,你有没有5分零蛋female vs male的stats啊?
: btw,我觉得你是赤果果得racist //rocket run

b***d
发帖数: 2695
134
难道不是很多玩意儿都只能疏不能堵???
我觉得凡事都要遵循客观规律,尊重事实
看junk tv,看过估计过两天就忘了
但是如果生活里被left out,或者被bully(女孩子)
可能会影响一生。。。。
你家LD怎么又换角色了? :)

【在 t******l 的大作中提到】
: 这么看来我娃看 Disney Junk TV 实际上相当于在打疫苗?也许这玩意儿真的是该疏不
: 该堵?
: 看来推娃还是要随其自然,也许 God 已经放了很多大人看不见的锦囊在路上让娃自己
: 捡起来。
: 应该让武器控制员看看这贴。。。

b***d
发帖数: 2695
135
牛! 赞数据男!
what's the confidence interval?

of

【在 t******l 的大作中提到】
: 所有参加的不分年级,得 5 分的 female 28.51% of total female,male 20.49% of
: total male。

b***d
发帖数: 2695
136
无聊,点进去看了一下
还以为是lz写的
原来是翻译nature的
:)
比较科普 :)

【在 c***x 的大作中提到】
:
: 难得你不Challenge我,我不Challenge你,不太习惯的说 :-)

x***1
发帖数: 999
137
欧洲风情万种的,的确比美国好多了。
美国就是从欧洲过来的不得志的人建的。看看美国的独立战争,英国打美国,比鸦片战
争还轻松,美国政府比清政府还无能。华盛顿领着几千个残兵败将,农忙季节还得回家
收庄稼,有空集结到一块,打一枪换个地方,乱哄哄,自由散漫,死个几十人,那叫重
大战役。最重大的发现是:打仗可以不用面对面,可以藏在树上,出其不意。英国觉得
和你美国打都丢人,算了,鸟不拉屎的地方,做生意的乌合之众,独立去吧,这才独立
了。

【在 c***x 的大作中提到】
:
: 难得你不Challenge我,我不Challenge你,不太习惯的说 :-)

b***d
发帖数: 2695
138
nature的科普文章,写得不错
不过deep learning一直号称有别与machine learning的
这里非要把人家放在ML之下 :)

【在 t******l 的大作中提到】
: 我作为民科表示这帖子写得不错,谢谢。
b***d
发帖数: 2695
139
现在你还觉得欧洲风情万种嘛?

【在 x***1 的大作中提到】
: 欧洲风情万种的,的确比美国好多了。
: 美国就是从欧洲过来的不得志的人建的。看看美国的独立战争,英国打美国,比鸦片战
: 争还轻松,美国政府比清政府还无能。华盛顿领着几千个残兵败将,农忙季节还得回家
: 收庄稼,有空集结到一块,打一枪换个地方,乱哄哄,自由散漫,死个几十人,那叫重
: 大战役。最重大的发现是:打仗可以不用面对面,可以藏在树上,出其不意。英国觉得
: 和你美国打都丢人,算了,鸟不拉屎的地方,做生意的乌合之众,独立去吧,这才独立
: 了。

x***1
发帖数: 999
140
所以欧洲人瞧不上美国人,美国人也以上欧洲学校为荣,美国没有纯正的贵族,就是个
发了财的土豪。

【在 x***1 的大作中提到】
: 欧洲风情万种的,的确比美国好多了。
: 美国就是从欧洲过来的不得志的人建的。看看美国的独立战争,英国打美国,比鸦片战
: 争还轻松,美国政府比清政府还无能。华盛顿领着几千个残兵败将,农忙季节还得回家
: 收庄稼,有空集结到一块,打一枪换个地方,乱哄哄,自由散漫,死个几十人,那叫重
: 大战役。最重大的发现是:打仗可以不用面对面,可以藏在树上,出其不意。英国觉得
: 和你美国打都丢人,算了,鸟不拉屎的地方,做生意的乌合之众,独立去吧,这才独立
: 了。

相关主题
关于刷还是不刷求科普: 奥数 vs. Math Olympiad vs. Math Count vs. Math circle vs. 超前学数学
是不是任我儿子继续迷象棋有明天考AMC 8的吗?
9岁, 如何报名考amc10算不算有数学天分
进入Parenting版参与讨论
b***d
发帖数: 2695
141
“美国人也以上欧洲学校为荣”,这都哪来的啊?
我肿么重来没有听说过这个?
一同事把儿子送欧洲去上学,有同事很困惑,
说为啥不上家门口的A,B,C?
有同事解惑:你以为A,B,C就那么好去啊?!

【在 x***1 的大作中提到】
: 所以欧洲人瞧不上美国人,美国人也以上欧洲学校为荣,美国没有纯正的贵族,就是个
: 发了财的土豪。

t******l
发帖数: 10908
142
看过两天就忘?。。。哈哈哈哈哈哈哈哈哈。。。你知道我娃翻来覆去看多少遍?我都
不好意思说。。。我就跟你说我们家的宽带是 150Mbps,Wireless 路由器是仨大号天
线阵 AC1900。。。我家两娃每人一个现役电脑外加一个现役 Samsung Tab A 9.7 inch
。。。因为都是急性子!!!

【在 b***d 的大作中提到】
: 难道不是很多玩意儿都只能疏不能堵???
: 我觉得凡事都要遵循客观规律,尊重事实
: 看junk tv,看过估计过两天就忘了
: 但是如果生活里被left out,或者被bully(女孩子)
: 可能会影响一生。。。。
: 你家LD怎么又换角色了? :)

b***d
发帖数: 2695
143
我说的是junk电视对情绪的影响,也就持续2,3天!!!!!
呼唤中文理解力!!!!!

看过两天就忘?。。。哈哈哈哈哈哈哈哈哈。。。你知道我娃翻来覆去看多少遍?我都
不好意思说。。。我就跟你说我们家的宽带是 150Mbps,Wireless 路由器是仨大号天
线阵 AC1900。。。我家两娃每人一个现役电脑外加一个现役 Samsung Tab A 9.7 inch
。。。因为都是急性子!!!

【在 t******l 的大作中提到】
: 看过两天就忘?。。。哈哈哈哈哈哈哈哈哈。。。你知道我娃翻来覆去看多少遍?我都
: 不好意思说。。。我就跟你说我们家的宽带是 150Mbps,Wireless 路由器是仨大号天
: 线阵 AC1900。。。我家两娃每人一个现役电脑外加一个现役 Samsung Tab A 9.7 inch
: 。。。因为都是急性子!!!

x***1
发帖数: 999
144
福利好,税高,没有美国这边这么拼,生活的好地方。我们在丹麦待过,神仙般的生活
,离开的时候,给诺贝尔奖委员会写了一封信,问能不能参加诺贝尔奖颁奖典礼,居然
回信了,说可以。到了美国,做梦都是丹麦的场景。

【在 b***d 的大作中提到】
: 现在你还觉得欧洲风情万种嘛?
b***d
发帖数: 2695
145
看来你对时事政治国际局势不太关心
所以你没有看懂我的问题 :)

【在 x***1 的大作中提到】
: 福利好,税高,没有美国这边这么拼,生活的好地方。我们在丹麦待过,神仙般的生活
: ,离开的时候,给诺贝尔奖委员会写了一封信,问能不能参加诺贝尔奖颁奖典礼,居然
: 回信了,说可以。到了美国,做梦都是丹麦的场景。

t******l
发帖数: 10908
146
每天都看怎么算?是不是要卷积?

【在 b***d 的大作中提到】
: 我说的是junk电视对情绪的影响,也就持续2,3天!!!!!
: 呼唤中文理解力!!!!!
:
: 看过两天就忘?。。。哈哈哈哈哈哈哈哈哈。。。你知道我娃翻来覆去看多少遍?我都
: 不好意思说。。。我就跟你说我们家的宽带是 150Mbps,Wireless 路由器是仨大号天
: 线阵 AC1900。。。我家两娃每人一个现役电脑外加一个现役 Samsung Tab A 9.7 inch
: 。。。因为都是急性子!!!

b***d
发帖数: 2695
147
哈哈哈,超赞卷积!!!!!但是我觉得幅值太不一样
还有老潮啊,我觉得你太在意竞赛成绩了!!!!!!
女孩子在teen的时候,我觉得家长应该更关注她们的心理变化
情绪波动很大,学校各种dynamics,女孩子里这时候会出现真正的
mean girl,会hurt其他女孩子,如果没有处理好,可能会影响孩子
一生。 这时候成绩神马的,应该放在次要地位, 孩子的性格,价值
观(女孩子的虚荣,攀比等)等才是这个阶段的focus。
等过了这个阶段,到了高中,自然就会回归到正常track,好好学习
seriously,自己的经历和很多家长的经验。

【在 t******l 的大作中提到】
: 每天都看怎么算?是不是要卷积?
x***1
发帖数: 999
148
你说穆斯林吧,不知道能把欧洲祸害成啥样。但相对还是少,宣传的有点过,穆斯林化
,不可能吧。北欧那地方,穆斯林还不爱去吧。

【在 b***d 的大作中提到】
: 看来你对时事政治国际局势不太关心
: 所以你没有看懂我的问题 :)

t******l
发帖数: 10908
149
不是我在意成绩。。。其实一般都是武器控制员更在意成绩。。。这不仅仅是我家,娃
相互之间都知道。。。
其实我对娃看点 junk TV 也没有到那么在意的程度,但是武器控制员同志。。。

【在 b***d 的大作中提到】
: 哈哈哈,超赞卷积!!!!!但是我觉得幅值太不一样
: 还有老潮啊,我觉得你太在意竞赛成绩了!!!!!!
: 女孩子在teen的时候,我觉得家长应该更关注她们的心理变化
: 情绪波动很大,学校各种dynamics,女孩子里这时候会出现真正的
: mean girl,会hurt其他女孩子,如果没有处理好,可能会影响孩子
: 一生。 这时候成绩神马的,应该放在次要地位, 孩子的性格,价值
: 观(女孩子的虚荣,攀比等)等才是这个阶段的focus。
: 等过了这个阶段,到了高中,自然就会回归到正常track,好好学习
: seriously,自己的经历和很多家长的经验。

b***d
发帖数: 2695
150
那也是你的责任,搞定武器控制员啊~
还有这个竞赛成绩啊,对娃的将来有多大的帮助?
比如我们某年面试的一个intern,拿了4,5块国际奥赛奖牌,金银铜都有
可是最后同志们都把他给据了,一个原因:social awkward。
你看,竞赛成绩这么好,却连个烂公司的intern都拿不到。。。

【在 t******l 的大作中提到】
: 不是我在意成绩。。。其实一般都是武器控制员更在意成绩。。。这不仅仅是我家,娃
: 相互之间都知道。。。
: 其实我对娃看点 junk TV 也没有到那么在意的程度,但是武器控制员同志。。。

相关主题
大家怎么推孩子的弱项?二年级女儿的report
少年智力开发觉得美帝数学进度慢的进来看一下
[转载] 为什么说usamo 简单分数应用题
进入Parenting版参与讨论
x***1
发帖数: 999
151
这是过去传统有钱人的看法,现在听起来好像胡说八道,不可思议,相当于中国有钱人
把孩子送到国外读书一样。欧洲想对保守很多,各种无意义的高校评比,洗脑,弄得美
国高校好象世界第一似的。但论文化熏陶,厚重感,气质,美国还真差的很远。

【在 b***d 的大作中提到】
: “美国人也以上欧洲学校为荣”,这都哪来的啊?
: 我肿么重来没有听说过这个?
: 一同事把儿子送欧洲去上学,有同事很困惑,
: 说为啥不上家门口的A,B,C?
: 有同事解惑:你以为A,B,C就那么好去啊?!

t******l
发帖数: 10908
152
问题你觉得人如果不参加奥赛,那就会自动 social intelligent 了?。。。我觉得更
大的可能是去昌平挖沙。。。
当然父母强迫娃参加奥赛,每天做 2+ 小时题目的,另说。

【在 b***d 的大作中提到】
: 那也是你的责任,搞定武器控制员啊~
: 还有这个竞赛成绩啊,对娃的将来有多大的帮助?
: 比如我们某年面试的一个intern,拿了4,5块国际奥赛奖牌,金银铜都有
: 可是最后同志们都把他给据了,一个原因:social awkward。
: 你看,竞赛成绩这么好,却连个烂公司的intern都拿不到。。。

l*******g
发帖数: 27064
153
死扣这种细节,大量时间花在无关紧要的东西上了
是不是你女儿很挑剔,用本子要自己喜欢的,什么桌子椅子摆放都有要求?

不同

【在 d****g 的大作中提到】
: "昨晚上,女儿要我帮她复习化学,我看她写了好多小卡片,,分别用不同的颜色,不同
: 的文字花式,很好看,就是时间上不知不觉花花流"
: --- 这个描述很仔细。很有警戒作用。。。学到了。

b***d
发帖数: 2695
154
从对科技社会的contribution来说,还真是美国的几个高校是世界第一
我个人觉得欧洲高校很好的一点是,phd是一件很serious的事情,非常严格
好多学校需要7,8年,而且defense也是极其严格;不像美国很多学校,
放水严重。
至于神马文化熏陶啊,厚重啊,气质啊,我觉得太虚幻了一些
受了高等教育,出来就是给社会做贡献的。不能做贡献,只摆气质,是不是
装了一点? //run

【在 x***1 的大作中提到】
: 这是过去传统有钱人的看法,现在听起来好像胡说八道,不可思议,相当于中国有钱人
: 把孩子送到国外读书一样。欧洲想对保守很多,各种无意义的高校评比,洗脑,弄得美
: 国高校好象世界第一似的。但论文化熏陶,厚重感,气质,美国还真差的很远。

x***1
发帖数: 999
155
这咋能说的女的呢?为伊,就是为了她嘛,男的想女的嘛。

【在 b***d 的大作中提到】
: 你看看,你不是女娃,你就无法理解我说的
: 女娃主要表现在情绪波动很大上,
: 男娃打游戏,只是浪费时间,而不会影响他的情绪,也不会影响很多天
: 女娃看电视里的drama和生活里drama的影响是不一样的,可能会很多天
: 都没法focus。
: 为伊消得人憔悴衣带渐宽终不悔说的都是女的吧?!
: 还有,你有没有5分零蛋female vs male的stats啊?
: btw,我觉得你是赤果果得racist //rocket run

b***d
发帖数: 2695
156
如果你发现不参加奥赛的social intelligence level远高于参加奥赛的,
你还要不要让她参加?
我可以像你保证,参加奥赛的,会损伤social intelligence level,
原因太简单了,别人都去practice,improve social intelligence level了
你却去刷题了。。。。
还有,开昌平沙矿的,基本都是没参加过奥赛的
参加奥赛的,倒是真有可能去昌平挖沙

【在 t******l 的大作中提到】
: 问题你觉得人如果不参加奥赛,那就会自动 social intelligent 了?。。。我觉得更
: 大的可能是去昌平挖沙。。。
: 当然父母强迫娃参加奥赛,每天做 2+ 小时题目的,另说。

t******l
发帖数: 10908
157
同学。。。不是我说您。。。联邦最高法院通过同性婚姻已经不是一天两天了。。。

【在 x***1 的大作中提到】
: 这咋能说的女的呢?为伊,就是为了她嘛,男的想女的嘛。
t******l
发帖数: 10908
158
你干脆说参加中国国家篮球队的,身高都远高于平均身高。。。马拉多纳当初参加中国
男篮就好了,一定玉树凌风。。。

【在 b***d 的大作中提到】
: 如果你发现不参加奥赛的social intelligence level远高于参加奥赛的,
: 你还要不要让她参加?
: 我可以像你保证,参加奥赛的,会损伤social intelligence level,
: 原因太简单了,别人都去practice,improve social intelligence level了
: 你却去刷题了。。。。
: 还有,开昌平沙矿的,基本都是没参加过奥赛的
: 参加奥赛的,倒是真有可能去昌平挖沙

x***1
发帖数: 999
159
来些实打实的。
最近几十年的contribution吧,二战以前,若贝尔奖有美国什么事?
就最近几十年,每年若贝尔奖不也有欧洲的吗?
二战以后,美国科技突飞猛进,不是一堆欧洲的科学家在起很大的作用吗?
等这些欧洲科学家的徒子徒孙死光了,真让人怀疑美国本土自己能不能培养出科学家。
倒是商业土豪一堆,插管吸血得心应手。

【在 b***d 的大作中提到】
: 从对科技社会的contribution来说,还真是美国的几个高校是世界第一
: 我个人觉得欧洲高校很好的一点是,phd是一件很serious的事情,非常严格
: 好多学校需要7,8年,而且defense也是极其严格;不像美国很多学校,
: 放水严重。
: 至于神马文化熏陶啊,厚重啊,气质啊,我觉得太虚幻了一些
: 受了高等教育,出来就是给社会做贡献的。不能做贡献,只摆气质,是不是
: 装了一点? //run

x***1
发帖数: 999
160
再看看美国现在的总统竞选,还不恶心死人。

【在 x***1 的大作中提到】
: 来些实打实的。
: 最近几十年的contribution吧,二战以前,若贝尔奖有美国什么事?
: 就最近几十年,每年若贝尔奖不也有欧洲的吗?
: 二战以后,美国科技突飞猛进,不是一堆欧洲的科学家在起很大的作用吗?
: 等这些欧洲科学家的徒子徒孙死光了,真让人怀疑美国本土自己能不能培养出科学家。
: 倒是商业土豪一堆,插管吸血得心应手。

相关主题
我招,我是猪大学录取率
请教:怎样说服孩儿他爸多关心孩子的学习?别鸡兔同笼了,来喝啤酒吧
娃又在推算他的质数公式。。。学校早晚要教的东西,早早学会了又如何?
进入Parenting版参与讨论
t******l
发帖数: 10908
161
另外这里还有费效比的问题,顺水推舟效率 10 倍,强扭瓜的效率是 1/10,这还没说
天花板问题。。。

【在 t******l 的大作中提到】
: 你干脆说参加中国国家篮球队的,身高都远高于平均身高。。。马拉多纳当初参加中国
: 男篮就好了,一定玉树凌风。。。

t******l
发帖数: 10908
162
另外这边讨论的是同样的时间,放在 学科竞赛 vs School Honor Roll。。。这
School Honor Roll 也不培养 social intelligence level。。。School Honor Roll
最多也就是培养 teacher's pet level,跟将来当工奴的能力相关性更大一些。
当然当工奴的能力,也就是听领导的话的能力,也是很重要。。。但这玩意儿等高中了
有了 critical thinking 的能力以后,再培养也不迟吧。。。小学就开始培养工奴能
力,这跟被洗脑的差别也不大不是?。。。 // run

【在 t******l 的大作中提到】
: 你干脆说参加中国国家篮球队的,身高都远高于平均身高。。。马拉多纳当初参加中国
: 男篮就好了,一定玉树凌风。。。

t******l
发帖数: 10908
163
但是另一方面,从 fit in social structure 的角度看,剪枝成硬盘比剪枝成超级数
学协处理器,可能更容易 fit into social structure。。。我觉得其中的深层原因可
能是前面所说的,现代 social structure 是一定程度反智的。。。原因很简单,超级
数学协处理器这枚硬币的另一面就是叛逆造反。。。其实都不用造反,都是超级数学协
处理器了,那显然没人愿意给 NIH 当千老了,NIH 不肯干。。。所以藤校入学标准显
然要一定程度的反智,才能保证硬盘型工奴的货源。。。当然也不能太反智,反智到连
试管的容积都读错,那也不行。。。所以现在的体系可能差不多正好。。。

)。

【在 t******l 的大作中提到】
: 我的看法不太一样,我觉得在智商相等的情况下,根据大脑灰质总量守恒定理,越听话
: 的 GPA 越好,但竞赛会越差。
: 或者说如果你把大脑灰质都剪枝成当硬盘使了,那显然没有足够的硅片面积来搞超级数
: 学协处理器。
: 当然娃的心思也是一个因素,但我觉得不是 1st order 因素。因为再走心也是五十步
: 和一百步的差别,总不是一大堆 de facto 零蛋(de facto 零蛋的意思是六年级只做
: 对了 AMC 8 的送分题。AMC 10 另说,AMC 10 的送分题对六年级也不是那么容易的)。

c***x
发帖数: 1826
164

Roll
发人深思 :-)
不是抬杠,你平心静气的说说,老师在Grade学生的时候,或者在设计grading system
的时候,到底应该reward什么?
ability ?
effort ?
outcome ?
participation ?
amenability ?
小学,中学,大学,应该有什么不同吗?

【在 t******l 的大作中提到】
: 另外这边讨论的是同样的时间,放在 学科竞赛 vs School Honor Roll。。。这
: School Honor Roll 也不培养 social intelligence level。。。School Honor Roll
: 最多也就是培养 teacher's pet level,跟将来当工奴的能力相关性更大一些。
: 当然当工奴的能力,也就是听领导的话的能力,也是很重要。。。但这玩意儿等高中了
: 有了 critical thinking 的能力以后,再培养也不迟吧。。。小学就开始培养工奴能
: 力,这跟被洗脑的差别也不大不是?。。。 // run

c***x
发帖数: 1826
165

interesting and insightful :-)
勤于思考的北京出租车司机侃大山,时不时也会冲进中南海的吧?

【在 t******l 的大作中提到】
: 我的看法不太一样,我觉得在智商相等的情况下,根据大脑灰质总量守恒定理,越听话
: 的 GPA 越好,但竞赛会越差。
: 或者说如果你把大脑灰质都剪枝成当硬盘使了,那显然没有足够的硅片面积来搞超级数
: 学协处理器。
: 当然娃的心思也是一个因素,但我觉得不是 1st order 因素。因为再走心也是五十步
: 和一百步的差别,总不是一大堆 de facto 零蛋(de facto 零蛋的意思是六年级只做
: 对了 AMC 8 的送分题。AMC 10 另说,AMC 10 的送分题对六年级也不是那么容易的)。

g***3
发帖数: 471
166
女儿自己也发现动作比较慢,因为有哥哥的参照,她希望我们能够帮助她,跟踪她的时
间掌握度,,
她成绩如果不好,就会马上跟我说,这样一下自己就轻松些,成绩好的,到不常说,,
还有她的自然科学,,法国称天和地自然科学,是班里最好的,这个在高中也算在理科
里,
就是生物科学,器官啥的,她也比较擅长,

【在 f**********g 的大作中提到】
: 你这个说不要紧,不能是泛泛的表面的“没关系,你一直都很聪明,这次考不好下次就
: 好了”或者“不管怎么样妈妈都相信你支持你”这种,起不了任何安慰作用。要有建设
: 性意见的安慰,帮娃一起回顾分析试验一下,
: 为什么以前行现在不行了?
: 为什么平时行考试的时候不行?
: 是可以很轻松的行还是要很辛苦才行?
: 是只有现在行还是以后都行?(找两本高三甚至大学的专业书来翻一翻)
: 你周围也都是很厉害的人,忽略炫耀嫉妒不好意思等负面情绪,跟大家就具体问题心平
: 气和的探讨请教一下

g***3
发帖数: 471
167
你是我在这个坛最敬佩喜欢的一个id,,你的俏皮和智慧,值得我好好学习。

【在 i**e 的大作中提到】
: 楼主开的楼
: 一直都挺欢乐的:)

g***3
发帖数: 471
168
这个专属于假期,,法国的问题,就在此,大部分人,一年4季,状态就如似在假期,,
一小部分人,想干点啥,就会受到很大的阻碍,,所以现在法国我们这里有点作为的年
轻人,都去了美国,英国,,,

【在 i**e 的大作中提到】
: 我个人更喜欢欧州的风格
: 我喜欢步行
: 喜欢街边的咖啡馆功克力店花店
: 喜欢街头艺人
: 喜欢小街小巷
: 喜欢一层店面楼上是住家的布局
: 喜欢阳台上的花花草草:)
: 还喜欢可以随便出入的教堂
: 欧州,在氛围上更接近中国,有人气

g***3
发帖数: 471
169
我的体会,,现在的初中的男生和女儿,都会面临一些同样的问题,
也许,女生,在小学高年级的时候,会开始比较注重穿着,打扮,然后一直继续下去,,
但是男生到了初中后,也会遇到,同样的问题,穿着上不合同时代的节拍,没有同伴的
集合,没有女孩子的注目,都会给有些特别的男生,带来困惑,,

【在 b***d 的大作中提到】
: 我一直为这个很困惑, 从智商而言,并不见得是女生比男生差
: 但是为什嘛math competition基本都是男生?
: 百思钟得其解:
: 1. 女孩子比男孩子早成熟很多,这个年龄的女孩子各种drama,花很多心思在
: 非学习之外的事情上,比如穿着,比如social,比如drama,比如生理变化;
: 男娃还是傻乎乎得一门心思在数数上
: 2. 男孩子一根筋比较多, 比较focus。
: 所以在teen之后,各种drama都尘埃落定,比如大学,比如工作中,优秀女生
: 可能又开始突显。

g***3
发帖数: 471
170
孩子越小,家长越能陪伴,塑造,越能给予孩子影响,让他们去,摸索,
我从怀第一孩子起,体会到自己,是逐渐的,翻天覆地的变化,提高,感受生活,感激
爱我的人,带给我的一切。
他们在这些年里,伴随着我,包括我的职业变迁过程,目睹我 日常情绪,精神面貌的
改善,,
我至从有了孩子后,尽量什么事,都带着他们在我的身边,让他们和我一起分享,快乐
或者有时的犹豫,厌烦, 我以前和国内做纺织工业贸易,经常出差,法国这里学校假
期又多,2个大孩子从我怀孕,到出生,到几个月,就被我带来带去,我检货的时候,
他们就在欧洲的供货商厂内办公室玩,看书,画画,在国内时,有更多的阿姨叔叔和他
们玩耍,,到吃饭的时候,就和我们一起去饭馆,他们了解国内的乡村和城市,,尤其
纺织厂都在10多年前江浙一带,有时要上厕所,工厂的条件,也不是那么好,,,,10
多年前,交通,也是挺复杂的,以前一个老客户,在一个岛上,每次要做船,拿着几大
箱子样品,2个大孩子,从小会帮些我,体谅我,也喜欢和大人玩,,,
等到老3出生后,有一个阶段,我什么都不想干,就想和宝宝呆一起,,然后他去幼儿
园后,我又改行,去上学,去学新的软件,遇到挫折,甚至哭泣,孩子们都看在眼里,
终于拿到文凭,开始工作,又遇到职场的残酷,工作时间长,无法照顾正在成长的孩子
们,老大那时正在初中,他觉得我不值得这样做,,他批评我放弃了以前的客户,,,
,,
然后,我又摸索买旧房翻新出租,,终于找到了自己家内家外的平衡,孩子们也很高兴
,经常听听我说说各种房客的事,我们忙的时候,他们也帮带带小弟弟,,,
孩子们,就是这么成长着,,

【在 c***x 的大作中提到】
:
: interesting and insightful :-)
: 勤于思考的北京出租车司机侃大山,时不时也会冲进中南海的吧?

相关主题
学校早晚要教的东西,早早学会了又如何?是不是任我儿子继续迷象棋
构建式数学在中国9岁, 如何报名考amc10
关于刷还是不刷求科普: 奥数 vs. Math Olympiad vs. Math Count vs. Math circle vs. 超前学数学
进入Parenting版参与讨论
g***3
发帖数: 471
171
美国趁两次世界大战,利用时机,迅速发展了自己,无论在经济模式,政治体制,军事
资源,科学技术等方面,固定了自己老大的地位,近十几年的第3次科技新革命,只可
能出现在美国这样的环境,,
法国很多知识分子,都很崇拜美国,,,,
法国最好学校的很多学生,都去美国,或想去美国交流,,
也许大学阶段初期,法国的基础文理工知识,比如数学还算扎实,一般理工学院,相比
美国又比较便宜,可能有美国人来欧洲上学。
我女儿现在班上,有些美国家庭的孩子,他们父母都在这里工作。

【在 x***1 的大作中提到】
: 所以欧洲人瞧不上美国人,美国人也以上欧洲学校为荣,美国没有纯正的贵族,就是个
: 发了财的土豪。

g***3
发帖数: 471
172
这个就是美国的优势,
它能吸引世界各地的人才,是个聚人才宝的地方,,
我老公姐姐的大儿子,以前在法国也是电脑方面的人才,在法国工作了几年,自己创了
业,最后他的公司还是给美国人买去了,自己也带着新婚的妻子去了美国,,,现在快
5年了,孩子也在那里出生,回来说,已经习惯美国了。
我朋友的一些孩子,也是在大学第3年去美国交流,然后有了点子创了公司,,在法国
也出了名,最后还是把公司移动美国发展了,,,
为啥法国留不住他们尼,,太可惜了,
我自己以前也有过小公司,,那个行政过程,环环节节,要你交这个那个,,比正真的
贸易更烦心,
现在更是变本加利,,,,

【在 x***1 的大作中提到】
: 来些实打实的。
: 最近几十年的contribution吧,二战以前,若贝尔奖有美国什么事?
: 就最近几十年,每年若贝尔奖不也有欧洲的吗?
: 二战以后,美国科技突飞猛进,不是一堆欧洲的科学家在起很大的作用吗?
: 等这些欧洲科学家的徒子徒孙死光了,真让人怀疑美国本土自己能不能培养出科学家。
: 倒是商业土豪一堆,插管吸血得心应手。

g***3
发帖数: 471
173
这个使我想起老大有一次哲学课后,回来我们讨论的一个主题,自由和约束
他说,老师给他们看一个老师很喜欢的爵士乐钢琴家的演奏,,他的那种发挥纯青自如
,自然奔放,传染给人那种自由,洒脱,欢快,忘我所以的感觉,,
能够达到这种地步,需要钢琴家以前长期的苦行僧似的锻炼,磨难,刻苦练习,,,
所以儿子说,都是相对的,他告诫弟弟,如果以后想获得自由,那么现在就要苦练,
我觉得做为父母,很多书本上的大套套,在孩子心智还没达到一定程度时,还是少说为
好,
最重要的,是和孩子心连心,让他们体会
在 cjdlx (五柳) 的大作中提到: 】
system
g***3
发帖数: 471
174
老大去年在理科班,他说,现在商科班的同学,以后会更有出席,
以前理科班的同学,就喜欢解题,生活上有些不拘小节,看见女同学,还会脸红,不会
大谈阔论,,,
现在的商科班,有几个特别出色的同学,,从小就满怀理想,,从小就在饭桌上和家人
探讨政治历史社会问题,,知道以后自己想干啥,意志力坚决,学习非常勤奋,有个同
学想成为税务法律师,现在已经在进修大学有关课程。
所以哥哥,一直想说服妹妹去读商科,,,

Roll

【在 t******l 的大作中提到】
: 另外这边讨论的是同样的时间,放在 学科竞赛 vs School Honor Roll。。。这
: School Honor Roll 也不培养 social intelligence level。。。School Honor Roll
: 最多也就是培养 teacher's pet level,跟将来当工奴的能力相关性更大一些。
: 当然当工奴的能力,也就是听领导的话的能力,也是很重要。。。但这玩意儿等高中了
: 有了 critical thinking 的能力以后,再培养也不迟吧。。。小学就开始培养工奴能
: 力,这跟被洗脑的差别也不大不是?。。。 // run

r*g
发帖数: 3159
175
请tutor.
还是没学懂,技术问题,用技术手段解决。去追求形而上的什么学习效率,太虚了。

【在 g***3 的大作中提到】
: 各位父母好,我是在法国的妈妈,去年,我在本坛发帖,求教家里2大孩子选学问题,
: 得到大家许多有益的启发,
: 平时经常潜水,这里大孩子父母有很多可鉴的经验,常常给我很多帮助。先谢谢大家。
: 目前,我又有一个新的问题,希望拿来和大家讨论。
: 我女儿目前上高一,是在一个英美系里,这个高中比较特殊,是法国和德国联合办的,
: 有英美系,是为家里父母一方英美人士,或法国常驻英美国家,回归人士的孩子而设的
: ,是需要通过国家竞考择优入取的。
: 这个系统的课程,比一般法国高中生每周多5个小时的学习。还是联合国下属学校之一
: ,课外活动比较多。
: 女儿从小学习到初中,比较顺利,她也比较好学。

g***3
发帖数: 471
176
正解,我现在就开始寻老师。

【在 r*g 的大作中提到】
: 请tutor.
: 还是没学懂,技术问题,用技术手段解决。去追求形而上的什么学习效率,太虚了。

k**n
发帖数: 6198
177
好奇一问,人脑神经元和机器神经元,到底哪个快?感觉目前谁也不能下定义
感觉最基本的就是人的大脑不能存储那么多数据(也许生物技术将来能开发90%的大脑灰
质,而这里面的大脑灰质有指数级的储藏功能).做个基因库或者干细胞或者未来RNA技术
啥啥啥的,把所有历史顶级棋手数据库转倒新生儿脑子里去,该数据库拒绝向机器传输(
所有九段高手临终前务必签协议,谨防巴拿马这类"内贼").再和阿发狗一战高下如何?
但是,人要生下来就变个智慧的小老头小老太的话,不需要同性恋来阻止人类繁殖而加速
灭亡,所有当妈的99%罢工辞职, 大概再也不想生娃娃了.生下来就比智商和情商,
TERRIBLE TWO的时候妈妈全趴下,养不活超级大神哈
看来世界的未来不是机器人,因为机器人其实是机器,而是人机器,人机器还会是人.

【在 c***x 的大作中提到】
:
: interesting and insightful :-)
: 勤于思考的北京出租车司机侃大山,时不时也会冲进中南海的吧?

k**n
发帖数: 6198
178
你娃们都要来美国的话,你会跟来吗?

,,

【在 g***3 的大作中提到】
: 这个专属于假期,,法国的问题,就在此,大部分人,一年4季,状态就如似在假期,,
: 一小部分人,想干点啥,就会受到很大的阻碍,,所以现在法国我们这里有点作为的年
: 轻人,都去了美国,英国,,,

g***3
发帖数: 471
179
你的这个设想,其实现在很多科学家已经在悄悄应用了,例如移植一种芯片到盲人眼球
里,让他们能够感触事物,,移植芯片到大脑里,能启动残废人运用他们残废的肢体,
,,
老大一直在问我这个问题,未来的人类,到底是哪一种,,
他相信人类要适应科技的发展,是人去创新科技,科技又造福于人类,,,
然后牵涉到一个哲学问题,人还是人吗? 未来的人的定义是什么?

【在 k**n 的大作中提到】
: 好奇一问,人脑神经元和机器神经元,到底哪个快?感觉目前谁也不能下定义
: 感觉最基本的就是人的大脑不能存储那么多数据(也许生物技术将来能开发90%的大脑灰
: 质,而这里面的大脑灰质有指数级的储藏功能).做个基因库或者干细胞或者未来RNA技术
: 啥啥啥的,把所有历史顶级棋手数据库转倒新生儿脑子里去,该数据库拒绝向机器传输(
: 所有九段高手临终前务必签协议,谨防巴拿马这类"内贼").再和阿发狗一战高下如何?
: 但是,人要生下来就变个智慧的小老头小老太的话,不需要同性恋来阻止人类繁殖而加速
: 灭亡,所有当妈的99%罢工辞职, 大概再也不想生娃娃了.生下来就比智商和情商,
: TERRIBLE TWO的时候妈妈全趴下,养不活超级大神哈
: 看来世界的未来不是机器人,因为机器人其实是机器,而是人机器,人机器还会是人.

g***3
发帖数: 471
180
以前,有时候,老大会说,妈妈,你很会带孩子,以后他有孩子了,希望能够得到我的
帮助,
现在,一阵子,说我意志有点偏激,希望我不能这样跟他未来的媳妇要求,,,
呵呵,老大现在觉得自己力量强大,学校里经济课什么都讲,比我说的正确的多,常常
强加于我他学到的理论知识,我只能慢慢耐心和他周旋。
以后,我不管到哪里,希望他们如果有需要我们的地方,我们能够在最近的地方,立刻
帮助到他们,尤其女儿,我自己是深有体会,我和父母关系不好,来源于童年,,,我
最大的理想,就是做一个和孩子的妈妈,,在女儿事业忙的时候,能够帮到她,这个很
重要。
他们现在都渐渐长大了,儿子下个月末,就18周岁了,,他们的心事,都藏不住,特别
是忧伤的时候,会马上跟我倾吐,,这个是我最希望的,,他们的朋友,常常请到家里
来,我们一起交谈,,
我把他们带来人世,最最希望的就是他们开心,自己感到自己的力量和价值,在这个社
会上以后能够占一个位置,帮助别人,,,
我们居住的地方,是一个很传统的城市,很多祖父母,都和他们的孩子住得很近,经常
是祖父母去到学校接孙辈孩子,,我一个朋友,2个女儿到美国工作后,和美国人结婚
,也生了孩子,我看她经常去美国照顾孩子,,,

【在 k**n 的大作中提到】
: 你娃们都要来美国的话,你会跟来吗?
:
: ,,

相关主题
有明天考AMC 8的吗?少年智力开发
算不算有数学天分[转载] 为什么说usamo 简单
大家怎么推孩子的弱项?二年级女儿的report
进入Parenting版参与讨论
C*****d
发帖数: 2253
181
你住在法国,有感到移民问题对自己生活有影响吗?
还是像美国一样,奇怪的区不去就好了?

【在 g***3 的大作中提到】
: 以前,有时候,老大会说,妈妈,你很会带孩子,以后他有孩子了,希望能够得到我的
: 帮助,
: 现在,一阵子,说我意志有点偏激,希望我不能这样跟他未来的媳妇要求,,,
: 呵呵,老大现在觉得自己力量强大,学校里经济课什么都讲,比我说的正确的多,常常
: 强加于我他学到的理论知识,我只能慢慢耐心和他周旋。
: 以后,我不管到哪里,希望他们如果有需要我们的地方,我们能够在最近的地方,立刻
: 帮助到他们,尤其女儿,我自己是深有体会,我和父母关系不好,来源于童年,,,我
: 最大的理想,就是做一个和孩子的妈妈,,在女儿事业忙的时候,能够帮到她,这个很
: 重要。
: 他们现在都渐渐长大了,儿子下个月末,就18周岁了,,他们的心事,都藏不住,特别

C*****d
发帖数: 2253
182
高一的化学,哪里需要那么多卡片?
要搞的话一张也都写完了。
搞那么多,就是完全没学懂。
这个完全没学懂,
也不是什么大不了的事,
顶多以后拿化学诺贝尔是没戏了。
如果成绩对她很重要,
就请个家教,
找最好的大学里学这个的女孩子,
稍稍补补,考试是没问题的。

【在 g***3 的大作中提到】
: 各位父母好,我是在法国的妈妈,去年,我在本坛发帖,求教家里2大孩子选学问题,
: 得到大家许多有益的启发,
: 平时经常潜水,这里大孩子父母有很多可鉴的经验,常常给我很多帮助。先谢谢大家。
: 目前,我又有一个新的问题,希望拿来和大家讨论。
: 我女儿目前上高一,是在一个英美系里,这个高中比较特殊,是法国和德国联合办的,
: 有英美系,是为家里父母一方英美人士,或法国常驻英美国家,回归人士的孩子而设的
: ,是需要通过国家竞考择优入取的。
: 这个系统的课程,比一般法国高中生每周多5个小时的学习。还是联合国下属学校之一
: ,课外活动比较多。
: 女儿从小学习到初中,比较顺利,她也比较好学。

C*****d
发帖数: 2253
183
这个就是我说了无数遍的,
家长不要瞎折腾孩子,
企图自己教这教那,
哪科不好,找个靠谱的家教补补是王道。

【在 g***3 的大作中提到】
: 现在数学他是班上第一,第2的,成绩几乎满分,我们找到一个巴黎高商在读的一年级
: 学生,帮他辅导,他在私教老师辅导下,做了练习,高3第一季,数学成绩就上来了,
: 自己信心大增,第2季,成绩更好,,,其他科目是,经济,,,哲学,,历史,地理
: ,,英语,汉语,西班牙语,和体育,,
: 他这次总分名列全校商科100多学生中,第3名,我们都为他高兴,,
: 所以,我觉得心理因素,占了大头,他们学校学生都是从小就很厉害的,

k**n
发帖数: 6198
184
这些生物技术的成功,从技术的实现以及和本质上基于对人体的了解,和人类对阿法购算
法的理解,差的数量级太大了.一个是新石器,一个是蒸汽机呢
虽然表现就是在成功率. 不要小看概率的提高, 貌似量变,其实质变. 不敢相信人类下
一个生物技术革命能有第一次工业革命加速度那么快. 因为我生活在新石器,暂时不想
风筝的事情,更不会想飞机:)
呼唤爱情算法快快出来就好了, 老娘我把未来女婿进门的照片一照,立刻告诉女儿们他
对你的爱到底得有多深. 养女娃最怕就是被臭小子们给骗了!!

【在 g***3 的大作中提到】
: 你的这个设想,其实现在很多科学家已经在悄悄应用了,例如移植一种芯片到盲人眼球
: 里,让他们能够感触事物,,移植芯片到大脑里,能启动残废人运用他们残废的肢体,
: ,,
: 老大一直在问我这个问题,未来的人类,到底是哪一种,,
: 他相信人类要适应科技的发展,是人去创新科技,科技又造福于人类,,,
: 然后牵涉到一个哲学问题,人还是人吗? 未来的人的定义是什么?

k**n
发帖数: 6198
185
我也有同样问题想问. 我英国的朋友说,他们北面的郡白人时不时要出来游行,因为穆斯
林太多了
-------------------
你住在法国,有感到移民问题对自己生活有影响吗?
还是像美国一样,奇怪的区不去就好了?

【在 g***3 的大作中提到】
: 以前,有时候,老大会说,妈妈,你很会带孩子,以后他有孩子了,希望能够得到我的
: 帮助,
: 现在,一阵子,说我意志有点偏激,希望我不能这样跟他未来的媳妇要求,,,
: 呵呵,老大现在觉得自己力量强大,学校里经济课什么都讲,比我说的正确的多,常常
: 强加于我他学到的理论知识,我只能慢慢耐心和他周旋。
: 以后,我不管到哪里,希望他们如果有需要我们的地方,我们能够在最近的地方,立刻
: 帮助到他们,尤其女儿,我自己是深有体会,我和父母关系不好,来源于童年,,,我
: 最大的理想,就是做一个和孩子的妈妈,,在女儿事业忙的时候,能够帮到她,这个很
: 重要。
: 他们现在都渐渐长大了,儿子下个月末,就18周岁了,,他们的心事,都藏不住,特别

g***3
发帖数: 471
186
影响还是很大的,我刚到法国时,是80年代末90年初,自己也还算一个小青年,学习,
打工,在巴黎走来走去,很少遇到非洲兄弟姐妹,如果不特意去巴黎北边,或东边的一
些区。
那时候,法国人很友好,我学习,工作,都挺顺利的,工作的环境,也是和国内打交道
,那时国内开始经济大发展,一些法国工业界人士,都喜欢我们这样的人,,自己生活
的环境,还是混在法国本地人里面,,我们每月去巴黎华人街采购,孩子们还是能分辨
出那里比较脏,乱,,,除了同胞,还有很多异族朋友,,除此外,我们居住的城市,
很少能够见到不同各大洲的人,,
我开始真正感受法国政治气候变化,是,最近10年,,左派政府强迫各个城市,建造社
会房,,,不然就每年罚款,,
象我们这里旧城,老城,没有空地,有些大家族的房子外卖,政府会强制性的收购,然
后造楼房,
然后,就开始住进一些家庭,然后这些家庭的孩子,就会一点点渗透到学校,,
我们这里还好了,凡尔赛是法国比较有名的古城,,教育水平比较高,居民整体程度也
高,,
能够被接受居住到这里的低收入家庭,相比其他地方,还是文明礼貌很多,,
但是公立小学,初中里面,闹事的,大多数还是那些家庭的孩子,他们人只占整体百分
个位数吧,但是很容易认出,,
然后他们就传统生很多孩子,,,那些孩子,尤其男孩子,大一点,家里住的小,就要
上街上玩,,
这里附近的居民,最怕他们上街,影响了周围居民的安宁,,,

【在 C*****d 的大作中提到】
: 你住在法国,有感到移民问题对自己生活有影响吗?
: 还是像美国一样,奇怪的区不去就好了?

k*******2
发帖数: 4163
187
个人的经验,提高学习效率要做好5定:
定时,定向,定质,定量,定法。
转给你一个老师的总结:
为此,同学们可采取“四定法”制定个人暑期学习计划。所谓“四定法”,即定向、定
量、定时、定法。“四定”之中要特别强调“定量”与“定时”,此两点是落实“循序
渐进”复习策略,提高暑期补习效率的关键因素。
“定向”,即补什么。补汉语语法知识。汉语语法知识包括古代汉语和现代汉语两类语
法知识。即补初中语文教学中被淡化的“文言词法与文言句法”、“现代汉语语法、修
辞知识”等内容,使弱化了的语法知识重新被我们重视起来,并加以修补和强化。
“定量”,即补多少。根据《高中语文课程标准》教学要求,针对《初中语文课程标准
》教学所欠缺的知识进行增补训练时,要立足于基础性运用的原则,尤其是因初中与高
中的要求不同而对高一新生高中语文学习造成困难的语法知识等内容。从现代汉语的词
、短语、句子、修辞等内容补起,明确梳理短语的结构、句子成分及结构类型、常用修
辞手法等内容。
“定时”,即用多少时间完成。针对补习的汉语语法内容和数量多少,结合自身学习需
要,科学设计补习时间安排表。因人而异,重在有效。
“定法”,即如何补。相传,古代有一中医为病人看病,一直不得其法。有一次无意中
按到病者患处,病者随即大声呼喊“啊!”中医对此加以针灸,痛症果然得以缓解。于
是这些特殊的痛点便被称为“阿是穴”。在此,我们不妨借鉴中医看病的做法,寻找同
学们的知识缺陷这一“阿是穴”,对症下药。即“头疼治头”、“脚疼治脚”,不搞“
一刀切”,以便使同学们的语文知识补习富有针对性和实效性。
他只是少说了一个“定质”,就是要达到的质量标准,如背单词,背100个能记住90个
还是95个,做十道题,能对8道还是10道,等。

【在 g***3 的大作中提到】
: 影响还是很大的,我刚到法国时,是80年代末90年初,自己也还算一个小青年,学习,
: 打工,在巴黎走来走去,很少遇到非洲兄弟姐妹,如果不特意去巴黎北边,或东边的一
: 些区。
: 那时候,法国人很友好,我学习,工作,都挺顺利的,工作的环境,也是和国内打交道
: ,那时国内开始经济大发展,一些法国工业界人士,都喜欢我们这样的人,,自己生活
: 的环境,还是混在法国本地人里面,,我们每月去巴黎华人街采购,孩子们还是能分辨
: 出那里比较脏,乱,,,除了同胞,还有很多异族朋友,,除此外,我们居住的城市,
: 很少能够见到不同各大洲的人,,
: 我开始真正感受法国政治气候变化,是,最近10年,,左派政府强迫各个城市,建造社
: 会房,,,不然就每年罚款,,

g***3
发帖数: 471
188
她化学还好,她习惯的学习方法,就是写字,记忆,,
现在课程内容多了,项目又多,时间上,我感觉她要改改了,
还有老师的问题,我也发现老公教,没效果,老公还喜欢教他们,,
老大至从跟了一个高商在读学生后,理解力,自信心,一下改变很多,
女儿,这个,我现在在找,就是一学年快要结束,老师不好找,,
还有女儿,刚才跟我说,他们上次物理化学考试成绩出来了,她第3名,很高兴,第一
名,是上面有个大3岁的姐姐,同样的高中,据说她们把以前的学习资料考试资料,都
留着,,女儿说,以后也要给弟弟留着。考第2名的,爸爸就是是管NASA的物理学家,,

【在 C*****d 的大作中提到】
: 高一的化学,哪里需要那么多卡片?
: 要搞的话一张也都写完了。
: 搞那么多,就是完全没学懂。
: 这个完全没学懂,
: 也不是什么大不了的事,
: 顶多以后拿化学诺贝尔是没戏了。
: 如果成绩对她很重要,
: 就请个家教,
: 找最好的大学里学这个的女孩子,
: 稍稍补补,考试是没问题的。

g***3
发帖数: 471
189
不会的啦,,和女儿从小交心,,她看准的人,准会被妈妈喜欢的啦,,
女儿这方面,比较听话,她听我们,哥哥的话,以前处的那个男朋友,就是我们跟她慢
慢分析后,几次她再观察,然后自己说,断了,,
现在班上据说,有一个很害羞的男孩,看上他,那个男孩妈妈是英国人,,她是负责小
学入学竟考的负责人,上次给我电话,然后说起他儿子和我女儿同班,我问女儿,才知
道的,,
呵呵,,女儿说他,太腼腆了,,,

【在 k**n 的大作中提到】
: 这些生物技术的成功,从技术的实现以及和本质上基于对人体的了解,和人类对阿法购算
: 法的理解,差的数量级太大了.一个是新石器,一个是蒸汽机呢
: 虽然表现就是在成功率. 不要小看概率的提高, 貌似量变,其实质变. 不敢相信人类下
: 一个生物技术革命能有第一次工业革命加速度那么快. 因为我生活在新石器,暂时不想
: 风筝的事情,更不会想飞机:)
: 呼唤爱情算法快快出来就好了, 老娘我把未来女婿进门的照片一照,立刻告诉女儿们他
: 对你的爱到底得有多深. 养女娃最怕就是被臭小子们给骗了!!

G********r
发帖数: 3161
190
看了这篇真的很感动,你是一个非常好的妈妈!

【在 g***3 的大作中提到】
: 孩子越小,家长越能陪伴,塑造,越能给予孩子影响,让他们去,摸索,
: 我从怀第一孩子起,体会到自己,是逐渐的,翻天覆地的变化,提高,感受生活,感激
: 爱我的人,带给我的一切。
: 他们在这些年里,伴随着我,包括我的职业变迁过程,目睹我 日常情绪,精神面貌的
: 改善,,
: 我至从有了孩子后,尽量什么事,都带着他们在我的身边,让他们和我一起分享,快乐
: 或者有时的犹豫,厌烦, 我以前和国内做纺织工业贸易,经常出差,法国这里学校假
: 期又多,2个大孩子从我怀孕,到出生,到几个月,就被我带来带去,我检货的时候,
: 他们就在欧洲的供货商厂内办公室玩,看书,画画,在国内时,有更多的阿姨叔叔和他
: 们玩耍,,到吃饭的时候,就和我们一起去饭馆,他们了解国内的乡村和城市,,尤其

相关主题
觉得美帝数学进度慢的进来看一下请教:怎样说服孩儿他爸多关心孩子的学习?
分数应用题娃又在推算他的质数公式。。。
我招,我是猪大学录取率
进入Parenting版参与讨论
x***1
发帖数: 999
191
了不起啊。

【在 g***3 的大作中提到】
: 孩子越小,家长越能陪伴,塑造,越能给予孩子影响,让他们去,摸索,
: 我从怀第一孩子起,体会到自己,是逐渐的,翻天覆地的变化,提高,感受生活,感激
: 爱我的人,带给我的一切。
: 他们在这些年里,伴随着我,包括我的职业变迁过程,目睹我 日常情绪,精神面貌的
: 改善,,
: 我至从有了孩子后,尽量什么事,都带着他们在我的身边,让他们和我一起分享,快乐
: 或者有时的犹豫,厌烦, 我以前和国内做纺织工业贸易,经常出差,法国这里学校假
: 期又多,2个大孩子从我怀孕,到出生,到几个月,就被我带来带去,我检货的时候,
: 他们就在欧洲的供货商厂内办公室玩,看书,画画,在国内时,有更多的阿姨叔叔和他
: 们玩耍,,到吃饭的时候,就和我们一起去饭馆,他们了解国内的乡村和城市,,尤其

C*****d
发帖数: 2253
192
我觉得你性格特别好,
小孩有样学样,也差不了。

【在 g***3 的大作中提到】
: 不会的啦,,和女儿从小交心,,她看准的人,准会被妈妈喜欢的啦,,
: 女儿这方面,比较听话,她听我们,哥哥的话,以前处的那个男朋友,就是我们跟她慢
: 慢分析后,几次她再观察,然后自己说,断了,,
: 现在班上据说,有一个很害羞的男孩,看上他,那个男孩妈妈是英国人,,她是负责小
: 学入学竟考的负责人,上次给我电话,然后说起他儿子和我女儿同班,我问女儿,才知
: 道的,,
: 呵呵,,女儿说他,太腼腆了,,,

g***3
发帖数: 471
193
定时,定向,定质,定量,定法。
记下了,好好琢磨

【在 k*******2 的大作中提到】
: 个人的经验,提高学习效率要做好5定:
: 定时,定向,定质,定量,定法。
: 转给你一个老师的总结:
: 为此,同学们可采取“四定法”制定个人暑期学习计划。所谓“四定法”,即定向、定
: 量、定时、定法。“四定”之中要特别强调“定量”与“定时”,此两点是落实“循序
: 渐进”复习策略,提高暑期补习效率的关键因素。
: “定向”,即补什么。补汉语语法知识。汉语语法知识包括古代汉语和现代汉语两类语
: 法知识。即补初中语文教学中被淡化的“文言词法与文言句法”、“现代汉语语法、修
: 辞知识”等内容,使弱化了的语法知识重新被我们重视起来,并加以修补和强化。
: “定量”,即补多少。根据《高中语文课程标准》教学要求,针对《初中语文课程标准

g***3
发帖数: 471
194
谢谢谢,理解万岁,我觉得孩子,就要将心比心,去理解他们,
老大初中最后2年,有些反复的,反叛期,有时候,对我出口很粗暴,甚至,还要动手
,他们内心,身体生理的变化,需要发泄,外面学校的压力,同学间的压力,还是无形
有的,,
我承认,我自己有时候也粗暴,特别在事多心急火撩得时候,我小时候,父亲对我很粗
暴,我在这种挣扎中长大,所以,一直也很要强,至从有了孩子,我一直想改善自己,
我对孩子们说,我就是他们的出气筒,他们最难的,最不想掩饰的,就跟妈妈来吧,
他们渐渐长大,会更珍惜我,,,

【在 G********r 的大作中提到】
: 看了这篇真的很感动,你是一个非常好的妈妈!
g***3
发帖数: 471
195
谢谢,,
我真心觉得,我们小时候,特别是我这种60年末,70年代初的国人,我们的父母在文革
中,受到的压力,都在我们身上有形无形的衍生,,
做为妈妈,最最重要的是,情绪上能够承受,承受你的孩子,不管他们丑,矮,笨,,
等等,,
孩子,是要看着妈妈的笑脸长大的,,,
这个我在2个大孩子小时候,还无法完全做到,,
在小儿子身上,我完全看到了效果,,我和老公认识快30年,,结婚快20年,,
我们一直在双方努力,就是情绪,,理解我们自己的童年,我们和自己父母的关系,

【在 x***1 的大作中提到】
: 了不起啊。
f**********g
发帖数: 4709
196
智商只是个笼统的概念啊,同样智商同样聪明的男孩女孩,可能一个更擅长抽象思维,
一个更擅长形象思维呢?
竞赛确实是更单纯focus在某个狭窄领域的一根筋行为,可能更适合男孩子一些。而大
学课程和工作,就要涉及更多非知识方面的因素,聪明的女孩子更容易安排协调好

【在 b***d 的大作中提到】
: 我一直为这个很困惑, 从智商而言,并不见得是女生比男生差
: 但是为什嘛math competition基本都是男生?
: 百思钟得其解:
: 1. 女孩子比男孩子早成熟很多,这个年龄的女孩子各种drama,花很多心思在
: 非学习之外的事情上,比如穿着,比如social,比如drama,比如生理变化;
: 男娃还是傻乎乎得一门心思在数数上
: 2. 男孩子一根筋比较多, 比较focus。
: 所以在teen之后,各种drama都尘埃落定,比如大学,比如工作中,优秀女生
: 可能又开始突显。

g***3
发帖数: 471
197
老大学了弗若伊的的3个我,,我们还去了维尔纳他的旧居,
老大,会分析妈妈,,我国内的姐姐比我大10岁,姐夫从我4岁就认识我,,他们很喜
欢我的孩子,
他们经常跟他说妈妈的小时候,,,和我自己认为的2样,,,然后,老大会从别人对
妈妈的评价里,自己得出妈妈的性格来由,,很有意义,,所以,我真觉得养孩子,是
世界上最最美好的事,,

【在 C*****d 的大作中提到】
: 我觉得你性格特别好,
: 小孩有样学样,也差不了。

i**e
发帖数: 19242
198
谢谢,谢谢!不敢当啊

【在 g***3 的大作中提到】
: 你是我在这个坛最敬佩喜欢的一个id,,你的俏皮和智慧,值得我好好学习。
x***1
发帖数: 999
199
乐一下,看看美国这边总统竞选的一些幺蛾子:
现在被一些华人无比崇拜支持的可爱的救世主富翁川普先生,在共和党内竞选中一路领
先:
1,和他一起竞选的一个女士,前HP的CEO,川普说:看你的脸,谁会投你的票。
2,川普先生认为对堕胎的女士要惩罚,后认为为对进行堕胎的医生要惩罚,后又认为
先维持现有法律不变,等他上台重新修法。
3,和FOX电台一个女主持人势不两立,川普认为在辩论赛上这个女主持人故意刁难他,
拒绝参加她主持的debat。
4,川普先生娶一模特,被对手爆出裸照,川普先生想出一绝招,比美,同时贴出对手
夫人和自己模特夫人的大头照,说:此时无声胜有声。
5,每次debat, 川普挂在嘴边的最得意的创作:修墙,在墨西哥边界,还得让墨西哥出
钱。
6,有次辩论,主持问,对15万亿美元的deficit,你有啥办法解决,川普先生说,我有
一揽子节省计划,可解决赤字问题,主持人又问,你的这些计划,加起来不到1000亿美
元,咋能解决15万亿赤字?川普先生说:你刁难我。这个主持人就是FOX电台那个女士。
7,移民问题上,川普先生主张把国内一千万非法移民见一个遣返一个,直到赶完为止
,凡穆斯林,不能给签证。
8,组建最强大军队,川普先生认为,美国为韩国和日本提供了军事保护,韩国和日本
必须出钱供养这些军队。
9,川普认为,既然朝鲜搞了原子弹,韩国和日本也要造原子弹,以达到地区军事平衡。
结论:我选川普先生,为啥?其他候选人,比川普还恶心。

【在 g***3 的大作中提到】
: 影响还是很大的,我刚到法国时,是80年代末90年初,自己也还算一个小青年,学习,
: 打工,在巴黎走来走去,很少遇到非洲兄弟姐妹,如果不特意去巴黎北边,或东边的一
: 些区。
: 那时候,法国人很友好,我学习,工作,都挺顺利的,工作的环境,也是和国内打交道
: ,那时国内开始经济大发展,一些法国工业界人士,都喜欢我们这样的人,,自己生活
: 的环境,还是混在法国本地人里面,,我们每月去巴黎华人街采购,孩子们还是能分辨
: 出那里比较脏,乱,,,除了同胞,还有很多异族朋友,,除此外,我们居住的城市,
: 很少能够见到不同各大洲的人,,
: 我开始真正感受法国政治气候变化,是,最近10年,,左派政府强迫各个城市,建造社
: 会房,,,不然就每年罚款,,

f**********g
发帖数: 4709
200
提供一个solution:多生几个娃
据我的观察啊,如果只有一两个娃,父母好像就会特别担心万一娃不成材怎么办,被耽
误了怎么办,尽最大所能的让娃成长到70-90分。可是这又有问题,因为你抹去了<60分
那个区间,>90分那个区间也一并被抹去了。娃的“正好不是当下社会中回报率最高的
那些行业职业最需要的raw talent”就被扼杀了。
但是有三个以上娃的,一般到了老三老四,父母就会很开明,随便他要做什么,能健康
成长,养活自己就行了。其实在美国这个地方,但凡是高中毕业的娃,都能找到个五六
万的工作养活自己。更不用说中国娃怎么都能上个地区(非社区)college。这样子反
而给娃更大的自由度。
这个区别,是为人父母的天性。除非有大智慧或者经历不寻常的父母才能免俗。
第二个solution:父母自推为成功人士。
将来娃就是富二代官二代,就算要以画画/弹琴/做义工/献身科研……为职业,父母也
养的起。你给娃存上10M的基金当生活费,就可以放心让娃发展“正好不是当下社会中
回报率最高的那些行业职业最需要的raw talent”了。

【在 c***x 的大作中提到】
:
: interesting and insightful :-)
: 勤于思考的北京出租车司机侃大山,时不时也会冲进中南海的吧?

相关主题
别鸡兔同笼了,来喝啤酒吧关于刷还是不刷
学校早晚要教的东西,早早学会了又如何?是不是任我儿子继续迷象棋
构建式数学在中国9岁, 如何报名考amc10
进入Parenting版参与讨论
f**********g
发帖数: 4709
201
嗯,而且在感情方面,女孩开窍更早。一但开窍了,会非常影响竞赛这种事情上面投入
的心思。男孩子,尤其是有竞赛资质的“nerd”(我说的是至少national那个level,
nerd非贬义),在这方面开窍尤其晚

【在 b***d 的大作中提到】
: 智商相等的情况下,竞赛就是个体力活
: 任何事情,不focus,都做不好
: assume智商相等,时间量一样多
: 谁花的时间多,谁投入,谁就会赢
: 女孩子会花很多时间和心思在别的事情上,
: 尤其在初中阶段,很多时候由不得自己
: 比如女娃,很emotional,一旦有被left out的感觉,
: 估计几天都没心思做数学题
: 自然竞赛的投入时间,精力,和集中度比男娃低
: 你不是女娃,你不懂

g***3
发帖数: 471
202
谢谢经验,老大现在大致是这样做的,
他也给妹妹介绍,,妹妹虽然shi,还是不太有效,大概我的心理也有点因素,我老看
她在学习,,她锻炼身体都没时间,,不希望她老是做作业啊,,,
她的房间乱,都没时间整理,衣服又多,,她喜欢画画,画料,各种笔,到处都是,她
又喜欢缝制衣服,自己喜欢的式样,从网上找来自己做,,所以布料,材料,,画,,
到处是,
她的时间需要有效利用,,

【在 k*******2 的大作中提到】
: 个人的经验,提高学习效率要做好5定:
: 定时,定向,定质,定量,定法。
: 转给你一个老师的总结:
: 为此,同学们可采取“四定法”制定个人暑期学习计划。所谓“四定法”,即定向、定
: 量、定时、定法。“四定”之中要特别强调“定量”与“定时”,此两点是落实“循序
: 渐进”复习策略,提高暑期补习效率的关键因素。
: “定向”,即补什么。补汉语语法知识。汉语语法知识包括古代汉语和现代汉语两类语
: 法知识。即补初中语文教学中被淡化的“文言词法与文言句法”、“现代汉语语法、修
: 辞知识”等内容,使弱化了的语法知识重新被我们重视起来,并加以修补和强化。
: “定量”,即补多少。根据《高中语文课程标准》教学要求,针对《初中语文课程标准

g***3
发帖数: 471
203
女儿的美国老师,现在一说到缺谱,就说如果他当选美国总统,以后就再也不回美国了,
呵呵

【在 x***1 的大作中提到】
: 乐一下,看看美国这边总统竞选的一些幺蛾子:
: 现在被一些华人无比崇拜支持的可爱的救世主富翁川普先生,在共和党内竞选中一路领
: 先:
: 1,和他一起竞选的一个女士,前HP的CEO,川普说:看你的脸,谁会投你的票。
: 2,川普先生认为对堕胎的女士要惩罚,后认为为对进行堕胎的医生要惩罚,后又认为
: 先维持现有法律不变,等他上台重新修法。
: 3,和FOX电台一个女主持人势不两立,川普认为在辩论赛上这个女主持人故意刁难他,
: 拒绝参加她主持的debat。
: 4,川普先生娶一模特,被对手爆出裸照,川普先生想出一绝招,比美,同时贴出对手
: 夫人和自己模特夫人的大头照,说:此时无声胜有声。

C*****d
发帖数: 2253
204
堕胎这个事,你这么说不太好,如果没有follow前因后果的同学,可能就造成误解了。
毕竟上下文是“如果现在的法律规定堕胎犯法,然后犯法的人要不要受惩罚”,trump
在堕胎问题上是非常中立的,他还在竞选中间说过planned parenthood为女性做了很多
事。这在别的共和党候选人身上是没可能的。

【在 x***1 的大作中提到】
: 乐一下,看看美国这边总统竞选的一些幺蛾子:
: 现在被一些华人无比崇拜支持的可爱的救世主富翁川普先生,在共和党内竞选中一路领
: 先:
: 1,和他一起竞选的一个女士,前HP的CEO,川普说:看你的脸,谁会投你的票。
: 2,川普先生认为对堕胎的女士要惩罚,后认为为对进行堕胎的医生要惩罚,后又认为
: 先维持现有法律不变,等他上台重新修法。
: 3,和FOX电台一个女主持人势不两立,川普认为在辩论赛上这个女主持人故意刁难他,
: 拒绝参加她主持的debat。
: 4,川普先生娶一模特,被对手爆出裸照,川普先生想出一绝招,比美,同时贴出对手
: 夫人和自己模特夫人的大头照,说:此时无声胜有声。

f**********g
发帖数: 4709
205
唉,打游戏本身也是一件目标单一行为单一非常需要focus的事情。从男孩女孩喜欢的
东西就能看出来,大脑结构或者思维方式还是很不一样的。
我们之前讨论的本来也是80分以上这个区间的女孩子。80-90分的女孩子,到了高中因
为不擅长抽象思维在数理化方面表现不够理想,也就是你常说的“decay”。而90分以
上的女孩子,因为青春期外力干扰等问题在竞赛方面表现不如男孩子,但是学校课程还
是可以轻松handle的,另外大学学业以及工作以后的表现也毫不逊色。
而且说到竞赛,我个人认为,好歹也要到AIME这个level的才能叫做竞赛

【在 t******l 的大作中提到】
: 男生会打游戏。。。我家女娃一边打游戏一边看 Disney 情感 Junk TV,也不一定好多
: 少。。。
: 另一方面,我承认 6 年级 AMC 8 拿 15 分以上可能是需要很 focus。。。但你说的不
: 能解释拿 5 分 de facto 零蛋吧(其实选择题乱选前 20 题撞概率也有 4 分)。。。

f**********g
发帖数: 4709
206
hand一个
我昨天的第一感想就是,好羡慕楼主有三个娃啊,而且都贴心又懂事。老大还是个很有
责任心有担当的大哥,羡慕死我了

【在 c***x 的大作中提到】
:
: interesting and insightful :-)
: 勤于思考的北京出租车司机侃大山,时不时也会冲进中南海的吧?

x***1
发帖数: 999
207
我看了那个采访,当时两个人都有些急,youtube上应该有,主持人穷追不舍,Trump迫
不得已说illegal的要punishment,我当时觉得没有错啊,后来三番五次的改变,就演
化成现在的版本:Trump要惩罚堕胎的人。

trump

【在 C*****d 的大作中提到】
: 堕胎这个事,你这么说不太好,如果没有follow前因后果的同学,可能就造成误解了。
: 毕竟上下文是“如果现在的法律规定堕胎犯法,然后犯法的人要不要受惩罚”,trump
: 在堕胎问题上是非常中立的,他还在竞选中间说过planned parenthood为女性做了很多
: 事。这在别的共和党候选人身上是没可能的。

f**********g
发帖数: 4709
208
哈哈,他家姑娘还没到能体会drama的时候啊。而且听潮水的描述,还是个比较迟钝的
假小子。所以这一天还要来的晚一些。
不过,你总是能等到看潮水纠结抓狂的那一天的

【在 b***d 的大作中提到】
: 你看看,你不是女娃,你就无法理解我说的
: 女娃主要表现在情绪波动很大上,
: 男娃打游戏,只是浪费时间,而不会影响他的情绪,也不会影响很多天
: 女娃看电视里的drama和生活里drama的影响是不一样的,可能会很多天
: 都没法focus。
: 为伊消得人憔悴衣带渐宽终不悔说的都是女的吧?!
: 还有,你有没有5分零蛋female vs male的stats啊?
: btw,我觉得你是赤果果得racist //rocket run

f**********g
发帖数: 4709
209
你要以发展的眼光看待问题啊,三代出贵族,美国从二战开始富起来,不止三代了啊

【在 b***d 的大作中提到】
: “美国人也以上欧洲学校为荣”,这都哪来的啊?
: 我肿么重来没有听说过这个?
: 一同事把儿子送欧洲去上学,有同事很困惑,
: 说为啥不上家门口的A,B,C?
: 有同事解惑:你以为A,B,C就那么好去啊?!

k**n
发帖数: 6198
210
哈哈,看现场就知道,他是被那个尖嘴猴腮的吊进去的

trump

【在 C*****d 的大作中提到】
: 堕胎这个事,你这么说不太好,如果没有follow前因后果的同学,可能就造成误解了。
: 毕竟上下文是“如果现在的法律规定堕胎犯法,然后犯法的人要不要受惩罚”,trump
: 在堕胎问题上是非常中立的,他还在竞选中间说过planned parenthood为女性做了很多
: 事。这在别的共和党候选人身上是没可能的。

相关主题
9岁, 如何报名考amc10算不算有数学天分
求科普: 奥数 vs. Math Olympiad vs. Math Count vs. Math circle vs. 超前学数学大家怎么推孩子的弱项?
有明天考AMC 8的吗?少年智力开发
进入Parenting版参与讨论
x***1
发帖数: 999
211
好多美国人对Trump不喜欢,总统权力受很大限制,要干个大事情太难。我觉得他很有
趣,感觉比其他政客更能维护美国人的利益。

了,

【在 g***3 的大作中提到】
: 女儿的美国老师,现在一说到缺谱,就说如果他当选美国总统,以后就再也不回美国了,
: 呵呵

g***3
发帖数: 471
212
被你这么一解释,就明了多了,老大上次也说,缺谱说对堕胎的女人要判刑,这也太极
端了,

trump

【在 C*****d 的大作中提到】
: 堕胎这个事,你这么说不太好,如果没有follow前因后果的同学,可能就造成误解了。
: 毕竟上下文是“如果现在的法律规定堕胎犯法,然后犯法的人要不要受惩罚”,trump
: 在堕胎问题上是非常中立的,他还在竞选中间说过planned parenthood为女性做了很多
: 事。这在别的共和党候选人身上是没可能的。

f**********g
发帖数: 4709
213
其实潮水这也不失为一种战略方式。
喜欢non-fiction的女孩子,一般开窍比较晚,更容易错过那段特别drama的时期,反而
受伤害比较小。我初中的时候都拿男孩子当哥们儿的,不管学习好的体育好的还是打架
斗殴逃学辍学的,所以虽然跟全年级八个班的男孩女孩都混的很熟,愣是没体会到一派
祥和之下的暗潮汹涌。等到了高中聊起来才知道初中时候的各种drama

【在 b***d 的大作中提到】
: 哈哈哈,超赞卷积!!!!!但是我觉得幅值太不一样
: 还有老潮啊,我觉得你太在意竞赛成绩了!!!!!!
: 女孩子在teen的时候,我觉得家长应该更关注她们的心理变化
: 情绪波动很大,学校各种dynamics,女孩子里这时候会出现真正的
: mean girl,会hurt其他女孩子,如果没有处理好,可能会影响孩子
: 一生。 这时候成绩神马的,应该放在次要地位, 孩子的性格,价值
: 观(女孩子的虚荣,攀比等)等才是这个阶段的focus。
: 等过了这个阶段,到了高中,自然就会回归到正常track,好好学习
: seriously,自己的经历和很多家长的经验。

i**e
发帖数: 19242
214
呵呵,这娇撒的:)
米国也有不少人半开玩笑半撒娇地说,Trump当选,俺要去加拿大:)

了,

【在 g***3 的大作中提到】
: 女儿的美国老师,现在一说到缺谱,就说如果他当选美国总统,以后就再也不回美国了,
: 呵呵

k**n
发帖数: 6198
215
其实我最近发现一个太肤浅但是实施起来特深刻的道理, 推正态分布娃就是比父母的见
识,脾气和交流方式. outlier在黑人窟或者大黑帮也会人中龙凤. 比如那个PAC妈妈,肯
定是个SUPER PAC.
退一万步,自己情商不行的话, 也要有操练的意识,推娃才能有效果.
我今早好容易逮着领导一辆车送娃上学,老大DROP了以后,我叨叨叨好久,就是抱怨对老
大的不满和未来的悲伤...............三岁老二都在那里说, 别说了!DROP老二以后,
对领导说,你不听我抱怨, 我真要伤了. 我容易吗我, 我这么急性子,不耐烦的一个人,
天天和老大玩装温柔, 磨耐心, 没地方爆发一下, 你当我是氮气钢瓶呢

【在 g***3 的大作中提到】
: 不会的啦,,和女儿从小交心,,她看准的人,准会被妈妈喜欢的啦,,
: 女儿这方面,比较听话,她听我们,哥哥的话,以前处的那个男朋友,就是我们跟她慢
: 慢分析后,几次她再观察,然后自己说,断了,,
: 现在班上据说,有一个很害羞的男孩,看上他,那个男孩妈妈是英国人,,她是负责小
: 学入学竟考的负责人,上次给我电话,然后说起他儿子和我女儿同班,我问女儿,才知
: 道的,,
: 呵呵,,女儿说他,太腼腆了,,,

f**********g
发帖数: 4709
216
不管怎么说,欧洲还是没落了啊。美国虽然是个暴发户,也不能否认之前以及现在的强
盛。
说到将来,我倒希望咱们祖国能够兴起,不管是从欧洲还是美国,能弄回去一些牛人也
行啊。可惜国内现在也是乌烟瘴气

【在 x***1 的大作中提到】
: 来些实打实的。
: 最近几十年的contribution吧,二战以前,若贝尔奖有美国什么事?
: 就最近几十年,每年若贝尔奖不也有欧洲的吗?
: 二战以后,美国科技突飞猛进,不是一堆欧洲的科学家在起很大的作用吗?
: 等这些欧洲科学家的徒子徒孙死光了,真让人怀疑美国本土自己能不能培养出科学家。
: 倒是商业土豪一堆,插管吸血得心应手。

i**e
发帖数: 19242
217
要不说拼爹呢?:)
PACMOM是国内top2出来的,孩子的智商能差么?
晚上孩子们都睡了,好好滴扯着你家属抱怨:)
换个时间做这件事就好了

,

【在 k**n 的大作中提到】
: 其实我最近发现一个太肤浅但是实施起来特深刻的道理, 推正态分布娃就是比父母的见
: 识,脾气和交流方式. outlier在黑人窟或者大黑帮也会人中龙凤. 比如那个PAC妈妈,肯
: 定是个SUPER PAC.
: 退一万步,自己情商不行的话, 也要有操练的意识,推娃才能有效果.
: 我今早好容易逮着领导一辆车送娃上学,老大DROP了以后,我叨叨叨好久,就是抱怨对老
: 大的不满和未来的悲伤...............三岁老二都在那里说, 别说了!DROP老二以后,
: 对领导说,你不听我抱怨, 我真要伤了. 我容易吗我, 我这么急性子,不耐烦的一个人,
: 天天和老大玩装温柔, 磨耐心, 没地方爆发一下, 你当我是氮气钢瓶呢

x***1
发帖数: 999
218
就美国的deficit问题,和美国人讨论了一下,结论是:
这个问题是个谁也没有办法解决的问题,主持人问谁,都等于白问,一点也不影响选举
。就相当于高考出了一道没有一个学生回答得上来的问题。当年奥巴马信誓旦旦要解决
这个问题,8年下来,double.
每个公民头上4万多的债务,47%不纳税,白人剧降,印人受宠,黑墨暴增,华人郁郁不
得志,这国家还能来吗?

【在 g***3 的大作中提到】
: 被你这么一解释,就明了多了,老大上次也说,缺谱说对堕胎的女人要判刑,这也太极
: 端了,
:
: trump

i**e
发帖数: 19242
219
莫捉急
孩子养着养着,你就会有自己的答案了:)

【在 c***x 的大作中提到】
:
: interesting and insightful :-)
: 勤于思考的北京出租车司机侃大山,时不时也会冲进中南海的吧?

i**e
发帖数: 19242
220
哎呀呀
有第3个solution吗?
//卢瑟迎风凌乱中问....

【在 f**********g 的大作中提到】
: 提供一个solution:多生几个娃
: 据我的观察啊,如果只有一两个娃,父母好像就会特别担心万一娃不成材怎么办,被耽
: 误了怎么办,尽最大所能的让娃成长到70-90分。可是这又有问题,因为你抹去了<60分
: 那个区间,>90分那个区间也一并被抹去了。娃的“正好不是当下社会中回报率最高的
: 那些行业职业最需要的raw talent”就被扼杀了。
: 但是有三个以上娃的,一般到了老三老四,父母就会很开明,随便他要做什么,能健康
: 成长,养活自己就行了。其实在美国这个地方,但凡是高中毕业的娃,都能找到个五六
: 万的工作养活自己。更不用说中国娃怎么都能上个地区(非社区)college。这样子反
: 而给娃更大的自由度。
: 这个区别,是为人父母的天性。除非有大智慧或者经历不寻常的父母才能免俗。

相关主题
[转载] 为什么说usamo 简单分数应用题
二年级女儿的report我招,我是猪
觉得美帝数学进度慢的进来看一下请教:怎样说服孩儿他爸多关心孩子的学习?
进入Parenting版参与讨论
g***3
发帖数: 471
221
这个生多个孩子的办法,我体会还是,要量力而行,
我觉得做父母的,要估量自己的情绪能力,
我是家里的老3,,上有姐姐,还有哥哥,他们都比我大很多,我的到来,实在是父亲
在那个年代,志不得意,然后想通过再要一个男孩,来弥补自己,
我母亲是不想再要孩子的,
我就是家里的多头,,从小就被当成街上捡来的,,
我自己,很长时间,没要孩子,起先,是为了事业,,后来,遇到一个很大的车祸,在
医院病房里有了很多思考的时间,,老大的到来,是非常喜悦的,,我还记得那天从化
验室拿到怀孕单的那个阳光灿烂的午后,他改变了我的人生,,那个时候,我还以为,
我有一个孩子,是够了,怀他,生孕,都没有妨碍我的工作,

【在 f**********g 的大作中提到】
: 提供一个solution:多生几个娃
: 据我的观察啊,如果只有一两个娃,父母好像就会特别担心万一娃不成材怎么办,被耽
: 误了怎么办,尽最大所能的让娃成长到70-90分。可是这又有问题,因为你抹去了<60分
: 那个区间,>90分那个区间也一并被抹去了。娃的“正好不是当下社会中回报率最高的
: 那些行业职业最需要的raw talent”就被扼杀了。
: 但是有三个以上娃的,一般到了老三老四,父母就会很开明,随便他要做什么,能健康
: 成长,养活自己就行了。其实在美国这个地方,但凡是高中毕业的娃,都能找到个五六
: 万的工作养活自己。更不用说中国娃怎么都能上个地区(非社区)college。这样子反
: 而给娃更大的自由度。
: 这个区别,是为人父母的天性。除非有大智慧或者经历不寻常的父母才能免俗。

x***1
发帖数: 999
222
说的好啊,国内乌烟瘴气,但在走上坡路。感觉欧洲美国在走下坡路。

【在 f**********g 的大作中提到】
: 不管怎么说,欧洲还是没落了啊。美国虽然是个暴发户,也不能否认之前以及现在的强
: 盛。
: 说到将来,我倒希望咱们祖国能够兴起,不管是从欧洲还是美国,能弄回去一些牛人也
: 行啊。可惜国内现在也是乌烟瘴气

k**n
发帖数: 6198
223
是啊,领导说你就会这么负能量!俺这不欺负老二还小,希望伊像没听见处理嘛.以后再也
不敢了,小兔崽子们其实都是我祖宗阿.很希望能像楼主那样以后能一起选男朋友,为了
未来的美好时刻,必须从今天每个时刻做起.......
关于讨论养蛙战略方针, 我家领导要朝见的. 平时都是技术细节讨论或者具体安排. 感
觉过上一段时间,是需要开个progress report的会议, 以及远景构想,每个季度要重审
strategic plan, 免得失了推的方向:))
完了,写检讨书啥的,提醒自己,改造自己,哈哈

【在 i**e 的大作中提到】
: 要不说拼爹呢?:)
: PACMOM是国内top2出来的,孩子的智商能差么?
: 晚上孩子们都睡了,好好滴扯着你家属抱怨:)
: 换个时间做这件事就好了
:
: ,

k**n
发帖数: 6198
224
民主党上台加福利, 共和党上台加军费,谁上来都不能动对方的粥. 克林顿多好的总统,
现在在民主党内都容纳不下了, 好容易发个言,说了句真话,回家就跪搓衣板了.
看楼主对法国的描述,民主是没戏了.邓小平说要防左也要防右.但是极左更可怕. 感觉
世界的希望在社会主义中国了呢. 最近看animal farm,写的太好了,推荐你看一看

【在 x***1 的大作中提到】
: 就美国的deficit问题,和美国人讨论了一下,结论是:
: 这个问题是个谁也没有办法解决的问题,主持人问谁,都等于白问,一点也不影响选举
: 。就相当于高考出了一道没有一个学生回答得上来的问题。当年奥巴马信誓旦旦要解决
: 这个问题,8年下来,double.
: 每个公民头上4万多的债务,47%不纳税,白人剧降,印人受宠,黑墨暴增,华人郁郁不
: 得志,这国家还能来吗?

f**********g
发帖数: 4709
225
你家老大很厉害啊

【在 g***3 的大作中提到】
: 这个使我想起老大有一次哲学课后,回来我们讨论的一个主题,自由和约束
: 他说,老师给他们看一个老师很喜欢的爵士乐钢琴家的演奏,,他的那种发挥纯青自如
: ,自然奔放,传染给人那种自由,洒脱,欢快,忘我所以的感觉,,
: 能够达到这种地步,需要钢琴家以前长期的苦行僧似的锻炼,磨难,刻苦练习,,,
: 所以儿子说,都是相对的,他告诫弟弟,如果以后想获得自由,那么现在就要苦练,
: 我觉得做为父母,很多书本上的大套套,在孩子心智还没达到一定程度时,还是少说为
: 好,
: 最重要的,是和孩子心连心,让他们体会
: 在 cjdlx (五柳) 的大作中提到: 】
: system

h*****m
发帖数: 1034
226
说过不要追杀我的。那我也只好杀一回马枪了。
上错了F-22培训班的F-16驾驶员,大摇大摆闯入对方阵地,看见地面的一群群的雷达站
,撇嘴一笑,心里说“你看不到我,你看不到我“。多长时间会被揍下来?

【在 t******l 的大作中提到】
: 当然另一方面,如果开着一架 F-22 隐形战机,但是后座的 F-16 中队转业过来武器控
: 制员,看见地面的一群群的雷达站就吓得开雷达干扰器、扔雷达干扰箔条。。。那前座
: 飞行员多半破口大骂后座武器控制员说,你丫干脆直接举块牌子曰:“F-22 在此,欢
: 迎来揍!” 。。。娘的你丫要是再敢扔雷达干扰箔条,信不信我一拉触发后座弹射椅
: 的红绳儿,你丫就跟雷达干扰箔条们一块儿下飞机算了!!

h**w
发帖数: 4510
227
版上讲男女,讲来讲去其实也就不出自己的经验和三观。
其实可以想想屠呦呦的例子。要说屠呦呦特别的聪明过人吧,估计这里不少人要从鼻子
里哼一下的。 我觉得中国在很长时间内不会再出一个屠呦呦,一个很大的原因就是,
中国的女性意识在抗战起因缘际会的空前高涨,在一段时间内不会再现了。 女性在那
个时代的思想解放,屠呦呦就是开出的最美的花。 这不是说她比别人更强什么的。在
那个“女性能抵半边天的”的时代背景下,女性可以得到类似的资源,没有过多的束缚
。比如说,屠呦呦可以把三个月的孩子扔下,不是要赞扬这种行为,但这是典型的“男
性优待”,女性在那个年代可以同样毫无心理负担的享受到。
其实在中国很多对中国女性的profile,和西方人对中国人的profile是类似的。比如
学校里成绩都不错,一出学校全完蛋这种。大家同不同意呢?

【在 f**********g 的大作中提到】
: 智商只是个笼统的概念啊,同样智商同样聪明的男孩女孩,可能一个更擅长抽象思维,
: 一个更擅长形象思维呢?
: 竞赛确实是更单纯focus在某个狭窄领域的一根筋行为,可能更适合男孩子一些。而大
: 学课程和工作,就要涉及更多非知识方面的因素,聪明的女孩子更容易安排协调好

h**w
发帖数: 4510
228
民主离民粹只有一步之遥。 大家要谅解。

【在 x***1 的大作中提到】
: 我看了那个采访,当时两个人都有些急,youtube上应该有,主持人穷追不舍,Trump迫
: 不得已说illegal的要punishment,我当时觉得没有错啊,后来三番五次的改变,就演
: 化成现在的版本:Trump要惩罚堕胎的人。
:
: trump

f**********g
发帖数: 4709
229
你们怎么对卡片的反应都这么负面?
我看到了只觉得新奇和欣赏,这么一个做事情细心又认真的女孩子,以后不管做文字还
是管理工作都很不错。而且高中枯燥的学习过程都能添加一些花样,那么以后自己的生
活也会过的很有情趣
至于因此浪费了时间,稍微引导一下,让她学会统筹安排就好了,实在没必要打击扼杀

【在 C*****d 的大作中提到】
: 高一的化学,哪里需要那么多卡片?
: 要搞的话一张也都写完了。
: 搞那么多,就是完全没学懂。
: 这个完全没学懂,
: 也不是什么大不了的事,
: 顶多以后拿化学诺贝尔是没戏了。
: 如果成绩对她很重要,
: 就请个家教,
: 找最好的大学里学这个的女孩子,
: 稍稍补补,考试是没问题的。

f**********g
发帖数: 4709
230
哈哈,你这个爱情算法也不靠谱啊。小伙子现在对你女儿的爱有一百分,不保证三年十
年以后还是如此。爱情和婚姻都需要经营,还有命……

【在 k**n 的大作中提到】
: 这些生物技术的成功,从技术的实现以及和本质上基于对人体的了解,和人类对阿法购算
: 法的理解,差的数量级太大了.一个是新石器,一个是蒸汽机呢
: 虽然表现就是在成功率. 不要小看概率的提高, 貌似量变,其实质变. 不敢相信人类下
: 一个生物技术革命能有第一次工业革命加速度那么快. 因为我生活在新石器,暂时不想
: 风筝的事情,更不会想飞机:)
: 呼唤爱情算法快快出来就好了, 老娘我把未来女婿进门的照片一照,立刻告诉女儿们他
: 对你的爱到底得有多深. 养女娃最怕就是被臭小子们给骗了!!

相关主题
娃又在推算他的质数公式。。。学校早晚要教的东西,早早学会了又如何?
大学录取率构建式数学在中国
别鸡兔同笼了,来喝啤酒吧关于刷还是不刷
进入Parenting版参与讨论
f**********g
发帖数: 4709
231
再次认证了我说的,
到了老三,不管父母是阅历上更成熟了还是情绪上更放松了,娃都能成长的更宽松自由
一些

【在 g***3 的大作中提到】
: 谢谢,,
: 我真心觉得,我们小时候,特别是我这种60年末,70年代初的国人,我们的父母在文革
: 中,受到的压力,都在我们身上有形无形的衍生,,
: 做为妈妈,最最重要的是,情绪上能够承受,承受你的孩子,不管他们丑,矮,笨,,
: 等等,,
: 孩子,是要看着妈妈的笑脸长大的,,,
: 这个我在2个大孩子小时候,还无法完全做到,,
: 在小儿子身上,我完全看到了效果,,我和老公认识快30年,,结婚快20年,,
: 我们一直在双方努力,就是情绪,,理解我们自己的童年,我们和自己父母的关系,

f**********g
发帖数: 4709
232
恁就是有大智慧心胸特别宽的人啊
再顶着锅盖加一句,
但是架不住只有一个娃,还是很难放开了让娃自由生长,这不还是推娃去了最好的高中
/坐着火箭都跑的不够快了

【在 i**e 的大作中提到】
: 哎呀呀
: 有第3个solution吗?
: //卢瑟迎风凌乱中问....

f**********g
发帖数: 4709
233
嗯,这个老三,必须是当妈的非常想要,当爸的也得大力支持才行

【在 g***3 的大作中提到】
: 这个生多个孩子的办法,我体会还是,要量力而行,
: 我觉得做父母的,要估量自己的情绪能力,
: 我是家里的老3,,上有姐姐,还有哥哥,他们都比我大很多,我的到来,实在是父亲
: 在那个年代,志不得意,然后想通过再要一个男孩,来弥补自己,
: 我母亲是不想再要孩子的,
: 我就是家里的多头,,从小就被当成街上捡来的,,
: 我自己,很长时间,没要孩子,起先,是为了事业,,后来,遇到一个很大的车祸,在
: 医院病房里有了很多思考的时间,,老大的到来,是非常喜悦的,,我还记得那天从化
: 验室拿到怀孕单的那个阳光灿烂的午后,他改变了我的人生,,那个时候,我还以为,
: 我有一个孩子,是够了,怀他,生孕,都没有妨碍我的工作,

C*****d
发帖数: 2253
234
比如明天你要去参选版主,
考题是九九乘法表,
你又是一个生活充满情趣的人,
你会今天晚上花几个小时给自己做九九乘法表的小卡片么?
或者说,有另一个候选人没有花几个小时给自己做九九乘法表的小卡片,咱也不能推出
他就生活没情趣。
这个妈妈来问的又不是孩子生活没情趣怎么办,而是化学要怎么学,和她有没有情趣有
什么关系。

【在 f**********g 的大作中提到】
: 你们怎么对卡片的反应都这么负面?
: 我看到了只觉得新奇和欣赏,这么一个做事情细心又认真的女孩子,以后不管做文字还
: 是管理工作都很不错。而且高中枯燥的学习过程都能添加一些花样,那么以后自己的生
: 活也会过的很有情趣
: 至于因此浪费了时间,稍微引导一下,让她学会统筹安排就好了,实在没必要打击扼杀

f**********g
发帖数: 4709
235
你这话让我想起来我昨天的胡思乱想,人类很久以后的将来,会不会全都是试管婴儿了
呢,女性不需要承担怀孕和哺乳期,娃小的时候也有专门的人带,真正从家庭中得到解
放,那么在工作和科研方面的成就一定比现在高。
可是吧,想起来又觉得冷冰冰的

【在 h**w 的大作中提到】
: 版上讲男女,讲来讲去其实也就不出自己的经验和三观。
: 其实可以想想屠呦呦的例子。要说屠呦呦特别的聪明过人吧,估计这里不少人要从鼻子
: 里哼一下的。 我觉得中国在很长时间内不会再出一个屠呦呦,一个很大的原因就是,
: 中国的女性意识在抗战起因缘际会的空前高涨,在一段时间内不会再现了。 女性在那
: 个时代的思想解放,屠呦呦就是开出的最美的花。 这不是说她比别人更强什么的。在
: 那个“女性能抵半边天的”的时代背景下,女性可以得到类似的资源,没有过多的束缚
: 。比如说,屠呦呦可以把三个月的孩子扔下,不是要赞扬这种行为,但这是典型的“男
: 性优待”,女性在那个年代可以同样毫无心理负担的享受到。
: 其实在中国很多对中国女性的profile,和西方人对中国人的profile是类似的。比如
: 学校里成绩都不错,一出学校全完蛋这种。大家同不同意呢?

f**********g
发帖数: 4709
236
这不是我觉得,化学学不好也没那么重要嘛,如果以后学文科或者商科的话
又,楼主女儿的化学其实学的挺好的。咱老祖宗还说”好记性不如烂笔头“呢,她这个
制作卡片的过程也是加深记忆的过程。只是装饰卡片的过程时间太长,耽误了别的科目
学习而已。
再又,如果真是要竞选个**主,这个花几小时制作出来的精美卡片说不准真能打败另一
个候选人(人老印不就是靠制作精美的ppt甩开老中大步迈入管理岗位的嘛)。
你真以为考九九乘法表就是计时回答8*8等于几呢?我精心制作出来的九九乘法表卡片
,一来帮助我自己记忆背诵,二来可以分享给版上所有的父母,懒爸懒妈直接copy打印
出来贴在墙上,勤快爹巧手妈们照着制作出更精美的卡片给自己家娃使用,三来这个精
美的卡片还可以作为艺术品做个版标,说不准还能投个稿啥的,我这利人利己的行为不
比你花几个钟头死背九九乘法表更说明能当好一个版主?
可惜我没有这个技能,我羡慕着呢

【在 C*****d 的大作中提到】
: 比如明天你要去参选版主,
: 考题是九九乘法表,
: 你又是一个生活充满情趣的人,
: 你会今天晚上花几个小时给自己做九九乘法表的小卡片么?
: 或者说,有另一个候选人没有花几个小时给自己做九九乘法表的小卡片,咱也不能推出
: 他就生活没情趣。
: 这个妈妈来问的又不是孩子生活没情趣怎么办,而是化学要怎么学,和她有没有情趣有
: 什么关系。

r*f
发帖数: 39119
237
什么学校会没有guidance counsellor?
h**w
发帖数: 4510
238
现在不是讲究男人也要放产假吗?这个是鼓励男人也回归他们的家庭责任,而不是让大
家都逃离。
John Lennon写Woman is the negro of the world。不过也不是说奴隶解放了就事儿都
没人做了。 大家一起平等地做呗。

【在 f**********g 的大作中提到】
: 你这话让我想起来我昨天的胡思乱想,人类很久以后的将来,会不会全都是试管婴儿了
: 呢,女性不需要承担怀孕和哺乳期,娃小的时候也有专门的人带,真正从家庭中得到解
: 放,那么在工作和科研方面的成就一定比现在高。
: 可是吧,想起来又觉得冷冰冰的

f**********g
发帖数: 4709
239
以后可以培养机器人奴隶嘛
不过,万一机器人暴乱上位成功,反把人类做了奴隶就惨了。而且人家机器人可能还不
屑用我们做奴隶,就只能肉体消灭了,更惨

【在 h**w 的大作中提到】
: 现在不是讲究男人也要放产假吗?这个是鼓励男人也回归他们的家庭责任,而不是让大
: 家都逃离。
: John Lennon写Woman is the negro of the world。不过也不是说奴隶解放了就事儿都
: 没人做了。 大家一起平等地做呗。

g***3
发帖数: 471
240
法国著名的青少年心理学家Marcel Rufo,说的一句话,我一直记得,
他说,那些没有摩擦,矛盾的家庭,都是不真实的,表面的,
恰恰相反,那些时常父母间有小矛盾的,目睹日常父母如何处理解决矛盾的家庭里出来
的孩子,长大了,更会处理私人生活,和工作上的矛盾。
我和家里那位日常性格习惯上的矛盾,被孩子们看的一清二楚,虽然现在矛盾越来越少
,但偶尔的爆发,都是孩子们在分析,调停,,
当然,孩子们小的时候,我处理事最好的办法,是尽量不把自己往容易着急的情局里推,
比如,你这个早上的事,为什么,就要急匆匆,早上挤在一起呢,,搞的所有人,都挺
不耐烦的,开了一个不好天的开始,难道没有其他办法吗,每天早上,是一天的开始,
精神状态太重要了,
老2出生后,我找人来家帮我看孩子,对人的第一要求,就是性子越慢越好,耐心,讲
话慢声细语,,喜欢笑,安详的笑,,我很幸运,我找到了这样的人,,她帮助我带大
2个孩子,一直到上幼儿园,,她跟我一起旅行,这样的人,对早期的宝宝,是最最安
心的。

,

【在 k**n 的大作中提到】
: 其实我最近发现一个太肤浅但是实施起来特深刻的道理, 推正态分布娃就是比父母的见
: 识,脾气和交流方式. outlier在黑人窟或者大黑帮也会人中龙凤. 比如那个PAC妈妈,肯
: 定是个SUPER PAC.
: 退一万步,自己情商不行的话, 也要有操练的意识,推娃才能有效果.
: 我今早好容易逮着领导一辆车送娃上学,老大DROP了以后,我叨叨叨好久,就是抱怨对老
: 大的不满和未来的悲伤...............三岁老二都在那里说, 别说了!DROP老二以后,
: 对领导说,你不听我抱怨, 我真要伤了. 我容易吗我, 我这么急性子,不耐烦的一个人,
: 天天和老大玩装温柔, 磨耐心, 没地方爆发一下, 你当我是氮气钢瓶呢

相关主题
关于刷还是不刷求科普: 奥数 vs. Math Olympiad vs. Math Count vs. Math circle vs. 超前学数学
是不是任我儿子继续迷象棋有明天考AMC 8的吗?
9岁, 如何报名考amc10算不算有数学天分
进入Parenting版参与讨论
g***3
发帖数: 471
241
我也希望我们中国的强盛,他们能够到世界上越多的地方占领,这个可以造福于我们的
后代。

【在 f**********g 的大作中提到】
: 不管怎么说,欧洲还是没落了啊。美国虽然是个暴发户,也不能否认之前以及现在的强
: 盛。
: 说到将来,我倒希望咱们祖国能够兴起,不管是从欧洲还是美国,能弄回去一些牛人也
: 行啊。可惜国内现在也是乌烟瘴气

g***3
发帖数: 471
242
这本书,很形象啊,看看作者当时写的年代,,
这本书,女儿初中时,就被推荐了,
然后她推荐我看了,,

统,

【在 k**n 的大作中提到】
: 民主党上台加福利, 共和党上台加军费,谁上来都不能动对方的粥. 克林顿多好的总统,
: 现在在民主党内都容纳不下了, 好容易发个言,说了句真话,回家就跪搓衣板了.
: 看楼主对法国的描述,民主是没戏了.邓小平说要防左也要防右.但是极左更可怕. 感觉
: 世界的希望在社会主义中国了呢. 最近看animal farm,写的太好了,推荐你看一看

g***3
发帖数: 471
243
我也一直认为,每个女人会由各自经历,原生家庭的影响的不同,对自己的成人生活构
图产生多样形,
并不是每个女人都想当妈妈的,我一直尊重没有孩子的女人,
我刚刚有关多生孩子那段,由于老大回家,我去张罗,就没继续写,,
我觉得,女人在生孩子前,理想的状态是,如果能经历过一些年的就职,生存,发展,
自己养活,养好自己,累积一些必须的社会生存技巧,对自己充分理解了,再去组织家
庭,生孕孩子,会对以后的家庭生活奠定一个很好的基点。
孩子也不是一个接一个生,先生一个,看看,自己搞不搞得定,孩子好吗,自己好吗,
家庭那位好吗,,如何大家都满意,那么再继续,,生活前,很少人,能够预测以后的
变化,,
屠呦呦那代,是非常特殊的教育背景下出现的,
我在参观东柏林以前德意志人民共和国博物馆时,看到了那时的劳动妇女,养育的孩子
,工业化的托儿所的出现,令人记忆最深的,就是那十几个几个月大的宝宝,一连串坐
在便池上,等着保育院来擦屁股的照片,,我想我曾经也是其中的一个宝宝,我的母亲
,也是为了工作,我3个月,就被送到工厂托儿所去了,,
安娜弗洛伊德,当时,在维也纳开的第一个托儿所,原意是为了帮助那些穷苦的妇女,
,,让他们的宝宝受到好的教育,,而并不是生产大工业化中一个个更多的小螺丝,,
,,

【在 h**w 的大作中提到】
: 版上讲男女,讲来讲去其实也就不出自己的经验和三观。
: 其实可以想想屠呦呦的例子。要说屠呦呦特别的聪明过人吧,估计这里不少人要从鼻子
: 里哼一下的。 我觉得中国在很长时间内不会再出一个屠呦呦,一个很大的原因就是,
: 中国的女性意识在抗战起因缘际会的空前高涨,在一段时间内不会再现了。 女性在那
: 个时代的思想解放,屠呦呦就是开出的最美的花。 这不是说她比别人更强什么的。在
: 那个“女性能抵半边天的”的时代背景下,女性可以得到类似的资源,没有过多的束缚
: 。比如说,屠呦呦可以把三个月的孩子扔下,不是要赞扬这种行为,但这是典型的“男
: 性优待”,女性在那个年代可以同样毫无心理负担的享受到。
: 其实在中国很多对中国女性的profile,和西方人对中国人的profile是类似的。比如
: 学校里成绩都不错,一出学校全完蛋这种。大家同不同意呢?

g***3
发帖数: 471
244
老3,是在家庭4个成员,眼睁睁都盼着他到来,而迟迟不来,最后算了,然后他又来了,
可想而知,他不但被父母捧,而且,更被哥哥姐姐捧,,

【在 f**********g 的大作中提到】
: 嗯,这个老三,必须是当妈的非常想要,当爸的也得大力支持才行
g***3
发帖数: 471
245
介绍你去看
Brave New World
是英国作家 Aldous Huxlen 1932 年写的一本书,,书上,已经描绘了你想象的场景
,我们现在生活的社会,正一点点慢慢的往这个方向靠拢,

【在 f**********g 的大作中提到】
: 你这话让我想起来我昨天的胡思乱想,人类很久以后的将来,会不会全都是试管婴儿了
: 呢,女性不需要承担怀孕和哺乳期,娃小的时候也有专门的人带,真正从家庭中得到解
: 放,那么在工作和科研方面的成就一定比现在高。
: 可是吧,想起来又觉得冷冰冰的

h**w
发帖数: 4510
246
你这个东德的例子,如同社会主义中国, 其实蛮切你的问题的。就是都是极端追求效
率的产物。 你的女儿会是这个样子 ,一定是因为她妈妈就是一个知情识趣的人。 这
个“效率”问题, 如果没有太大的问题,也不用太担心。你女儿也不是说玩物丧志,
这就是个失之东隅,得之桑榆的事。

【在 g***3 的大作中提到】
: 我也一直认为,每个女人会由各自经历,原生家庭的影响的不同,对自己的成人生活构
: 图产生多样形,
: 并不是每个女人都想当妈妈的,我一直尊重没有孩子的女人,
: 我刚刚有关多生孩子那段,由于老大回家,我去张罗,就没继续写,,
: 我觉得,女人在生孩子前,理想的状态是,如果能经历过一些年的就职,生存,发展,
: 自己养活,养好自己,累积一些必须的社会生存技巧,对自己充分理解了,再去组织家
: 庭,生孕孩子,会对以后的家庭生活奠定一个很好的基点。
: 孩子也不是一个接一个生,先生一个,看看,自己搞不搞得定,孩子好吗,自己好吗,
: 家庭那位好吗,,如何大家都满意,那么再继续,,生活前,很少人,能够预测以后的
: 变化,,

t******l
发帖数: 10908
247
所以要实战演习。其实尽早演习抢滩 AMC 10 Bastogne 的目的也就是这个,及早知道
自己是 F-22 还是 F-16,这样好及早采取不同的人生战略。
另外地面雷达站对 隐形机 F-22 和 非隐形机 F-16 的反应还是不一样的,当然到那时
可能就有点晚了。

:说过不要追杀我的。那我也只好杀一回马枪了。
h*****m
发帖数: 1034
248
嗯,有道理,及早下水,摸摸深浅。
现在演习AMC10实在太早,刚刚研究了一下AOPS,有为低年级准备的Beast academy,下
手了一套4年级的。timefall对这套书有何评论?

【在 t******l 的大作中提到】
: 所以要实战演习。其实尽早演习抢滩 AMC 10 Bastogne 的目的也就是这个,及早知道
: 自己是 F-22 还是 F-16,这样好及早采取不同的人生战略。
: 另外地面雷达站对 隐形机 F-22 和 非隐形机 F-16 的反应还是不一样的,当然到那时
: 可能就有点晚了。
:
: :说过不要追杀我的。那我也只好杀一回马枪了。
: :

c***x
发帖数: 1826
249

谢谢这位妈妈分享经验和心得,很感慨很感动。
你一定好善解人意,我昨天是有点想问怎么平衡你自己的生活和照顾三个孩子的。果然
被你猜了心。
想起我自己的总结:生活就是最好的教科书;想起izze的总结:言传身教是最好的
parenting。

【在 g***3 的大作中提到】
: 孩子越小,家长越能陪伴,塑造,越能给予孩子影响,让他们去,摸索,
: 我从怀第一孩子起,体会到自己,是逐渐的,翻天覆地的变化,提高,感受生活,感激
: 爱我的人,带给我的一切。
: 他们在这些年里,伴随着我,包括我的职业变迁过程,目睹我 日常情绪,精神面貌的
: 改善,,
: 我至从有了孩子后,尽量什么事,都带着他们在我的身边,让他们和我一起分享,快乐
: 或者有时的犹豫,厌烦, 我以前和国内做纺织工业贸易,经常出差,法国这里学校假
: 期又多,2个大孩子从我怀孕,到出生,到几个月,就被我带来带去,我检货的时候,
: 他们就在欧洲的供货商厂内办公室玩,看书,画画,在国内时,有更多的阿姨叔叔和他
: 们玩耍,,到吃饭的时候,就和我们一起去饭馆,他们了解国内的乡村和城市,,尤其

c***x
发帖数: 1826
250

这个,稍微解释一下,我自己做老师的,昨天看潮水抱怨很多老师给Teacher's pets高
分,觉得是很好的警示,所以问问他好的grading system应该是什么样的。
但是你的回帖很有意义,算是意外的收获。你家老大的心智很成熟,可能是好的教育,
也可能是你在生活中给他的启示。完全同意你说的,“最重要的,是和孩子心连心,让
他们体会”。我家娃才三岁,我就对这一点深有体会。
话说,你家娃多,年龄又分散,以后你多来聊天,有问题大家和你一起分析,你同时也
给我们分享你的经验和体会。这个论坛,本来就应该是这样的。

【在 c***x 的大作中提到】
:
: 谢谢这位妈妈分享经验和心得,很感慨很感动。
: 你一定好善解人意,我昨天是有点想问怎么平衡你自己的生活和照顾三个孩子的。果然
: 被你猜了心。
: 想起我自己的总结:生活就是最好的教科书;想起izze的总结:言传身教是最好的
: parenting。

相关主题
大家怎么推孩子的弱项?二年级女儿的report
少年智力开发觉得美帝数学进度慢的进来看一下
[转载] 为什么说usamo 简单分数应用题
进入Parenting版参与讨论
c***x
发帖数: 1826
251

可以赶上写下一部科幻电影的剧本了 :-)
有没有看过最近的两部 and ?

【在 k**n 的大作中提到】
: 好奇一问,人脑神经元和机器神经元,到底哪个快?感觉目前谁也不能下定义
: 感觉最基本的就是人的大脑不能存储那么多数据(也许生物技术将来能开发90%的大脑灰
: 质,而这里面的大脑灰质有指数级的储藏功能).做个基因库或者干细胞或者未来RNA技术
: 啥啥啥的,把所有历史顶级棋手数据库转倒新生儿脑子里去,该数据库拒绝向机器传输(
: 所有九段高手临终前务必签协议,谨防巴拿马这类"内贼").再和阿发狗一战高下如何?
: 但是,人要生下来就变个智慧的小老头小老太的话,不需要同性恋来阻止人类繁殖而加速
: 灭亡,所有当妈的99%罢工辞职, 大概再也不想生娃娃了.生下来就比智商和情商,
: TERRIBLE TWO的时候妈妈全趴下,养不活超级大神哈
: 看来世界的未来不是机器人,因为机器人其实是机器,而是人机器,人机器还会是人.

c***x
发帖数: 1826
252

那这个爱情测算要每18个月就做一次 :-)
还是呼唤生物技术,直接一针算了,保证臭小子一辈子深爱你娃,多快好省。
问题是你娃如果有一天不爱他了,又怎么办呢?再打一针彻底遗忘针,哈哈。

【在 k**n 的大作中提到】
: 这些生物技术的成功,从技术的实现以及和本质上基于对人体的了解,和人类对阿法购算
: 法的理解,差的数量级太大了.一个是新石器,一个是蒸汽机呢
: 虽然表现就是在成功率. 不要小看概率的提高, 貌似量变,其实质变. 不敢相信人类下
: 一个生物技术革命能有第一次工业革命加速度那么快. 因为我生活在新石器,暂时不想
: 风筝的事情,更不会想飞机:)
: 呼唤爱情算法快快出来就好了, 老娘我把未来女婿进门的照片一照,立刻告诉女儿们他
: 对你的爱到底得有多深. 养女娃最怕就是被臭小子们给骗了!!

c***x
发帖数: 1826
253

确实是母爱伟大的一种体现。我有时候也会反思,这样做到底是不是对的,是不是最好。
至少你的答案和经历,看来是正面的。

【在 g***3 的大作中提到】
: 介绍你去看
: Brave New World
: 是英国作家 Aldous Huxlen 1932 年写的一本书,,书上,已经描绘了你想象的场景
: ,我们现在生活的社会,正一点点慢慢的往这个方向靠拢,

c***x
发帖数: 1826
254

有没有想起来,我曾经说过的话?

【在 g***3 的大作中提到】
: 你是我在这个坛最敬佩喜欢的一个id,,你的俏皮和智慧,值得我好好学习。
c***x
发帖数: 1826
255

哈哈,三羊妈妈看样子也是一个可爱的nerd,太好了。
我家领导主张你的第一个solution,然后写了个模型,放了点理想的参数,得出结论,
要生25个才可以在平均意义上散养。
我倾向于你的第二个solution,但是这个10M的基金好像难度也不小。
既然角点解都行不通,那么就只好求内点最优解了。
我猜这也是绝大部分用心的为人父母者看着并不太潇洒的原因吧。
这是我对“可怜天下父母心”的最新理解。

【在 f**********g 的大作中提到】
: 提供一个solution:多生几个娃
: 据我的观察啊,如果只有一两个娃,父母好像就会特别担心万一娃不成材怎么办,被耽
: 误了怎么办,尽最大所能的让娃成长到70-90分。可是这又有问题,因为你抹去了<60分
: 那个区间,>90分那个区间也一并被抹去了。娃的“正好不是当下社会中回报率最高的
: 那些行业职业最需要的raw talent”就被扼杀了。
: 但是有三个以上娃的,一般到了老三老四,父母就会很开明,随便他要做什么,能健康
: 成长,养活自己就行了。其实在美国这个地方,但凡是高中毕业的娃,都能找到个五六
: 万的工作养活自己。更不用说中国娃怎么都能上个地区(非社区)college。这样子反
: 而给娃更大的自由度。
: 这个区别,是为人父母的天性。除非有大智慧或者经历不寻常的父母才能免俗。

c***x
发帖数: 1826
256

,
明明就是Judy Hobbs遇上flash,还不承认 :-)

【在 k**n 的大作中提到】
: 其实我最近发现一个太肤浅但是实施起来特深刻的道理, 推正态分布娃就是比父母的见
: 识,脾气和交流方式. outlier在黑人窟或者大黑帮也会人中龙凤. 比如那个PAC妈妈,肯
: 定是个SUPER PAC.
: 退一万步,自己情商不行的话, 也要有操练的意识,推娃才能有效果.
: 我今早好容易逮着领导一辆车送娃上学,老大DROP了以后,我叨叨叨好久,就是抱怨对老
: 大的不满和未来的悲伤...............三岁老二都在那里说, 别说了!DROP老二以后,
: 对领导说,你不听我抱怨, 我真要伤了. 我容易吗我, 我这么急性子,不耐烦的一个人,
: 天天和老大玩装温柔, 磨耐心, 没地方爆发一下, 你当我是氮气钢瓶呢

c***x
发帖数: 1826
257

听美女姐姐的,不着急。
其实养着养着,也挺有意思的。
我有时候甚至觉得,总是想她将来怎么样怎么样,是不是太“无私”了,我就不能“自
私”一点吗,好好Enjoy当下养她的那种快乐,甜蜜,惊喜,烦恼,以后的事情,以后
再说吧。当然来版上看看,你们大孩子父母们的经验之谈,就好像在读一本《what to
expect》,也很受益。

【在 i**e 的大作中提到】
: 莫捉急
: 孩子养着养着,你就会有自己的答案了:)

c***x
发帖数: 1826
258

推,
这个很有启发,为人父母以后,我们总是努力不在孩子面前有任何争吵,有时候都快憋
出病来了。
下次适当的吵吵,估计女儿会无条件帮我 :-)
认真的说,处理矛盾,协调问题,确实是应该好好学习的一课。

【在 g***3 的大作中提到】
: 法国著名的青少年心理学家Marcel Rufo,说的一句话,我一直记得,
: 他说,那些没有摩擦,矛盾的家庭,都是不真实的,表面的,
: 恰恰相反,那些时常父母间有小矛盾的,目睹日常父母如何处理解决矛盾的家庭里出来
: 的孩子,长大了,更会处理私人生活,和工作上的矛盾。
: 我和家里那位日常性格习惯上的矛盾,被孩子们看的一清二楚,虽然现在矛盾越来越少
: ,但偶尔的爆发,都是孩子们在分析,调停,,
: 当然,孩子们小的时候,我处理事最好的办法,是尽量不把自己往容易着急的情局里推,
: 比如,你这个早上的事,为什么,就要急匆匆,早上挤在一起呢,,搞的所有人,都挺
: 不耐烦的,开了一个不好天的开始,难道没有其他办法吗,每天早上,是一天的开始,
: 精神状态太重要了,

c***x
发帖数: 1826
259

经济上是这样的,但是政治上,从2015年起有退步。
你看过《动物农场》、《美丽新世界》,估计也就看过《1984》了,大致往那个方向去
了。

【在 f**********g 的大作中提到】
: 不管怎么说,欧洲还是没落了啊。美国虽然是个暴发户,也不能否认之前以及现在的强
: 盛。
: 说到将来,我倒希望咱们祖国能够兴起,不管是从欧洲还是美国,能弄回去一些牛人也
: 行啊。可惜国内现在也是乌烟瘴气

g***3
发帖数: 471
260
我猜测,妹妹可能比我们这代小,
对东柏林那个时代,也许,没有我们这代这么有感受,
尤其,我是在柏林墙被堆到那个岁月初遇到我后来的丈夫,他们那时一起的一群来上海
合作的法国年轻工程师们,,
参观东柏林博物馆,仿佛使我回到我从小成长的那个时代,马克思大街,列宁马克思恩
格斯同志的大幅画像,,后来稍开放后,服装画报的流行服饰,
妇女半边天后产生的一系列于此而来的托儿所,儿童卫生,小学,教育,,,
我们这代人,还是成长在那个时代
有事,再聊

你这个东德的例子,如同社会主义中国, 其实蛮切你的问题的。就是都是极端追求效


【在 h**w 的大作中提到】
: 你这个东德的例子,如同社会主义中国, 其实蛮切你的问题的。就是都是极端追求效
: 率的产物。 你的女儿会是这个样子 ,一定是因为她妈妈就是一个知情识趣的人。 这
: 个“效率”问题, 如果没有太大的问题,也不用太担心。你女儿也不是说玩物丧志,
: 这就是个失之东隅,得之桑榆的事。

相关主题
我招,我是猪大学录取率
请教:怎样说服孩儿他爸多关心孩子的学习?别鸡兔同笼了,来喝啤酒吧
娃又在推算他的质数公式。。。学校早晚要教的东西,早早学会了又如何?
进入Parenting版参与讨论
t******u
发帖数: 1837
261
这个说法真是扯淡扯的没边
你看看慈禧太后的爹妈有啥见识了,并没有影响慈禧成为了军事家和政治家,还是个很
有水平的文科女青年
一个娃娃过了12岁以后,基本就靠环境了,这个环境有两个,一个是大环境,所生活的
那个城市的氛围,另一个是小环境,不是父母买的三室一厅,而是他能接触到的老师,
学区,还有同学。
烂学区的孩子成绩都差,那不是父母见识差造成的,而是环境比父母的见识更差,比如
一个Chinatown的开餐馆的父母生出来的孩子没有几个成才的只因为他们生下来耳染目
濡的内容就认为世界就该像Chinatown那样从东街肮脏恶臭到西头街。

【在 k**n 的大作中提到】
: 民主党上台加福利, 共和党上台加军费,谁上来都不能动对方的粥. 克林顿多好的总统,
: 现在在民主党内都容纳不下了, 好容易发个言,说了句真话,回家就跪搓衣板了.
: 看楼主对法国的描述,民主是没戏了.邓小平说要防左也要防右.但是极左更可怕. 感觉
: 世界的希望在社会主义中国了呢. 最近看animal farm,写的太好了,推荐你看一看

t******u
发帖数: 1837
262
慈禧太后最终统治了国家,是因为天天晚上跟皇上陪读,研墨,就等于皇上直接对她进
行了军事和政治的培训基础课了,当时女性不能参政的。
一个人后来能走出啥路线出来,直接取决于她成长过程中逐步地后来遇到了些什么人,
这些人才是真正对她命运发生重大影响的,跟父母见识有个鸟关系,胡锦涛让他儿子经
商了吗?让他女儿嫁给一个秃顶老头了吗,那个环境下遇上了,就只能这么走了,谁还
真能见识到自己过十年能当上国家主席啊
t******l
发帖数: 10908
263
我看到了帖子,忙没来得及回。会回你那个贴。


:【 在 cjdlx (五柳) 的大作中提到: 】
i**e
发帖数: 19242
264
扑哧
你就直接处理了,会有什么严重后果吗?:)
你俩闺女?

【在 k**n 的大作中提到】
: 是啊,领导说你就会这么负能量!俺这不欺负老二还小,希望伊像没听见处理嘛.以后再也
: 不敢了,小兔崽子们其实都是我祖宗阿.很希望能像楼主那样以后能一起选男朋友,为了
: 未来的美好时刻,必须从今天每个时刻做起.......
: 关于讨论养蛙战略方针, 我家领导要朝见的. 平时都是技术细节讨论或者具体安排. 感
: 觉过上一段时间,是需要开个progress report的会议, 以及远景构想,每个季度要重审
: strategic plan, 免得失了推的方向:))
: 完了,写检讨书啥的,提醒自己,改造自己,哈哈

i**e
发帖数: 19242
265
北滩呢?
伊会教导嫩,除非放娃入自然,这世上木有不推的父母,哈哈

【在 f**********g 的大作中提到】
: 恁就是有大智慧心胸特别宽的人啊
: 再顶着锅盖加一句,
: 但是架不住只有一个娃,还是很难放开了让娃自由生长,这不还是推娃去了最好的高中
: /坐着火箭都跑的不够快了

i**e
发帖数: 19242
266
纯抬杠
这理想状态出来,就圣女了啊,亲!
立足于自我的追求和独立是对滴
谈恋爱成家是可以并行地
anyway,爱情,it 说来就来了,能规划么?:)

【在 g***3 的大作中提到】
: 我也一直认为,每个女人会由各自经历,原生家庭的影响的不同,对自己的成人生活构
: 图产生多样形,
: 并不是每个女人都想当妈妈的,我一直尊重没有孩子的女人,
: 我刚刚有关多生孩子那段,由于老大回家,我去张罗,就没继续写,,
: 我觉得,女人在生孩子前,理想的状态是,如果能经历过一些年的就职,生存,发展,
: 自己养活,养好自己,累积一些必须的社会生存技巧,对自己充分理解了,再去组织家
: 庭,生孕孩子,会对以后的家庭生活奠定一个很好的基点。
: 孩子也不是一个接一个生,先生一个,看看,自己搞不搞得定,孩子好吗,自己好吗,
: 家庭那位好吗,,如何大家都满意,那么再继续,,生活前,很少人,能够预测以后的
: 变化,,

i**e
发帖数: 19242
267
老师,木有想到您会抽查,没复习也没有用功记忆过,完全不知是那句您说过的话儿:)
交白卷了...

【在 c***x 的大作中提到】
:
: 经济上是这样的,但是政治上,从2015年起有退步。
: 你看过《动物农场》、《美丽新世界》,估计也就看过《1984》了,大致往那个方向去
: 了。

i**e
发帖数: 19242
268
第3个solution:顺其自然:)
我,不相信,富豪们对孩子就那点儿期望,甘心于养着孩子
没有自己的追求人生没有自己的立足点,100个米林养着,心里也是空虚的啊,能过好
吗?

【在 c***x 的大作中提到】
:
: 经济上是这样的,但是政治上,从2015年起有退步。
: 你看过《动物农场》、《美丽新世界》,估计也就看过《1984》了,大致往那个方向去
: 了。

i**e
发帖数: 19242
269
十几年了,我一直都固守着自己的观点:父母不必在孩子面前事事一致
小吵怡情;大吵益智
重点是会吵架 //我顶锅盖速遁了...

【在 c***x 的大作中提到】
:
: 经济上是这样的,但是政治上,从2015年起有退步。
: 你看过《动物农场》、《美丽新世界》,估计也就看过《1984》了,大致往那个方向去
: 了。

i**e
发帖数: 19242
270
正经的事儿,我说完了
现在开始8挂:)
原来你丈夫是法式帅哥,孩子们是混血式美少啊! //可不回应:)

【在 g***3 的大作中提到】
: 我猜测,妹妹可能比我们这代小,
: 对东柏林那个时代,也许,没有我们这代这么有感受,
: 尤其,我是在柏林墙被堆到那个岁月初遇到我后来的丈夫,他们那时一起的一群来上海
: 合作的法国年轻工程师们,,
: 参观东柏林博物馆,仿佛使我回到我从小成长的那个时代,马克思大街,列宁马克思恩
: 格斯同志的大幅画像,,后来稍开放后,服装画报的流行服饰,
: 妇女半边天后产生的一系列于此而来的托儿所,儿童卫生,小学,教育,,,
: 我们这代人,还是成长在那个时代
: 有事,再聊
:

相关主题
学校早晚要教的东西,早早学会了又如何?是不是任我儿子继续迷象棋
构建式数学在中国9岁, 如何报名考amc10
关于刷还是不刷求科普: 奥数 vs. Math Olympiad vs. Math Count vs. Math circle vs. 超前学数学
进入Parenting版参与讨论
t******l
发帖数: 10908
271
我觉得 grading system 里适合程度的 favor teacher's pets,是符合将来的社
会现实的,我不觉得有啥不对。
或者另一个角度说,教师作为 service provider,(就好比律师),首先是对 client
负责,而不是为社会做贡献。合理程度 favor teacher's pets 其实是有利于先
进娃(包括竞赛娃)将来在社会上的生存能力/机会的,你懂的。
另一方面,就好比反智的大清被益智的大英打得满地找牙。但这并不是大清私塾教师的
过错,因为科举等等等等制度并不是大清私塾教师搞的,大清私塾教师也不能改变社会
。。。或者说,这事儿对大清私塾教师而言,是 inevitable。


:【 在 timefall (时光崩塌) 的大作中提到: 】
c***x
发帖数: 1826
272

谢谢智慧,明白了 :-)

【在 i**e 的大作中提到】
: 第3个solution:顺其自然:)
: 我,不相信,富豪们对孩子就那点儿期望,甘心于养着孩子
: 没有自己的追求人生没有自己的立足点,100个米林养着,心里也是空虚的啊,能过好
: 吗?

c***x
发帖数: 1826
273

client
其实这个答案是有点出乎我的意料的。
谢谢好意,也明白了 :-)

【在 t******l 的大作中提到】
: 我觉得 grading system 里适合程度的 favor teacher's pets,是符合将来的社
: 会现实的,我不觉得有啥不对。
: 或者另一个角度说,教师作为 service provider,(就好比律师),首先是对 client
: 负责,而不是为社会做贡献。合理程度 favor teacher's pets 其实是有利于先
: 进娃(包括竞赛娃)将来在社会上的生存能力/机会的,你懂的。
: 另一方面,就好比反智的大清被益智的大英打得满地找牙。但这并不是大清私塾教师的
: 过错,因为科举等等等等制度并不是大清私塾教师搞的,大清私塾教师也不能改变社会
: 。。。或者说,这事儿对大清私塾教师而言,是 inevitable。
:
: :

c***x
发帖数: 1826
274

谢过了,再来说你们两个几句。
翻看了一下去年4月底楼主妈妈来问问题时的几个高楼,查了一下diary,那几天我很忙
,也没有跳那几个楼。时过一年,再以抽离的身份去看过去的事情,倒是很多感慨。
你们两个的帖子都自己删了,只能在别人的回帖里看到你们说过些什么。当时GSJT说过
一句话,那个楼里你们是唯二不偏激的人。他是极其少夸人的,而他的这个论断,我深
表赞同。
一个ID删掉的帖子,就像一个人清空了历史。你们去年10月初删帖时,我正好不在,回
来一看就难过一次;现在看,再难过一次。但是再一想,灌过的水,不论理性深刻,还
是智慧幽默,都像你们的音容笑貌,留在了我们的记忆里。
但是,请你们以后不要再删帖了。
有一首歌叫“相见不如怀念”,我不同意,因为“怀念不如相见”。

【在 i**e 的大作中提到】
: 正经的事儿,我说完了
: 现在开始8挂:)
: 原来你丈夫是法式帅哥,孩子们是混血式美少啊! //可不回应:)

t******l
发帖数: 10908
275
我个人的看法,intellectual 不是 built for superiority,而是 built for
surviving unknown and changing environment。
(其实 arguably,self-awareness 也是 built for survival。。。古人孙子说,知
己知彼。。。)
所以我个人觉得对教师而言,更重要的是这一场 “试演/彩排” 是不是逐步符合将来
的实际社会,而不是简单的公平合理。


:【 在 timefall (时光崩塌) 的大作中提到: 】
c***x
发帖数: 1826
276

这一点我是完全同意的。
还记得Bloom's taxonomy吗?你我都是强调金字塔的上层而非下层。superiority基本
上更依赖下层的认知能力,surviving unknown and changing environment更依赖于上
层的认知能力。
Nevertheless,第一呢,没有下层,也很难有上层。第二呢,在一个稳定的社会结构里
,生产的效率主要通过不断的细化分工来实现,于是每个人只要把自己所在岗位的工作
做好,做到极致,集体来看就是最优的;真正需要个人去面对未知和变化的环境的机会
,并不是很多。所以,这个也算你所说的fit into the social structure的一种。
最后,我自己做学生的时候时常听到一句话,“优秀是一种习惯”。我现在的理解是,
superiority是智商里面,更加可以培养的那部分。而真正的raw intellectual,确实
是surviving unknown and changing environment。这个与生物进化论的观点,也是一
致的。
可是,对于个体来说,life is too short to forget about superiority.
是不是很twisted? 你笑我好了,我不怪你。
想到将来AlphaGo当道,倒是真的应该put more weight on "surviving unknown and
changing environment",尤其想到我家小娃。
所以,求你家二娃现在在看的书,在玩的游戏,在打酱油的班。。。
至于公平合理,当你真的面对几十双纯真而渴望的眼睛,他们会天然唤起你的这种本能。

【在 t******l 的大作中提到】
: 我个人的看法,intellectual 不是 built for superiority,而是 built for
: surviving unknown and changing environment。
: (其实 arguably,self-awareness 也是 built for survival。。。古人孙子说,知
: 己知彼。。。)
: 所以我个人觉得对教师而言,更重要的是这一场 “试演/彩排” 是不是逐步符合将来
: 的实际社会,而不是简单的公平合理。
:
: :
: :【 在 timefall (时光崩塌) 的大作中提到: 】

t******l
发帖数: 10908
277
我觉得 “反智” 这点,源于 social structure 这枚硬币的 “military” 的这一面。
或者就好比我们看蚂蚁蚁群时,说不定天顶星人正看着我们,跟我们看着蚁群差别不大
,都是在看着一个 反智 的 social structure。
从这个角度看,life is too short for one's own memory。而 superiority 只
要够用(survival)就好。


:【 在 timefall (时光崩塌) 的大作中提到: 】
t******l
发帖数: 10908
278
另外老邢的 app 太差,死机是家常便饭,慢的像蜗牛,回的一贴好像也丢了。。。当
然也可能是戒网利器。。。


:【 在 timefall (时光崩塌) 的大作中提到: 】
g***3
发帖数: 471
279
书是智慧的积累,具体的人也可以按自己的实际生活,去体会,或得出自己的经验。然
后和周围一起生活着的人分享,体验。

【在 c***x 的大作中提到】
:
: 这一点我是完全同意的。
: 还记得Bloom's taxonomy吗?你我都是强调金字塔的上层而非下层。superiority基本
: 上更依赖下层的认知能力,surviving unknown and changing environment更依赖于上
: 层的认知能力。
: Nevertheless,第一呢,没有下层,也很难有上层。第二呢,在一个稳定的社会结构里
: ,生产的效率主要通过不断的细化分工来实现,于是每个人只要把自己所在岗位的工作
: 做好,做到极致,集体来看就是最优的;真正需要个人去面对未知和变化的环境的机会
: ,并不是很多。所以,这个也算你所说的fit into the social structure的一种。
: 最后,我自己做学生的时候时常听到一句话,“优秀是一种习惯”。我现在的理解是,

g***3
发帖数: 471
280
我们现在在家里饭桌上,经常讨论未来的问题,人类社会模式的问题,法国这里高中,
孩子们都看过这些经典的书,如何面对现在,和将来,他们正在寻找自己的路。他们的
信心比我强。

【在 c***x 的大作中提到】
:
: 这一点我是完全同意的。
: 还记得Bloom's taxonomy吗?你我都是强调金字塔的上层而非下层。superiority基本
: 上更依赖下层的认知能力,surviving unknown and changing environment更依赖于上
: 层的认知能力。
: Nevertheless,第一呢,没有下层,也很难有上层。第二呢,在一个稳定的社会结构里
: ,生产的效率主要通过不断的细化分工来实现,于是每个人只要把自己所在岗位的工作
: 做好,做到极致,集体来看就是最优的;真正需要个人去面对未知和变化的环境的机会
: ,并不是很多。所以,这个也算你所说的fit into the social structure的一种。
: 最后,我自己做学生的时候时常听到一句话,“优秀是一种习惯”。我现在的理解是,

相关主题
有明天考AMC 8的吗?少年智力开发
算不算有数学天分[转载] 为什么说usamo 简单
大家怎么推孩子的弱项?二年级女儿的report
进入Parenting版参与讨论
g***3
发帖数: 471
281
妹妹,我知道我这种思想,说出来,肯定会被大多数人,以为理论的理想。
我以前还没有象现在这么坚决的如此认为,,
爱情,不代表婚姻,,但是婚姻,一定要有爱情,为基点,,,人的一生,随着生活,
年龄的不同阶段,可能会有几段不同色彩的爱情,但是,如果希望要有孩子,那么这个
有孩子的婚姻,最希望保持一次,,
我现在越来越觉得,人生,就是一个漫长的规划,,是的,,爱情,不能预估,但是自
己的生活,可以规划,,自己的生活,可以自己驾驭,自己锻炼,我有女儿也有儿子,
他们也是往这个方向,规划自己的未来,,
我小时,因为姐姐哥哥都比我大很多,他们有一些恋爱经历,又由于我们那时群居,很
多邻居的孩子,姐姐哥哥朋友的事,我想都给从小的我,潜移默化的影响。
我自己的母亲,也是一个工作的女性,从小就给们灌输经济独立,人格自由的意识。
女性,在没有婚姻之前,完全可以保持自己人格的自由,给自己最佳的环境,挑选那个
可以和自己走过人生大部分岁月的最佳人选。
说到具体,我想到潮水,很喜欢说慨率的事,
我想说,人生的路途,能够遇到怎么样的人,也是一个概率的事,
如何能够遇到适合自己,或者自己想遇到的人,也是一个概率的问题,
我觉得,人生最关键的不是学习好,而是,能够遇人,,
为了能够遇到自己渴望遇见的那些人群,必须把自己锻炼成那样的人。
我老大,现在完全明白了这个道理,,他说妈妈,以前根本不喜欢数学,他梦想的是最
好的商校,因为他崇拜的人,绝大多数是那些学校出来的,,他常被他们的真知烁语,
人生轨迹而敬仰,如何能进那些学校,那么概率最大的路,就是能考进去,要考进去,
就要苦练,,然后,他现在在数学上投入很多,又开始喜欢上数学,,,

【在 i**e 的大作中提到】
: 纯抬杠
: 这理想状态出来,就圣女了啊,亲!
: 立足于自我的追求和独立是对滴
: 谈恋爱成家是可以并行地
: anyway,爱情,it 说来就来了,能规划么?:)

g***3
发帖数: 471
282
这个我有很多周围人的具体例子,以后有时间来说。

【在 i**e 的大作中提到】
: 十几年了,我一直都固守着自己的观点:父母不必在孩子面前事事一致
: 小吵怡情;大吵益智
: 重点是会吵架 //我顶锅盖速遁了...

g***3
发帖数: 471
283
我在这里潜水,已经有些年了,这里大多数id,给我的启发,是很有益的,
集思广益就是这样的。
说到偏激,我常常想到我的老大,他这个年龄,是最偏激的阶段了,我们经常讨论问题
,昨天,我们出去逛街,我跟他们说到我在这个坛的交流,说到缺谱,说到美国大学配
额等,他开始不同意我的看法,那么,我就听他的,拿出他的论证,,,我们一起化时
间,去查看,,,,
为什么,会偏激,,
人的观点,往往从自我开始,,
接触得多了,体验得多了,视角不同,,就会慢慢理解,这个过程中,最重要的是情绪
,不要让自己的情绪,被别人牵着走,,,

【在 c***x 的大作中提到】
:
: 这一点我是完全同意的。
: 还记得Bloom's taxonomy吗?你我都是强调金字塔的上层而非下层。superiority基本
: 上更依赖下层的认知能力,surviving unknown and changing environment更依赖于上
: 层的认知能力。
: Nevertheless,第一呢,没有下层,也很难有上层。第二呢,在一个稳定的社会结构里
: ,生产的效率主要通过不断的细化分工来实现,于是每个人只要把自己所在岗位的工作
: 做好,做到极致,集体来看就是最优的;真正需要个人去面对未知和变化的环境的机会
: ,并不是很多。所以,这个也算你所说的fit into the social structure的一种。
: 最后,我自己做学生的时候时常听到一句话,“优秀是一种习惯”。我现在的理解是,

g***3
发帖数: 471
284
谢谢,月光,你也来了,
我一直认为,你有过人的思维,和独特的见解,,你说到的很多情况,经常增加了我新
的认知。
说到慈禧,,
这个例子,我也赞同,,我觉得人青少年以后,能够遇到原生家庭外的特别的机遇,特
殊的环境,可以完全改变一个人生的轨迹,
这个就是概率的问题了,,,,,
孩子们现在放假了,,他们来来去去频率多,我们这里白天,我有时穿插在他们活动间
,,写写我的看法,可能,有时,上下,不连续,,或者跑题,,
希望,大家看了,能明白,
非常开心,能够和大家分享父母的喜悦。

【在 t******u 的大作中提到】
: 慈禧太后最终统治了国家,是因为天天晚上跟皇上陪读,研墨,就等于皇上直接对她进
: 行了军事和政治的培训基础课了,当时女性不能参政的。
: 一个人后来能走出啥路线出来,直接取决于她成长过程中逐步地后来遇到了些什么人,
: 这些人才是真正对她命运发生重大影响的,跟父母见识有个鸟关系,胡锦涛让他儿子经
: 商了吗?让他女儿嫁给一个秃顶老头了吗,那个环境下遇上了,就只能这么走了,谁还
: 真能见识到自己过十年能当上国家主席啊

i**e
发帖数: 19242
285
简单地说就是要运气好嘛,呵呵
都是命
自己是什么档次的就容易遇到什么档次的人:)
这种运气和福份,不知道上苍是怎么分配的
也或许是每个人自己积攒earn的,好比佛教说积德的言行
anyway, 保持积极向上的心态是对的,谁都不敢说心念不会改变事情的走向不是?

【在 g***3 的大作中提到】
: 妹妹,我知道我这种思想,说出来,肯定会被大多数人,以为理论的理想。
: 我以前还没有象现在这么坚决的如此认为,,
: 爱情,不代表婚姻,,但是婚姻,一定要有爱情,为基点,,,人的一生,随着生活,
: 年龄的不同阶段,可能会有几段不同色彩的爱情,但是,如果希望要有孩子,那么这个
: 有孩子的婚姻,最希望保持一次,,
: 我现在越来越觉得,人生,就是一个漫长的规划,,是的,,爱情,不能预估,但是自
: 己的生活,可以规划,,自己的生活,可以自己驾驭,自己锻炼,我有女儿也有儿子,
: 他们也是往这个方向,规划自己的未来,,
: 我小时,因为姐姐哥哥都比我大很多,他们有一些恋爱经历,又由于我们那时群居,很
: 多邻居的孩子,姐姐哥哥朋友的事,我想都给从小的我,潜移默化的影响。

i**e
发帖数: 19242
286
抱抱
写的这么伤感啊:)
偶尔看到以前的帖子,一般会面红耳赤地默:真的是我写的?好自以为是啊
呵呵

【在 c***x 的大作中提到】
:
: 这一点我是完全同意的。
: 还记得Bloom's taxonomy吗?你我都是强调金字塔的上层而非下层。superiority基本
: 上更依赖下层的认知能力,surviving unknown and changing environment更依赖于上
: 层的认知能力。
: Nevertheless,第一呢,没有下层,也很难有上层。第二呢,在一个稳定的社会结构里
: ,生产的效率主要通过不断的细化分工来实现,于是每个人只要把自己所在岗位的工作
: 做好,做到极致,集体来看就是最优的;真正需要个人去面对未知和变化的环境的机会
: ,并不是很多。所以,这个也算你所说的fit into the social structure的一种。
: 最后,我自己做学生的时候时常听到一句话,“优秀是一种习惯”。我现在的理解是,

i**e
发帖数: 19242
287
我的感想就是
很多事情都是正态分布的,多数人是落在“大肚子”上的:)
恋爱择偶的时候,自然而然遵循的是 门当户对
虽然古时候是大鸣大放的“门当户对”
自由乱爱之后,本质还是 门当户对
门当户对是自然选择在婚姻上的诠释:)

【在 g***3 的大作中提到】
: 妹妹,我知道我这种思想,说出来,肯定会被大多数人,以为理论的理想。
: 我以前还没有象现在这么坚决的如此认为,,
: 爱情,不代表婚姻,,但是婚姻,一定要有爱情,为基点,,,人的一生,随着生活,
: 年龄的不同阶段,可能会有几段不同色彩的爱情,但是,如果希望要有孩子,那么这个
: 有孩子的婚姻,最希望保持一次,,
: 我现在越来越觉得,人生,就是一个漫长的规划,,是的,,爱情,不能预估,但是自
: 己的生活,可以规划,,自己的生活,可以自己驾驭,自己锻炼,我有女儿也有儿子,
: 他们也是往这个方向,规划自己的未来,,
: 我小时,因为姐姐哥哥都比我大很多,他们有一些恋爱经历,又由于我们那时群居,很
: 多邻居的孩子,姐姐哥哥朋友的事,我想都给从小的我,潜移默化的影响。

i**e
发帖数: 19242
288
找不到你帖子了
只是想说
你大儿子“觉醒”了,有很强烈的inner drive了
我一直以为男孩子就是这样成长的,有一个契机激发一下,他们就找到了自己有内动力了
恭喜你!

【在 g***3 的大作中提到】
: 这个我有很多周围人的具体例子,以后有时间来说。
f**********g
发帖数: 4709
289
俺们明明讨论的是什么情形下才能放手让娃发展自己的“正好不是当下社会中回报率最
高的那些行业职业最需要的raw talent”,怎么就成了富豪养废孩子了呢
这个前提是娃有talent又有passion,可是这个talent不一定能成功甚至可能以后找工
作困难,经济宽裕或者娃多的父母就不会为了担心娃将来能不能养活自己而强制扼杀娃
的兴趣,允许这个outlier的天赋能自由发展一段时间
比如说娃的兴趣天赋在纯数学在理论物理,父母不会为了担心他以后找不到工作要去
subway卖汉堡,就引导娃转学计算机或者经济金融
又或者娃的兴趣天赋在生物在化学,父母也不用担心以后一年只能赚三万五养不活妻儿
让他改把医生做目标
再或者娃的天赋兴趣在弹琴,在画画,在体育这些成材率太低的方面,父母也不用小心
翼翼苦口婆心的建议说,你看这个虽然好,可是一万个人里也不能出来一个,其他的养
活自己都困难,最好是当兴趣而不是当职业,你看咱要不要同时选个经济实惠的minor
……
这些可都是在咱工薪阶层的父母中实实在在发生着的纠结,
就连我们这代人,当初家里要是有能提供自费一两年的经济支持,很多人都不会因为全
额奖学金困在现在的专业里……

【在 i**e 的大作中提到】
: 第3个solution:顺其自然:)
: 我,不相信,富豪们对孩子就那点儿期望,甘心于养着孩子
: 没有自己的追求人生没有自己的立足点,100个米林养着,心里也是空虚的啊,能过好
: 吗?

i**e
发帖数: 19242
290
好吧 好吧 u win:)
look back
或许出国只不过是在当时看来的一个更好的选择21:)

【在 f**********g 的大作中提到】
: 俺们明明讨论的是什么情形下才能放手让娃发展自己的“正好不是当下社会中回报率最
: 高的那些行业职业最需要的raw talent”,怎么就成了富豪养废孩子了呢
: 这个前提是娃有talent又有passion,可是这个talent不一定能成功甚至可能以后找工
: 作困难,经济宽裕或者娃多的父母就不会为了担心娃将来能不能养活自己而强制扼杀娃
: 的兴趣,允许这个outlier的天赋能自由发展一段时间
: 比如说娃的兴趣天赋在纯数学在理论物理,父母不会为了担心他以后找不到工作要去
: subway卖汉堡,就引导娃转学计算机或者经济金融
: 又或者娃的兴趣天赋在生物在化学,父母也不用担心以后一年只能赚三万五养不活妻儿
: 让他改把医生做目标
: 再或者娃的天赋兴趣在弹琴,在画画,在体育这些成材率太低的方面,父母也不用小心

相关主题
觉得美帝数学进度慢的进来看一下请教:怎样说服孩儿他爸多关心孩子的学习?
分数应用题娃又在推算他的质数公式。。。
我招,我是猪大学录取率
进入Parenting版参与讨论
g***3
发帖数: 471
291
人的运气,从某种程度上,也是可以被诱发出来的。
这个说来话长,

【在 i**e 的大作中提到】
: 简单地说就是要运气好嘛,呵呵
: 都是命
: 自己是什么档次的就容易遇到什么档次的人:)
: 这种运气和福份,不知道上苍是怎么分配的
: 也或许是每个人自己积攒earn的,好比佛教说积德的言行
: anyway, 保持积极向上的心态是对的,谁都不敢说心念不会改变事情的走向不是?

g***3
发帖数: 471
292
门当户对是一小半因素,落那个娘肚子上这个是一大半因素。
什么样的妈妈,出什么样的孩子,这个我现在也看到太多了,,
所以,我常和孩子们说,交什么样的朋友,特别是他们以后的生活伴侣,要仔细观察他
们的妈妈,,
妈妈的正负影响,简直是烙印在他们的孩子们身上的,只是很多人自己也许从来没有意
识到罢了,
尤其是性格,,,

【在 i**e 的大作中提到】
: 我的感想就是
: 很多事情都是正态分布的,多数人是落在“大肚子”上的:)
: 恋爱择偶的时候,自然而然遵循的是 门当户对
: 虽然古时候是大鸣大放的“门当户对”
: 自由乱爱之后,本质还是 门当户对
: 门当户对是自然选择在婚姻上的诠释:)

g***3
发帖数: 471
293
那是很久的事了,以前我用的id 似乎是 gtdf ,因为后来怎么都再进不来,就后面加
了数字。
anyway 如果能够再找得到这个贴,现在来看应该很有意思,,
那个曾经对老大依附,更确切的说,利用感情,操从他的男孩,,现在可以说,和老大
完全是处于2个不同世界里的人,从初2后,他们就越来越少的交结,,更不说高中后了
,连在街上,也会很少遇到,
虽然我们一直居住在一条街的2个尽头,,记得当时,妹妹曾建议我搬家,,,

力了

【在 i**e 的大作中提到】
: 找不到你帖子了
: 只是想说
: 你大儿子“觉醒”了,有很强烈的inner drive了
: 我一直以为男孩子就是这样成长的,有一个契机激发一下,他们就找到了自己有内动力了
: 恭喜你!

i**e
发帖数: 19242
294
这个,你必须展开说说:)

【在 g***3 的大作中提到】
: 人的运气,从某种程度上,也是可以被诱发出来的。
: 这个说来话长,

i**e
发帖数: 19242
295
拼爹拼娘呗:)
还真就是耳濡目染
你再看看总结总结
小姑娘小伙子们有crush,fall in love的时候,都啥规律?
下意识地被跟自己父母相像的人吸引?
还是被自己个性相近 或相反的人吸引?
还是先走外貌协会的门?
还是被理念一致的人吸引?
挺有意思的:) 不知有木有人做研究

【在 g***3 的大作中提到】
: 门当户对是一小半因素,落那个娘肚子上这个是一大半因素。
: 什么样的妈妈,出什么样的孩子,这个我现在也看到太多了,,
: 所以,我常和孩子们说,交什么样的朋友,特别是他们以后的生活伴侣,要仔细观察他
: 们的妈妈,,
: 妈妈的正负影响,简直是烙印在他们的孩子们身上的,只是很多人自己也许从来没有意
: 识到罢了,
: 尤其是性格,,,

i**e
发帖数: 19242
296
说叉了:)
我的意思是本楼里你的一个帖子
表明你大儿子喜欢上了数学,喜欢上了商科的课程,对自己的前景由规划和向往
原来的帖子,真的是没啥印象了,要不有人说我是个老迷糊呢:)
你看,搬家这建议,太不靠谱了,呵呵

【在 g***3 的大作中提到】
: 那是很久的事了,以前我用的id 似乎是 gtdf ,因为后来怎么都再进不来,就后面加
: 了数字。
: anyway 如果能够再找得到这个贴,现在来看应该很有意思,,
: 那个曾经对老大依附,更确切的说,利用感情,操从他的男孩,,现在可以说,和老大
: 完全是处于2个不同世界里的人,从初2后,他们就越来越少的交结,,更不说高中后了
: ,连在街上,也会很少遇到,
: 虽然我们一直居住在一条街的2个尽头,,记得当时,妹妹曾建议我搬家,,,
:
: 力了

i**e
发帖数: 19242
297
生命里曾经的不愉快的过客给我们历练促使我们成长

【在 g***3 的大作中提到】
: 那是很久的事了,以前我用的id 似乎是 gtdf ,因为后来怎么都再进不来,就后面加
: 了数字。
: anyway 如果能够再找得到这个贴,现在来看应该很有意思,,
: 那个曾经对老大依附,更确切的说,利用感情,操从他的男孩,,现在可以说,和老大
: 完全是处于2个不同世界里的人,从初2后,他们就越来越少的交结,,更不说高中后了
: ,连在街上,也会很少遇到,
: 虽然我们一直居住在一条街的2个尽头,,记得当时,妹妹曾建议我搬家,,,
:
: 力了

h*****m
发帖数: 1034
298
这几天跟这个高楼,非常羡慕楼主三个优秀的孩子,特别是老大,不仅成绩好,还已经
到了能独立思考,还能帮助弟弟妹妹的成熟程度,俨然一个进取负责的男子汉形象了。
这肯定是与lz的培养分不开的。lz是有大智慧的人,不仅教育子女方面有丰富的经验,
关于人生,社会,家庭,事业也都有很多独特的见解。真是应了那句话,父母是孩子最
好的榜样。lz在这里跟大家分享自己教育子女的经验和心得体会,给人以很大的启迪。
这里我想请教一下,子女长大的过程中有没有身份认同的问题?我的两个小孩还小,在
上小学。有时推他们中文狠了些,或甚至要求他们吃饭时用筷子,他们不高兴的时候都
要说“我们是美国人,为什么要学中文,用筷子?”。我一般都是说“国籍上你们是美
国人,可美国人是从不同地方来的,我和妈妈是从中国来的是华人,你们是华裔美国人
,华人就要学中文用筷子。”一般他们就悻悻然的不说什么了,可过一阵又开始这个论
调。我感觉,“美国人”在他们心里的分量好像要比“华人”重。现在他们还小,我还
能压得住,不知大一些以后尤其是到了叛逆期,会不会对自己的华裔身份有认同问题。
尤其是中国日益崛起,像你这贴里说的“我也希望我们中国的强盛,他们能够到世界上
越多的地方占领”,但这样将来与美国发生摩擦的可能性也越来越大。如果有一天美国
和中国发生激烈的对抗,不知道孩子的“华裔”和“美国”双重身份会不会给他们带来
困扰。lz的孩子特别是将近成年的老大,对自己的华裔血统是怎样看待的?当法国和中
国有龌龊的时候,他是更倾向法国呢还是中国?当中国强盛了,“能够到世界上越多的
地方占领”时,威胁到法国的利益了,他是感到自豪呢,还是感到愤慨不安呢?

【在 g***3 的大作中提到】
: 我也希望我们中国的强盛,他们能够到世界上越多的地方占领,这个可以造福于我们的
: 后代。

h*****m
发帖数: 1034
299
你们的讨论发人深省,但对于这最后一点略有不同意见。实际社会是丰富多彩的,环境
可谓千差万别,在教室里尤其是评成绩时来一场“试演/彩排”未免有些太理想主义了。
不知怎么,看到这我就突然把它跟AA或diversity联系起来了,diversity的一个理论基
础不就是将来学生离开学校在社会里要接触到各族裔的人,所以学校里就要先“彩排”
一下吗?或者我太敏感了?
我觉得,大家还是各司其职。作为老师,在给予学生知识的基础上,尽量做到公平合理
才是最重要的。

【在 t******l 的大作中提到】
: 我个人的看法,intellectual 不是 built for superiority,而是 built for
: surviving unknown and changing environment。
: (其实 arguably,self-awareness 也是 built for survival。。。古人孙子说,知
: 己知彼。。。)
: 所以我个人觉得对教师而言,更重要的是这一场 “试演/彩排” 是不是逐步符合将来
: 的实际社会,而不是简单的公平合理。
:
: :
: :【 在 timefall (时光崩塌) 的大作中提到: 】

g***3
发帖数: 471
300
就拿最近一个例子说,
我们这里放假前1周,因为老公在外公差,然后回法国后,腿上静脉曲张,要准备一个
手术,我需要在家照顾他。 老大很想去海边,想放松一下紧张的状态,他跟以前初中
最铁的2个伙伴计划到西南海岸去租一个小房子,然后去冲浪,,
他跟我们说起,我让他安排好以后,给我们具体方案和预算,,
他们都是高3学生,其他2个都已18岁了,,只是目前有好多考试,,5,6月都有各类科
目的全国会考,,
前2天,那2个伙伴犹豫着,最后似乎不想去,要在家复习,老大跟我说起来,很沮丧,
我们都劝他,他太想去南方海边冲浪了,他说他自己一个人去,想要带妹妹一起,但是
妹妹我已安排课外学习方法课活动了,,妹妹说,一个人去多没意思,,,,
昨天下午,他高兴的告诉我,一个女同学,邀请他去他们海边的度假别墅,,她是冲浪
好手,还可以教他云云,,,
昨天我们很多事,就没再说下去,,
刚才中午,和儿子一起做午饭,,我就问他了,怎么她邀请他,他说,他跟同学们在说
,好想去冲浪,可惜没运气啊,那个女同学看到了老大和别人的对话,就邀请他了,就
这么简单,,
其实这个女同学,老大今年才开始认识她,以前他们那一个帮都是从初中就都是好朋友
,老大不好意思直接跟她说,但是我想,老大大概是有意这么做的,

【在 i**e 的大作中提到】
: 这个,你必须展开说说:)
相关主题
别鸡兔同笼了,来喝啤酒吧关于刷还是不刷
学校早晚要教的东西,早早学会了又如何?是不是任我儿子继续迷象棋
构建式数学在中国9岁, 如何报名考amc10
进入Parenting版参与讨论
g***3
发帖数: 471
301
没有,当时,搬家这个,我们还真考虑了,
只是,我们住的地方,已经是一个相对,理想的人文教育环境最佳的地方,还有,我们
特别造了自己的房子,,,
这件事,还是坚持了4,5年的持久战,孩子自己一点点意识到对人的认识,负量感情的
危险,,
这件事,他一生会记得,,

【在 i**e 的大作中提到】
: 说叉了:)
: 我的意思是本楼里你的一个帖子
: 表明你大儿子喜欢上了数学,喜欢上了商科的课程,对自己的前景由规划和向往
: 原来的帖子,真的是没啥印象了,要不有人说我是个老迷糊呢:)
: 你看,搬家这建议,太不靠谱了,呵呵

g***3
发帖数: 471
302
这个我也很感兴趣的,
我特别注意附近小孩子成长,他们父母性格类型,,,,说来话很长,,

【在 i**e 的大作中提到】
: 拼爹拼娘呗:)
: 还真就是耳濡目染
: 你再看看总结总结
: 小姑娘小伙子们有crush,fall in love的时候,都啥规律?
: 下意识地被跟自己父母相像的人吸引?
: 还是被自己个性相近 或相反的人吸引?
: 还是先走外貌协会的门?
: 还是被理念一致的人吸引?
: 挺有意思的:) 不知有木有人做研究

g***3
发帖数: 471
303
很开心,我的一些心得,能够被越来越多的妹妹们认可。你提的问题,我们都经历过,
我和大家探讨一下我自己的心得。
子女长大的过程中有没有身份认同的问题?
当然有,,,这个首先取决于家长自己,在外国生活工作的经历和感受。
我自己是20岁出头到的法国,因为已是成人,学习,工作中,有时遇到一些同事开中国
的玩笑,我可能因为是成人,而且自己对自己蛮有信心的,感受似乎并不强烈。
孩子从小就跟我去国内,随着国内经济发展长大,他们是很为中国骄傲的。他们看到的
上海,比法国大多数城市,都要发达,,他们最亲的大家庭,都是我上海的父母姐姐姐
夫舅舅表哥,,
在法国,我们居住的地方,居民文化教育程度相比全国,较高的水平,很多人从事的工
作,很多都会和中国有联系的,很多人也经常公差,,所以对中国,对上海的概念,比
那些从来没有出过国的人群认知上开阔很多,孩子们从幼儿园,就开始和这里的孩子一
起长大,,他们的外貌,对他们来说,是平常的,
直到有一天,老大7岁左右,在一次和外省学校足球比赛中,一个黑人孩子,不知为何
,进攻我儿子,骂他小中国人,老大当时,一傻,老大的同伴们就一起冲向这个黑孩子
,为儿子抱不平,这件事,老大还是很难过的,我开始跟他们聊妈妈的历史,为什么来
到法国,当时中国是那个状态,等等, 妹妹,一直和哥哥在一个学校,哥哥朋友很多
,妹妹小时,也和哥哥的朋友们一起玩,妹妹到是很少遇到如此问题。我们旅行很多,
每年夏天,都回上海,喜欢那里的吃,玩,大一点后,那里流行的衣服,影碟,等等,
他们小时候,在国内,别人问,他们会说是法国人,在法国,他们会承认妈妈是上海人
,,他们一直为我骄傲吧,也许我做的行业,能够让他们接触到国内的一些大工厂的老
板,欧洲这里的工厂,,
他们初中后,学校学到的有关中国的地理知识,也会让他们骄傲,,比如,记得有一次
,女儿说,上海是世界上吞吐量最大的港口,,,,他们越大,越对中国感兴趣,,也
为自己有中国血统而骄傲。
也越会对意外的事,应付自如,,比如,我看到网上讨论被歧视的事,就跟他们讨论,
,妹妹说,刚开始上初中时,有一次,在食堂吃饭,有一个男同学,说她是中国春卷,
,女儿马上回击,,就是中国春卷,也比rien好,,法语里,,rien有双层意思,有没
有,和愚蠢极点的意思,,那个男同学马上就住嘴了,,,

【在 h*****m 的大作中提到】
: 这几天跟这个高楼,非常羡慕楼主三个优秀的孩子,特别是老大,不仅成绩好,还已经
: 到了能独立思考,还能帮助弟弟妹妹的成熟程度,俨然一个进取负责的男子汉形象了。
: 这肯定是与lz的培养分不开的。lz是有大智慧的人,不仅教育子女方面有丰富的经验,
: 关于人生,社会,家庭,事业也都有很多独特的见解。真是应了那句话,父母是孩子最
: 好的榜样。lz在这里跟大家分享自己教育子女的经验和心得体会,给人以很大的启迪。
: 这里我想请教一下,子女长大的过程中有没有身份认同的问题?我的两个小孩还小,在
: 上小学。有时推他们中文狠了些,或甚至要求他们吃饭时用筷子,他们不高兴的时候都
: 要说“我们是美国人,为什么要学中文,用筷子?”。我一般都是说“国籍上你们是美
: 国人,可美国人是从不同地方来的,我和妈妈是从中国来的是华人,你们是华裔美国人
: ,华人就要学中文用筷子。”一般他们就悻悻然的不说什么了,可过一阵又开始这个论

g***3
发帖数: 471
304
有关学习中文,,
他们出生时,我就请了一位南方大学,普通话说得非常优雅的老师,来我家帮助我,照
顾2个孩子,
他们会说的第一个词,是蘑菇,大象等等,,我老公以前在国内工作时,会一点中文,
后来又在法国上夜校,但是坚持不了,他鼓励我们说中文。2个孩子上幼儿园后,开始
法语说得多,我没有象很多书上写的,双语孩子,妈妈要坚持说一种语言,爸爸要坚持
说另一种,我以为,语言是一种表达方式,是体现心理最直接的交流方式,
我希望孩子们在交流上,每天能够畅所预言,,不受任何阻碍,随着身体的长大,他们
接触的事物广阔,,很多词汇,中文根本不会,连我自己都不会,很多物质,现象,都
是我出国后,才出现的,,
有时,他们激动,马上要告诉我,就越来越多法语,,我和他们说法语,也说中文,,
根据不同的场景,老大6岁,妹妹4岁时,我们送他们每周星期天,中文学校,学习写字
,,2年后,也许当时教学内容,比较不符合法国这里的实际情况,孩子们没有兴趣,
作业也不主动,老公觉得,这样每个星期天,可以做其他更有意义的活动,我们就放弃
了。
不过,每天夏天,我们回上海,还有我的工作一直坚持到老3出生,所以,2个大的经常
和我回国。
在国内,他们参加当地活动,,他们都踢过足球,打过网球,学过乒乓,女儿还学习中
国舞蹈,,
女儿在上初中时,选了中文为2外,,老大在初4时,开始发奋学习中文,现在他的中文
水平,可以看简单的文章,可以用微信简单的答复,他们学习的内容,有中国当代地理
经济,比如计划生育,中国旅游习惯的变化,,古代文化,比如中药,李时珍,本草纲
目,,,西游记,
女儿现在是英文,德语,中文,他们的语言能力都非常好,
老3,现在是能听懂,没有强迫他学写字,他是英文,中文,德语,,
有关语言,我觉得,孩子们到10多岁后,自己有意识了,会很快学好的,
使用筷子,这个在我家,他们从小就习惯的,没有特殊的问题,,看吃的是什么,中餐
用筷子,西餐用刀叉。
我觉得孩子们,一定年龄认知,我和他们说一定程度的话,,有时候,想想,我们几十
年的感受,他们几岁的年龄,怎么会一下子明白尼,

【在 h*****m 的大作中提到】
: 这几天跟这个高楼,非常羡慕楼主三个优秀的孩子,特别是老大,不仅成绩好,还已经
: 到了能独立思考,还能帮助弟弟妹妹的成熟程度,俨然一个进取负责的男子汉形象了。
: 这肯定是与lz的培养分不开的。lz是有大智慧的人,不仅教育子女方面有丰富的经验,
: 关于人生,社会,家庭,事业也都有很多独特的见解。真是应了那句话,父母是孩子最
: 好的榜样。lz在这里跟大家分享自己教育子女的经验和心得体会,给人以很大的启迪。
: 这里我想请教一下,子女长大的过程中有没有身份认同的问题?我的两个小孩还小,在
: 上小学。有时推他们中文狠了些,或甚至要求他们吃饭时用筷子,他们不高兴的时候都
: 要说“我们是美国人,为什么要学中文,用筷子?”。我一般都是说“国籍上你们是美
: 国人,可美国人是从不同地方来的,我和妈妈是从中国来的是华人,你们是华裔美国人
: ,华人就要学中文用筷子。”一般他们就悻悻然的不说什么了,可过一阵又开始这个论

g***3
发帖数: 471
305
人是感情动物,一个地方住久了,接触当地的事物达到熟同自己的程度时,肯定会把当
地的事物无意识转移认同到自己身上,如果你对孩子否定他的认同,就如同你否认他的
经历和感受存在,孩子们会以自己有限的力量,来维护自己的感受,他们就会出现对抗
和厌恶的表现。
不知你们孩子和中国接触时间的频率多少,,这个奠定了他们对中国的认识多少,
他们自己的认识,不是家长强加于他们,要他们知道的认知,
孩子自己的认知,和家长的认知,是2回事,,孩子自己的认知,是通过他们自己接触
人,接触事物,吃饭,玩耍,学习,听周围人的事,感受周围人的情感,,,感受周围
人对他们关注的程度,,,
一方面,你们应该高兴,如果孩子认为自己是美国人,说明他们认为美国好,也说明爸
爸妈妈给的在美国的生活好,这个是完全正面的,另一方面,,也说明孩子有强烈鲜明
的感情,这个是做大多事的最好动力。
孩子会随年龄的增长,自己书面知识的开阔,更深层次的看问题,社会不是极黑就白,
更何况现在的大社会,将来的社会,会没有分界,没有种族,,,只是利益的分配,
匆匆,希望自己说的明白。

【在 h*****m 的大作中提到】
: 这几天跟这个高楼,非常羡慕楼主三个优秀的孩子,特别是老大,不仅成绩好,还已经
: 到了能独立思考,还能帮助弟弟妹妹的成熟程度,俨然一个进取负责的男子汉形象了。
: 这肯定是与lz的培养分不开的。lz是有大智慧的人,不仅教育子女方面有丰富的经验,
: 关于人生,社会,家庭,事业也都有很多独特的见解。真是应了那句话,父母是孩子最
: 好的榜样。lz在这里跟大家分享自己教育子女的经验和心得体会,给人以很大的启迪。
: 这里我想请教一下,子女长大的过程中有没有身份认同的问题?我的两个小孩还小,在
: 上小学。有时推他们中文狠了些,或甚至要求他们吃饭时用筷子,他们不高兴的时候都
: 要说“我们是美国人,为什么要学中文,用筷子?”。我一般都是说“国籍上你们是美
: 国人,可美国人是从不同地方来的,我和妈妈是从中国来的是华人,你们是华裔美国人
: ,华人就要学中文用筷子。”一般他们就悻悻然的不说什么了,可过一阵又开始这个论

x***1
发帖数: 999
306
满满的正能量

【在 g***3 的大作中提到】
: 有关学习中文,,
: 他们出生时,我就请了一位南方大学,普通话说得非常优雅的老师,来我家帮助我,照
: 顾2个孩子,
: 他们会说的第一个词,是蘑菇,大象等等,,我老公以前在国内工作时,会一点中文,
: 后来又在法国上夜校,但是坚持不了,他鼓励我们说中文。2个孩子上幼儿园后,开始
: 法语说得多,我没有象很多书上写的,双语孩子,妈妈要坚持说一种语言,爸爸要坚持
: 说另一种,我以为,语言是一种表达方式,是体现心理最直接的交流方式,
: 我希望孩子们在交流上,每天能够畅所预言,,不受任何阻碍,随着身体的长大,他们
: 接触的事物广阔,,很多词汇,中文根本不会,连我自己都不会,很多物质,现象,都
: 是我出国后,才出现的,,

f**********g
发帖数: 4709
307
这个说的不错,可是吧,跟伴侣相处那么久都没看出来的问题,也不能指望跟丈母娘/
婆婆见一两次面就能看出来啊,除非极品

【在 g***3 的大作中提到】
: 门当户对是一小半因素,落那个娘肚子上这个是一大半因素。
: 什么样的妈妈,出什么样的孩子,这个我现在也看到太多了,,
: 所以,我常和孩子们说,交什么样的朋友,特别是他们以后的生活伴侣,要仔细观察他
: 们的妈妈,,
: 妈妈的正负影响,简直是烙印在他们的孩子们身上的,只是很多人自己也许从来没有意
: 识到罢了,
: 尤其是性格,,,

f**********g
发帖数: 4709
308
我觉得大家实在不必对中小学老师期望太高,
只要想想,华人有没有人推自己娃当中小学老师的?你自己的小学中学同学中都是什么
样的人在当中小学老师?就知道中小学老师是个什么样的人群,他们的见识和能力水平
并不会随着自己进入“老师”这个职业就自动上一个档次

了。

【在 h*****m 的大作中提到】
: 你们的讨论发人深省,但对于这最后一点略有不同意见。实际社会是丰富多彩的,环境
: 可谓千差万别,在教室里尤其是评成绩时来一场“试演/彩排”未免有些太理想主义了。
: 不知怎么,看到这我就突然把它跟AA或diversity联系起来了,diversity的一个理论基
: 础不就是将来学生离开学校在社会里要接触到各族裔的人,所以学校里就要先“彩排”
: 一下吗?或者我太敏感了?
: 我觉得,大家还是各司其职。作为老师,在给予学生知识的基础上,尽量做到公平合理
: 才是最重要的。

t******l
发帖数: 10908
309
我觉得对于:(1)系统性政策性的歧视;(2)个人的主观但有一定程度恶意的歧视;
(3)个人的主观但无恶意的歧视;(4)个人的主观上并没有歧视的意图,但客观上因
为各种原因造成的实际上的个人的歧视;这四种不同的情况应该分情况具体讨论。

了。

【在 f**********g 的大作中提到】
: 我觉得大家实在不必对中小学老师期望太高,
: 只要想想,华人有没有人推自己娃当中小学老师的?你自己的小学中学同学中都是什么
: 样的人在当中小学老师?就知道中小学老师是个什么样的人群,他们的见识和能力水平
: 并不会随着自己进入“老师”这个职业就自动上一个档次
:
: 了。

x***1
发帖数: 999
310
说得真好。
在孩子功课上,我感觉大家是一心一意的,始终是真诚的,方方面面,不敢马虎。
在孩子的中文上,我是得过且过,只觉得是一种第二语言。我一直主张,孩子将来是在
生他的地方生活,是在和他们同龄的我们认为是国外的人打交道,让他们尽量地融到当
地人的生活中,多和当地的人交朋友,养成当地人的风土人情,多灌输当地人的优秀。
实际上颜色并不重要,而你的亲和的态度,愿意和别人一起进步的团结精神才是关键。
不要动不动就往颜色上扯,自己先给自己建一堵墙,主动和别人分开。
错就错在我们赖在这儿不回去。

【在 g***3 的大作中提到】
: 人是感情动物,一个地方住久了,接触当地的事物达到熟同自己的程度时,肯定会把当
: 地的事物无意识转移认同到自己身上,如果你对孩子否定他的认同,就如同你否认他的
: 经历和感受存在,孩子们会以自己有限的力量,来维护自己的感受,他们就会出现对抗
: 和厌恶的表现。
: 不知你们孩子和中国接触时间的频率多少,,这个奠定了他们对中国的认识多少,
: 他们自己的认识,不是家长强加于他们,要他们知道的认知,
: 孩子自己的认知,和家长的认知,是2回事,,孩子自己的认知,是通过他们自己接触
: 人,接触事物,吃饭,玩耍,学习,听周围人的事,感受周围人的情感,,,感受周围
: 人对他们关注的程度,,,
: 一方面,你们应该高兴,如果孩子认为自己是美国人,说明他们认为美国好,也说明爸

相关主题
9岁, 如何报名考amc10算不算有数学天分
求科普: 奥数 vs. Math Olympiad vs. Math Count vs. Math circle vs. 超前学数学大家怎么推孩子的弱项?
有明天考AMC 8的吗?少年智力开发
进入Parenting版参与讨论
t******l
发帖数: 10908
311
我个人觉得智人所说的 理智 vs 感情,其实是 space-time 上的 high dimension
pattern 的表象而已。比如其中一个 dimension 是 systematic vs sporadic,比如其
中另一个 dimension 是 causal vs statistical vs chaotic,等等等等。。。
其实我实际上想说的是:不同的智人 / 不同的AlphaGo 对相同的玩意儿的反应可能完
全不一样,不一定需要强求一致。

【在 g***3 的大作中提到】
: 人是感情动物,一个地方住久了,接触当地的事物达到熟同自己的程度时,肯定会把当
: 地的事物无意识转移认同到自己身上,如果你对孩子否定他的认同,就如同你否认他的
: 经历和感受存在,孩子们会以自己有限的力量,来维护自己的感受,他们就会出现对抗
: 和厌恶的表现。
: 不知你们孩子和中国接触时间的频率多少,,这个奠定了他们对中国的认识多少,
: 他们自己的认识,不是家长强加于他们,要他们知道的认知,
: 孩子自己的认知,和家长的认知,是2回事,,孩子自己的认知,是通过他们自己接触
: 人,接触事物,吃饭,玩耍,学习,听周围人的事,感受周围人的情感,,,感受周围
: 人对他们关注的程度,,,
: 一方面,你们应该高兴,如果孩子认为自己是美国人,说明他们认为美国好,也说明爸

t******l
发帖数: 10908
312
当然常见例子里的更复杂的,比如对 individual 而言是个 near pure chaotic 的
process,但对于 collective 而言却成为一个 statistically directed behavior。
。。比如 fish schooling、bird flocking,当然不那么傻鼻的比如 orca pod。。。
对于这一类的研究,俗称:社会学。。。而古人云:也就是不要 “见树不见林” 的学
问。。。

【在 t******l 的大作中提到】
: 我个人觉得智人所说的 理智 vs 感情,其实是 space-time 上的 high dimension
: pattern 的表象而已。比如其中一个 dimension 是 systematic vs sporadic,比如其
: 中另一个 dimension 是 causal vs statistical vs chaotic,等等等等。。。
: 其实我实际上想说的是:不同的智人 / 不同的AlphaGo 对相同的玩意儿的反应可能完
: 全不一样,不一定需要强求一致。

g***3
发帖数: 471
313
你说到要害处了,所以啊所以,我很希望,他们能够在决定自己终身大事前,多观察人
,多有一点接触人,待人的经验,多和将来可能的那个家庭未来成员接触,,,
我自己的痛,就是对婆家婚前不了解,尤其婆婆,孩子们知道的,,,

【在 f**********g 的大作中提到】
: 这个说的不错,可是吧,跟伴侣相处那么久都没看出来的问题,也不能指望跟丈母娘/
: 婆婆见一两次面就能看出来啊,除非极品

g***3
发帖数: 471
314
是是,我说的是一般的泛指,大多数的情况,

【在 t******l 的大作中提到】
: 我个人觉得智人所说的 理智 vs 感情,其实是 space-time 上的 high dimension
: pattern 的表象而已。比如其中一个 dimension 是 systematic vs sporadic,比如其
: 中另一个 dimension 是 causal vs statistical vs chaotic,等等等等。。。
: 其实我实际上想说的是:不同的智人 / 不同的AlphaGo 对相同的玩意儿的反应可能完
: 全不一样,不一定需要强求一致。

g***3
发帖数: 471
315
这个,我也有同感。

【在 f**********g 的大作中提到】
: 我觉得大家实在不必对中小学老师期望太高,
: 只要想想,华人有没有人推自己娃当中小学老师的?你自己的小学中学同学中都是什么
: 样的人在当中小学老师?就知道中小学老师是个什么样的人群,他们的见识和能力水平
: 并不会随着自己进入“老师”这个职业就自动上一个档次
:
: 了。

g***3
发帖数: 471
316
是的,语言首先是搭建感情的桥梁。
桥梁搭得稳健了,我们再在桥梁上修饰精美。
我2大孩子现在人大了,意识到学习中文的重要性,他们要求我跟他们坚持说中文,女
儿说,即使他们有时说不出来,用法语,或英语回答我,但是女儿说,单单多听,也是
对他们的练习。

【在 x***1 的大作中提到】
: 说得真好。
: 在孩子功课上,我感觉大家是一心一意的,始终是真诚的,方方面面,不敢马虎。
: 在孩子的中文上,我是得过且过,只觉得是一种第二语言。我一直主张,孩子将来是在
: 生他的地方生活,是在和他们同龄的我们认为是国外的人打交道,让他们尽量地融到当
: 地人的生活中,多和当地的人交朋友,养成当地人的风土人情,多灌输当地人的优秀。
: 实际上颜色并不重要,而你的亲和的态度,愿意和别人一起进步的团结精神才是关键。
: 不要动不动就往颜色上扯,自己先给自己建一堵墙,主动和别人分开。
: 错就错在我们赖在这儿不回去。

g***3
发帖数: 471
317
还有一点,年龄越小的孩子,对事自然的感情,越占大部,,我觉得理智,是随年龄经
验的增长,相对递增。当然,有些人,可能永远理智大于感情,有些人可能永远用感情
来处事,这个又和他们从娘胎里,和出生后前几年,大脑器官内部机构的演变正反记忆
积累相关,,,

【在 t******l 的大作中提到】
: 我个人觉得智人所说的 理智 vs 感情,其实是 space-time 上的 high dimension
: pattern 的表象而已。比如其中一个 dimension 是 systematic vs sporadic,比如其
: 中另一个 dimension 是 causal vs statistical vs chaotic,等等等等。。。
: 其实我实际上想说的是:不同的智人 / 不同的AlphaGo 对相同的玩意儿的反应可能完
: 全不一样,不一定需要强求一致。

h*****m
发帖数: 1034
318
多谢分享!小声说一句,我是个男地,混入妈妈队伍的爸爸。: )
真的很羡慕你有很多的机会带孩子回中国增强他们对中国认知和感情上的联系,尤其是
能够引起他们自豪感的发达城市上海。我们带他们回去的机会不多,但我也经常有意识
地给他们讲中国是多么的伟大,希望他们能以自己的华裔身份骄傲。比如说,为什么中
国叫"China",是因为古代瓷器这种高科技产品只有中国能够生产,商人要不远万里把
它运到欧洲,还有丝绸以及丝绸之路,以及连美洲的发现都与中国有关 - 通往东方的
线路受阻促使欧洲人开发海上航线。近代中国落后了乏善可讲,幸好现在中国又在赶超
,下次中国发射火箭的时候也可以好好讲讲。他们倒也对于中国有关的事情感兴趣,昨
天弟弟看到个什么东西上面有个Made in China,就赶紧跑过来给我看看。现在想来,
我之前说的他们说自己是"美国人“更有可能是找个借口来逃避学习。
我是真心希望在美国这个所谓的种族大熔炉里每个人都只把自己当作“美国人“,不以
人的肤色种族来判断别人,可是实际中他们的相貌就决定了别人眼中他们的种族。不知
道法国怎么样,美国这里还是有基于种族的一些政策,将来他们升学或工作时都有可能
受到影响,不知道到时会不会对他们的身份认同造成困扰。
你的女儿口齿伶俐,能及时恰到好处地反击,真是厉害。之前我家老大曾经和一黑人小
孩发生争吵,对方说他是Chinese如何如何,他就不知道怎么说。幸好,他也没说对方
是黑人如何。我给老师写了封措词比较严肃的信,学校用一种和稀泥的态度敷衍过去。
或许问题根本没有那么严重,或许到时候这些所谓的问题都不再是问题,但现在想起来
总是有点淡淡的忧虑。。

【在 g***3 的大作中提到】
: 很开心,我的一些心得,能够被越来越多的妹妹们认可。你提的问题,我们都经历过,
: 我和大家探讨一下我自己的心得。
: 子女长大的过程中有没有身份认同的问题?
: 当然有,,,这个首先取决于家长自己,在外国生活工作的经历和感受。
: 我自己是20岁出头到的法国,因为已是成人,学习,工作中,有时遇到一些同事开中国
: 的玩笑,我可能因为是成人,而且自己对自己蛮有信心的,感受似乎并不强烈。
: 孩子从小就跟我去国内,随着国内经济发展长大,他们是很为中国骄傲的。他们看到的
: 上海,比法国大多数城市,都要发达,,他们最亲的大家庭,都是我上海的父母姐姐姐
: 夫舅舅表哥,,
: 在法国,我们居住的地方,居民文化教育程度相比全国,较高的水平,很多人从事的工

h*****m
发帖数: 1034
319
说来惭愧,我督促他们说中文,学中文的一大动力就是感觉自己的英文不够好,现在简
单的日常话题还能应付,但怕他们长大以后与他们的交流受到阻碍。所以从小开始,就
一直坚持在家里必须说中文,他们不会说的中文词汇用英文说出来,我翻译给他们,再
让他们重复说一次。平时给他们讲故事时遇到估计他们不懂得词儿,也停下来解释一下
。也让他们看中文动画片,在网上听中文故事。至少现在还在坚持,希望能坚持到象你
家孩子那样认识到学中文重要性的年龄,我就放心了。
还是有效果的,昨天老二叽叽喳喳跟我讲他的一个同学是如何淘气的时候,很顺畅地说
“他竟敢在老师耳朵边上大喊“。我都不记得什么时候给他讲过“竟敢”这个词儿了,
也可能是从那个动画片里学的吧。
等他们大些了,也准备每年在暑假把他们放回国内待些日子,据说对提高中文水平大有
帮助。

【在 g***3 的大作中提到】
: 有关学习中文,,
: 他们出生时,我就请了一位南方大学,普通话说得非常优雅的老师,来我家帮助我,照
: 顾2个孩子,
: 他们会说的第一个词,是蘑菇,大象等等,,我老公以前在国内工作时,会一点中文,
: 后来又在法国上夜校,但是坚持不了,他鼓励我们说中文。2个孩子上幼儿园后,开始
: 法语说得多,我没有象很多书上写的,双语孩子,妈妈要坚持说一种语言,爸爸要坚持
: 说另一种,我以为,语言是一种表达方式,是体现心理最直接的交流方式,
: 我希望孩子们在交流上,每天能够畅所预言,,不受任何阻碍,随着身体的长大,他们
: 接触的事物广阔,,很多词汇,中文根本不会,连我自己都不会,很多物质,现象,都
: 是我出国后,才出现的,,

h*****m
发帖数: 1034
320
哈哈,你有点搞糊涂了。
他们是从另外一个楼里穿越来的,说的话题跟这里讨论的中学生学习貌似没什么关系,
至少他们说的“老师“不一定是指中小学老师。
我也没有跟全他们的论道过程,只是看到了那篇帖子,有感而发罢了。

【在 f**********g 的大作中提到】
: 我觉得大家实在不必对中小学老师期望太高,
: 只要想想,华人有没有人推自己娃当中小学老师的?你自己的小学中学同学中都是什么
: 样的人在当中小学老师?就知道中小学老师是个什么样的人群,他们的见识和能力水平
: 并不会随着自己进入“老师”这个职业就自动上一个档次
:
: 了。

相关主题
[转载] 为什么说usamo 简单分数应用题
二年级女儿的report我招,我是猪
觉得美帝数学进度慢的进来看一下请教:怎样说服孩儿他爸多关心孩子的学习?
进入Parenting版参与讨论
t******l
发帖数: 10908
321
关于这个问题,我觉得不妨先听/看一下音乐中的爵士因子,比如这首法文小曲儿的三
个不同的版本:
http://www.youtube.com/watch?v=JWfsp8kwJto
http://www.youtube.com/watch?v=hL4zY5KNVmg
http://www.youtube.com/watch?v=xXBNlApwh0c

【在 g***3 的大作中提到】
: 还有一点,年龄越小的孩子,对事自然的感情,越占大部,,我觉得理智,是随年龄经
: 验的增长,相对递增。当然,有些人,可能永远理智大于感情,有些人可能永远用感情
: 来处事,这个又和他们从娘胎里,和出生后前几年,大脑器官内部机构的演变正反记忆
: 积累相关,,,

t******l
发帖数: 10908
322
如果觉得很感性的话,那不妨再看看下面这个 video:
http://www.youtube.com/watch?v=stsBqdo9Svc

【在 t******l 的大作中提到】
: 关于这个问题,我觉得不妨先听/看一下音乐中的爵士因子,比如这首法文小曲儿的三
: 个不同的版本:
: http://www.youtube.com/watch?v=JWfsp8kwJto
: http://www.youtube.com/watch?v=hL4zY5KNVmg
: http://www.youtube.com/watch?v=xXBNlApwh0c

t******l
发帖数: 10908
323
看完上面这些 video,就会发现,其实所谓的那些 “感性”,不过就是一些 space-
time-pattern,用一种 educated sporadic-chaotic 的方式,比如就这么整:
Spontaneous Symmetry Breaking
-- by timefall, 2016.04.18
You play major,
You play minor;
You sound augmented,
You sound diminished;
Then, ... (You gave me perfect-fifth?)
You syncopate left,
You swing right;
You swing loose,
You syncopate tight;
Then, you synchronize... (By the way, anybody know who is Bach?)
But who made your vibrato up,
And who made your vibrato down;
I don't know which pitch go relax,
while others' come with tense;
(I just forgot what pitch-absolute means.)
(Curiously) How your vowels open,
with your consonants labial;
And how your consonants nasal,
with your vowels closed;
Then what, you rest? now?... (But everybody impressed.)
So you think you are emotional,
But do you?
t******l
发帖数: 10908
324
虽然这是一首破烂尿不湿,但说明的道理是:尽管不会数学证明的数学学渣们,觉得
“理智” 这种 veli veli directive 的玩意儿是多么的高大上。但事实就是 “veli
veli directive” 这枚硬币的另一面就是 “veli veli biasd” -- 在这个问题上就
是你必须知道确切的(known)目标函数,否则整个 optimization system
pathetically fail。。。
而真实世界的情况就是,波切利这枚小破 super-string 在台上唱歌的时候,这哥们哪
里知道台下和远方的 N 多听众一个一个小破 super-string 的确切 formula。退一步
说,就算知道,那哥们比阿发狗差一百万被的猴脑算得过来用啥频率让大部分小破
super-string 发生共振么?所以这哥们唯一的办法就是有限度地放弃 “理智”,也就
是 directive approach。而代之以 educated sporadic-chaotic,来对付 partially-
unknown objective function 的环境 -- 也就是智人所说的 “情感”。。。其实说到
底万变不离其宗还是 space-time-pattern。。。

【在 t******l 的大作中提到】
: 看完上面这些 video,就会发现,其实所谓的那些 “感性”,不过就是一些 space-
: time-pattern,用一种 educated sporadic-chaotic 的方式,比如就这么整:
: Spontaneous Symmetry Breaking
: -- by timefall, 2016.04.18
: You play major,
: You play minor;
: You sound augmented,
: You sound diminished;
: Then, ... (You gave me perfect-fifth?)
: You syncopate left,

t******l
发帖数: 10908
325
现在无轨电车拉回小娃的 emotional 的问题:
虽然 control-freak 型的推妈们,总是一根筋儿的认为一定能找到娃将来任何时刻的
目标函数,精确到每一个 quantum-bit。。。但上帝这位虽然搞些 alpha release 幺
蛾子,就急着出货的急性子马工,但还不至于是数学学渣。。。所以上帝这位马工,事
先就在父母和小孩的大脑里,直接都硬件代码 flash-ROM 了那些 educated sporadic-
chaotic 的 BIOS code,这样一出生就能直接 boot-load 了。。。所以还是那句话:
So you think you are emotional,
But do you?
当然,话说回来,alpha release 的个别产品出点幺蛾子也是家常便饭,但目前上帝好
像不负责 BIOS 重刷的 Windows Update。。。这个估计要等刷试管的千老刷到那步,
那时大伙儿随便下载个 patch 重刷大脑的 BIOS 也就不是个事儿,但现在估计不行。
。。

veli
partially-

【在 t******l 的大作中提到】
: 虽然这是一首破烂尿不湿,但说明的道理是:尽管不会数学证明的数学学渣们,觉得
: “理智” 这种 veli veli directive 的玩意儿是多么的高大上。但事实就是 “veli
: veli directive” 这枚硬币的另一面就是 “veli veli biasd” -- 在这个问题上就
: 是你必须知道确切的(known)目标函数,否则整个 optimization system
: pathetically fail。。。
: 而真实世界的情况就是,波切利这枚小破 super-string 在台上唱歌的时候,这哥们哪
: 里知道台下和远方的 N 多听众一个一个小破 super-string 的确切 formula。退一步
: 说,就算知道,那哥们比阿发狗差一百万被的猴脑算得过来用啥频率让大部分小破
: super-string 发生共振么?所以这哥们唯一的办法就是有限度地放弃 “理智”,也就
: 是 directive approach。而代之以 educated sporadic-chaotic,来对付 partially-

c***x
发帖数: 1826
326

谢谢楼主分享经验,同赞正能量。
我们觉得满满的正能量,充分说明了人类社会里的正反馈机制的重要性,最后就体现为
马太效应。
这个也算是为推爸推妈们“不要输在起跑线上”的“感性”行为提供了一种“理性”解
释。

【在 g***3 的大作中提到】
: 有关学习中文,,
: 他们出生时,我就请了一位南方大学,普通话说得非常优雅的老师,来我家帮助我,照
: 顾2个孩子,
: 他们会说的第一个词,是蘑菇,大象等等,,我老公以前在国内工作时,会一点中文,
: 后来又在法国上夜校,但是坚持不了,他鼓励我们说中文。2个孩子上幼儿园后,开始
: 法语说得多,我没有象很多书上写的,双语孩子,妈妈要坚持说一种语言,爸爸要坚持
: 说另一种,我以为,语言是一种表达方式,是体现心理最直接的交流方式,
: 我希望孩子们在交流上,每天能够畅所预言,,不受任何阻碍,随着身体的长大,他们
: 接触的事物广阔,,很多词汇,中文根本不会,连我自己都不会,很多物质,现象,都
: 是我出国后,才出现的,,

c***x
发帖数: 1826
327

“尤赞认真,贵在真诚”。
这个年头“真”的人不多了,尤其在网上。这么“真”的感受,值得我也认真的回复一
个:
人无远虑必有近忧。用心的父母都是战战兢兢,辗转反侧的,至少心里如此。
推娃中文,是一个值得努力坚持的方向;也许有一天,娃会倒过来推我们英文。
想想,他们要求我们说英文,和我们要求他们说中文,是否一样很公平?
他们要求我们跟上最新最酷的东西,和我们要求他们追寻中国根中国文化,是否一样很
合理?
如果,比我们年幼几十年的孩子们,有勇气花力气去实现那些我们制定的认为有意义的
目标,我们成年人,是不是也应该学习他们的那种不断进步的精气神,像孩子那样去突
破自我?

【在 h*****m 的大作中提到】
: 多谢分享!小声说一句,我是个男地,混入妈妈队伍的爸爸。: )
: 真的很羡慕你有很多的机会带孩子回中国增强他们对中国认知和感情上的联系,尤其是
: 能够引起他们自豪感的发达城市上海。我们带他们回去的机会不多,但我也经常有意识
: 地给他们讲中国是多么的伟大,希望他们能以自己的华裔身份骄傲。比如说,为什么中
: 国叫"China",是因为古代瓷器这种高科技产品只有中国能够生产,商人要不远万里把
: 它运到欧洲,还有丝绸以及丝绸之路,以及连美洲的发现都与中国有关 - 通往东方的
: 线路受阻促使欧洲人开发海上航线。近代中国落后了乏善可讲,幸好现在中国又在赶超
: ,下次中国发射火箭的时候也可以好好讲讲。他们倒也对于中国有关的事情感兴趣,昨
: 天弟弟看到个什么东西上面有个Made in China,就赶紧跑过来给我看看。现在想来,
: 我之前说的他们说自己是"美国人“更有可能是找个借口来逃避学习。

c***x
发帖数: 1826
328

赞理智与情感的一种解,新颖有趣!

【在 t******l 的大作中提到】
: 我个人觉得智人所说的 理智 vs 感情,其实是 space-time 上的 high dimension
: pattern 的表象而已。比如其中一个 dimension 是 systematic vs sporadic,比如其
: 中另一个 dimension 是 causal vs statistical vs chaotic,等等等等。。。
: 其实我实际上想说的是:不同的智人 / 不同的AlphaGo 对相同的玩意儿的反应可能完
: 全不一样,不一定需要强求一致。

c***x
发帖数: 1826
329

现在不是流行称其为big data吗?

【在 t******l 的大作中提到】
: 当然常见例子里的更复杂的,比如对 individual 而言是个 near pure chaotic 的
: process,但对于 collective 而言却成为一个 statistically directed behavior。
: 。。比如 fish schooling、bird flocking,当然不那么傻鼻的比如 orca pod。。。
: 对于这一类的研究,俗称:社会学。。。而古人云:也就是不要 “见树不见林” 的学
: 问。。。

c***x
发帖数: 1826
330

还是小波哥的最好听。不过就这首歌而言,那个叫潮水的,唱得也很不错。

【在 t******l 的大作中提到】
: 关于这个问题,我觉得不妨先听/看一下音乐中的爵士因子,比如这首法文小曲儿的三
: 个不同的版本:
: http://www.youtube.com/watch?v=JWfsp8kwJto
: http://www.youtube.com/watch?v=hL4zY5KNVmg
: http://www.youtube.com/watch?v=xXBNlApwh0c

相关主题
娃又在推算他的质数公式。。。学校早晚要教的东西,早早学会了又如何?
大学录取率构建式数学在中国
别鸡兔同笼了,来喝啤酒吧关于刷还是不刷
进入Parenting版参与讨论
c***x
发帖数: 1826
331

sporadic-
这个与潮水曾经
介绍的大脑灰质剪枝理论,和提到的大脑neural-plasticity理论,
存在内在的不一致性。

【在 t******l 的大作中提到】
: 现在无轨电车拉回小娃的 emotional 的问题:
: 虽然 control-freak 型的推妈们,总是一根筋儿的认为一定能找到娃将来任何时刻的
: 目标函数,精确到每一个 quantum-bit。。。但上帝这位虽然搞些 alpha release 幺
: 蛾子,就急着出货的急性子马工,但还不至于是数学学渣。。。所以上帝这位马工,事
: 先就在父母和小孩的大脑里,直接都硬件代码 flash-ROM 了那些 educated sporadic-
: chaotic 的 BIOS code,这样一出生就能直接 boot-load 了。。。所以还是那句话:
: So you think you are emotional,
: But do you?
: 当然,话说回来,alpha release 的个别产品出点幺蛾子也是家常便饭,但目前上帝好
: 像不负责 BIOS 重刷的 Windows Update。。。这个估计要等刷试管的千老刷到那步,

t******l
发帖数: 10908
332
想了想应该你是对的,BIOS 应该也可以修剪,当然难度比修建人肉计算器高狠多,并
且 boot-load 以后等越久可能难度越大。当然另一方面时机更重要,古人云:Flash
BISO 赶早不如赶巧。。。


:【 在 timefall (时光崩塌) 的大作中提到: 】
h*****m
发帖数: 1034
333
振聋发聩,余音绕梁。大拇指!

【在 c***x 的大作中提到】
:
: sporadic-
: 这个与潮水曾经
: 介绍的大脑灰质剪枝理论,和提到的大脑neural-plasticity理论,
: 存在内在的不一致性。

t******l
发帖数: 10908
334
俺修改后加上个标题,把这首音乐术语堆砌诗,彻底变成理论物理讽刺嘲弄诗。。。哈
哈哈哈哈哈哈。。。
当然,这多半又要让中学语文老师想起过去物理考试拿零蛋的日子。俺的语文成绩又要
悲剧。。。(俺为啥说 “又” 涅?)。。。
话说回来,俺承认俺今天比较无厘头一点点。。。

【在 t******l 的大作中提到】
: 看完上面这些 video,就会发现,其实所谓的那些 “感性”,不过就是一些 space-
: time-pattern,用一种 educated sporadic-chaotic 的方式,比如就这么整:
: Spontaneous Symmetry Breaking
: -- by timefall, 2016.04.18
: You play major,
: You play minor;
: You sound augmented,
: You sound diminished;
: Then, ... (You gave me perfect-fifth?)
: You syncopate left,

g***3
发帖数: 471
335
抱歉啊,你是个好爸爸,爸爸对孩子的影响相比妈妈,对孩子,可能更让他们珍惜,因
为一般爸爸相比妈妈,和孩子相处时间较少。 孩子们往往会更重视,喜爱的人,讲的
话,说的故事。
孩子们由于他们的外表,他们源生族这个事实,肯定是抹不掉的。
我发现,我的孩子对自己的身份认同,有一个换漫的过程,偶然和他们说起来,老大说
,以前在家附近公立初中的时候,偶然有些小黑小阿会嘲笑他,但是儿子人源好,从小
一起朋友多,说他的人,都是后来搬过来,他们在学校根本没有影响,儿子不在意他们。
后来高中去了凡尔赛最好的私立高中,绝大部同学层次高,这种事就不存在了,更多是
羡慕他,能够经常到处旅行,他们学习中文,老大经常帮助同学。还有,他学习一般还
可以,给学校,给老师的影响,都是好学生的形象,,对中国的看法,更由于他现在自
己能够从学校学习的经济,历史,哲学等课程中思考中国,他对中国的热爱,是无法抹
去的。
说到这点,我想在目前,我们遇到的一个小事,问问大家的建议。
我家住在凡尔赛宫往巴黎去的最近别墅区。近年来,国内来凡尔赛宫旅游的旅游团成百
上千,我们这里由于是古城,,空间很紧张,,一些商人,买了沿大马路商铺,开了3
个饭店,孩子们上下学乘坐的bus,就在这条大路上停,昨天儿子回家说,,妈妈,我
真难过,他问我怎么办,,他说国人,到处都是,站在马路上,几十个,几十个,鼻子
一哼,喉咙一吼,,,就在路上就地处理,他说,他在公车上看到,很多车上的学生们
,都看到了,他说我们的形象啊,怎么办,,

【在 h*****m 的大作中提到】
: 多谢分享!小声说一句,我是个男地,混入妈妈队伍的爸爸。: )
: 真的很羡慕你有很多的机会带孩子回中国增强他们对中国认知和感情上的联系,尤其是
: 能够引起他们自豪感的发达城市上海。我们带他们回去的机会不多,但我也经常有意识
: 地给他们讲中国是多么的伟大,希望他们能以自己的华裔身份骄傲。比如说,为什么中
: 国叫"China",是因为古代瓷器这种高科技产品只有中国能够生产,商人要不远万里把
: 它运到欧洲,还有丝绸以及丝绸之路,以及连美洲的发现都与中国有关 - 通往东方的
: 线路受阻促使欧洲人开发海上航线。近代中国落后了乏善可讲,幸好现在中国又在赶超
: ,下次中国发射火箭的时候也可以好好讲讲。他们倒也对于中国有关的事情感兴趣,昨
: 天弟弟看到个什么东西上面有个Made in China,就赶紧跑过来给我看看。现在想来,
: 我之前说的他们说自己是"美国人“更有可能是找个借口来逃避学习。

g***3
发帖数: 471
336
很有意思,特别是你举得例子里,都是我喜欢的,,尤其是Yves Montand 的 Feuilles
mortes
我也会弹这首歌,,
记得以前,你也学过弹钢琴吧。那个爵士乐的改变,也改变了气氛。
乐理里,大调,小调,各种音名调,,各种风格的节奏,各种不同文化,民族长期发展
的音律,,给人的感觉,就是制造不同的气氛,不同的感受,不同的文化,不同的地理
位置,,
还有听到的人,根据自己大脑以前的记忆情感,自己先源的文化,去解释听到的音律,,

【在 t******l 的大作中提到】
: 看完上面这些 video,就会发现,其实所谓的那些 “感性”,不过就是一些 space-
: time-pattern,用一种 educated sporadic-chaotic 的方式,比如就这么整:
: Spontaneous Symmetry Breaking
: -- by timefall, 2016.04.18
: You play major,
: You play minor;
: You sound augmented,
: You sound diminished;
: Then, ... (You gave me perfect-fifth?)
: You syncopate left,

t******l
发帖数: 10908
337
我无轨电车开回来讨论这个问题哈。
我现在在想,也许大部分人 80% 的时间精力都是给浪费掉的,只是浪费的方式不同。
有 “放羊型浪费” vs “折腾型浪费” 之分。
“放羊型浪费” 比较简单,顾名思义就是放娃去玩了。
而 “折腾型浪费”,在这线程里,比如小学老师要求啥玩意儿都一丝不苟工工整整,
建立了一茬好习惯。到高中了,然后高中化学老师说不要浪费那么多时间做卡片,够用
就行。然后在做卡片上,把以前建立的追求工整好习惯忘掉,重新建立新的快糙猛做卡
片习惯。80% 的时间精力就在这小学到高中来回折腾中浪费掉了。
而中小学教育里面 80% 的瞎几把傻鼻课,也就是 “折腾型浪费” 的另一种实现方式。

【在 f**********g 的大作中提到】
: 我觉得大家实在不必对中小学老师期望太高,
: 只要想想,华人有没有人推自己娃当中小学老师的?你自己的小学中学同学中都是什么
: 样的人在当中小学老师?就知道中小学老师是个什么样的人群,他们的见识和能力水平
: 并不会随着自己进入“老师”这个职业就自动上一个档次
:
: 了。

g***3
发帖数: 471
338
你这一段,蛮难理解的。。
音乐,我一直认为是世界语言,但是,玩的人,和听的,人,
大脑深处,如果有研究,用统计的方法,可能就如你描述,
但是,这也要统计他们的背景,,例如,中国传统的5音,较西方的7音律,变调,高低
区域的宽旷,都是一听就有明显的区别的,还有印度音乐的独特节奏,南美人,非洲人
,阿拉伯人的音乐节奏,都是当地人长期生活方式下的一种反应,对他们来说,就是情
感的一种体现。
巴赫的平均律,是一种具有数学美的建造结构。更理智,是一种沉思。
巴赫的音乐,在我的生活中占着很大的一部分,,来法初期的岁月,他的音乐伴随我渡
过了很多困难的时光,所以,在常人眼里,也许是理智的重复的调调,在我来说,充满
了无数的情感,激动,平静,安宁,,
我平时,一直在弹巴赫的2部,3部创意曲,平均律,一生都弹不完,孩子们会很容易就
辨别出巴赫风格的乐曲,好像就是我家的调调,,可能,这也帮助他们建筑了他们脑系
统的基点。
就如,我小时候,我妈妈一直唱越剧,,我从就在台前,台后,被那些公子佳人的喜乐
悲欢故事,而感动,红楼梦,孟丽君,白蛇传,,,我感性的性格先入为主一样,,,
后来长大,遇事,挫折,必须完善理智,,,
太有意思了,

veli
partially-

【在 t******l 的大作中提到】
: 虽然这是一首破烂尿不湿,但说明的道理是:尽管不会数学证明的数学学渣们,觉得
: “理智” 这种 veli veli directive 的玩意儿是多么的高大上。但事实就是 “veli
: veli directive” 这枚硬币的另一面就是 “veli veli biasd” -- 在这个问题上就
: 是你必须知道确切的(known)目标函数,否则整个 optimization system
: pathetically fail。。。
: 而真实世界的情况就是,波切利这枚小破 super-string 在台上唱歌的时候,这哥们哪
: 里知道台下和远方的 N 多听众一个一个小破 super-string 的确切 formula。退一步
: 说,就算知道,那哥们比阿发狗差一百万被的猴脑算得过来用啥频率让大部分小破
: super-string 发生共振么?所以这哥们唯一的办法就是有限度地放弃 “理智”,也就
: 是 directive approach。而代之以 educated sporadic-chaotic,来对付 partially-

g***3
发帖数: 471
339
问题提得好,
我建议上面那位爸爸,还是需要在自己孩子还没觉察前,化功夫抓一下,自己的英文。
如果有一天,孩子觉察到自己父母的居住国语言不行的时候,他们也许会失落,会难过,
我有一些周围的例子,孩子们到十几岁,就不和父母说话了,无意识里,他们觉得父母
居住国语言不好,就不了解居住国文化,人文,可能会看不起自己的父母,可能会不再
说父母源生国的语言,有的更对抗父母,,,
谨慎,,
我的原则,,孩子说什么话,我都接受,,
只要他们一回家,就问妈妈在哪里,就和我们滔滔不绝,我们一起做饭,娱乐,看电影
,逛街,评论衣着,

【在 c***x 的大作中提到】
:
: sporadic-
: 这个与潮水曾经
: 介绍的大脑灰质剪枝理论,和提到的大脑neural-plasticity理论,
: 存在内在的不一致性。

t******l
发帖数: 10908
340
或者我用一个更生动的比方:去 HomeDepot 买一把洗碗刷子。
“放羊型浪费“ 的搞法是,去 HomeDepot 跟波涛汹涌的卖工具美眉调情了一下午,最
后才忽然想起来原来是要买把洗碗刷子,于是买了洗碗刷子回家,这时已经半天被浪费
掉了。。。
而 “折腾型浪费” 的搞法是,去 HomeDepot 看见一把刷子就买回家,回到家刷碗刷
了半天才发现买的是一把马桶刷子,于是开车到远方的另一家 HomeDepot 把马桶刷子
退成 store credit,然后拿 store credit 跑第三家 HomeDepot 买了一把洗碗刷子,
回到家时已经天黑了。。。当然比起 “放羊型浪费” 而言,“折腾型浪费” 的优点
是让人感觉这一下午特充实。。。
当然,其实还有一种浪费,叫 “傻叉型浪费”,其搞法是:跑到 HomeDepot 一看,娘
的,小学语文老师只教了 fiction 虚词,HomeDepot 里的 non-fiction 实词大字一个
都不认知。。。于是在 HomeDepot 货架之间随机乱窜一下午,终于撞到一把洗碗刷子
。而拿出去 checkout 的时候已经是床前明月光了。。。当然,体育老师的看法是这下
午一点儿也不浪费,track & field 课可以免修了。。。比如达特茅斯地下室学 Java
技校速成班的搞法。。。

式。

【在 t******l 的大作中提到】
: 我无轨电车开回来讨论这个问题哈。
: 我现在在想,也许大部分人 80% 的时间精力都是给浪费掉的,只是浪费的方式不同。
: 有 “放羊型浪费” vs “折腾型浪费” 之分。
: “放羊型浪费” 比较简单,顾名思义就是放娃去玩了。
: 而 “折腾型浪费”,在这线程里,比如小学老师要求啥玩意儿都一丝不苟工工整整,
: 建立了一茬好习惯。到高中了,然后高中化学老师说不要浪费那么多时间做卡片,够用
: 就行。然后在做卡片上,把以前建立的追求工整好习惯忘掉,重新建立新的快糙猛做卡
: 片习惯。80% 的时间精力就在这小学到高中来回折腾中浪费掉了。
: 而中小学教育里面 80% 的瞎几把傻鼻课,也就是 “折腾型浪费” 的另一种实现方式。

相关主题
关于刷还是不刷求科普: 奥数 vs. Math Olympiad vs. Math Count vs. Math circle vs. 超前学数学
是不是任我儿子继续迷象棋有明天考AMC 8的吗?
9岁, 如何报名考amc10算不算有数学天分
进入Parenting版参与讨论
t******l
发帖数: 10908
341
:我建议上面那位爸爸,还是需要在自己孩子还没觉察前,化功夫抓一下,自己的英文。
对这个我有不同的看法,我觉得父母没有必要在自己的娃前伪装自己。。。父母作为
non-native-speaker,尽力做到 effective communication 就足够了。。。我就用洋
泾浜英文教我娃数学,因为这样跟学校术语口风一致,对娃而言简单方便。。。而我娃
从来不会在意我的英文洋泾浜的程度,只要不造成误解。。。说实话,我还真没见过有
娃推父母 AP English 考高分追藤的,反过来倒是很常见就是了 // run。。。

过,

【在 g***3 的大作中提到】
: 问题提得好,
: 我建议上面那位爸爸,还是需要在自己孩子还没觉察前,化功夫抓一下,自己的英文。
: 如果有一天,孩子觉察到自己父母的居住国语言不行的时候,他们也许会失落,会难过,
: 我有一些周围的例子,孩子们到十几岁,就不和父母说话了,无意识里,他们觉得父母
: 居住国语言不好,就不了解居住国文化,人文,可能会看不起自己的父母,可能会不再
: 说父母源生国的语言,有的更对抗父母,,,
: 谨慎,,
: 我的原则,,孩子说什么话,我都接受,,
: 只要他们一回家,就问妈妈在哪里,就和我们滔滔不绝,我们一起做饭,娱乐,看电影
: ,逛街,评论衣着,

t******l
发帖数: 10908
342
我写这段是源于,很多时候我看到/听说武器控制员们推得鸡飞狗跳,但说到底大部分
非天才娃最终还是 80% 的时间是浪费掉,差别也就是从 A 型浪费推成 B 型浪费。。
。我觉得像我这样直接躺床上睡觉浪费掉,也不见得就不够高尚。。。当然追求不同的
浪费型式,可能也是武器控制员们人生乐趣的一种。。。但对俺而言,都是花了一下午
买了一把洗碗刷子,差别不大。。。所以我干脆不管。。。

【在 t******l 的大作中提到】
: 或者我用一个更生动的比方:去 HomeDepot 买一把洗碗刷子。
: “放羊型浪费“ 的搞法是,去 HomeDepot 跟波涛汹涌的卖工具美眉调情了一下午,最
: 后才忽然想起来原来是要买把洗碗刷子,于是买了洗碗刷子回家,这时已经半天被浪费
: 掉了。。。
: 而 “折腾型浪费” 的搞法是,去 HomeDepot 看见一把刷子就买回家,回到家刷碗刷
: 了半天才发现买的是一把马桶刷子,于是开车到远方的另一家 HomeDepot 把马桶刷子
: 退成 store credit,然后拿 store credit 跑第三家 HomeDepot 买了一把洗碗刷子,
: 回到家时已经天黑了。。。当然比起 “放羊型浪费” 而言,“折腾型浪费” 的优点
: 是让人感觉这一下午特充实。。。
: 当然,其实还有一种浪费,叫 “傻叉型浪费”,其搞法是:跑到 HomeDepot 一看,娘

g***3
发帖数: 471
343
我没有深展论点,口音,不是问题,孩子大了,会理解,重要的是让孩子明白,父母关
注当地的新闻,社会日常大事,这些需要使用当地语言。。。

文。

【在 t******l 的大作中提到】
: :我建议上面那位爸爸,还是需要在自己孩子还没觉察前,化功夫抓一下,自己的英文。
: 对这个我有不同的看法,我觉得父母没有必要在自己的娃前伪装自己。。。父母作为
: non-native-speaker,尽力做到 effective communication 就足够了。。。我就用洋
: 泾浜英文教我娃数学,因为这样跟学校术语口风一致,对娃而言简单方便。。。而我娃
: 从来不会在意我的英文洋泾浜的程度,只要不造成误解。。。说实话,我还真没见过有
: 娃推父母 AP English 考高分追藤的,反过来倒是很常见就是了 // run。。。
:
: 过,

i**e
发帖数: 19242
344
能再说一下是怎么回事吗?
我越想越觉得搬家好不靠谱的建议 汗
也不过几年 想法这么大的差别呢

【在 g***3 的大作中提到】
: 没有,当时,搬家这个,我们还真考虑了,
: 只是,我们住的地方,已经是一个相对,理想的人文教育环境最佳的地方,还有,我们
: 特别造了自己的房子,,,
: 这件事,还是坚持了4,5年的持久战,孩子自己一点点意识到对人的认识,负量感情的
: 危险,,
: 这件事,他一生会记得,,

i**e
发帖数: 19242
345
给100个点赞!

【在 g***3 的大作中提到】
: 有关学习中文,,
: 他们出生时,我就请了一位南方大学,普通话说得非常优雅的老师,来我家帮助我,照
: 顾2个孩子,
: 他们会说的第一个词,是蘑菇,大象等等,,我老公以前在国内工作时,会一点中文,
: 后来又在法国上夜校,但是坚持不了,他鼓励我们说中文。2个孩子上幼儿园后,开始
: 法语说得多,我没有象很多书上写的,双语孩子,妈妈要坚持说一种语言,爸爸要坚持
: 说另一种,我以为,语言是一种表达方式,是体现心理最直接的交流方式,
: 我希望孩子们在交流上,每天能够畅所预言,,不受任何阻碍,随着身体的长大,他们
: 接触的事物广阔,,很多词汇,中文根本不会,连我自己都不会,很多物质,现象,都
: 是我出国后,才出现的,,

i**e
发帖数: 19242
346
我对祖国有归属感,米国住再久也没有这个感觉
顺推 生长在米国的孩子的归属感认同感在米国
我很幸运 跟孩子能交心情感上非常近
孩子爱屋及乌,中国是妈妈的母国,中国对她来说就具有非常非常的特殊的意义
中华血脉是孩子identity的一部分 很自然而然的接受
很幸运 这方面没有太多struggle

【在 g***3 的大作中提到】
: 人是感情动物,一个地方住久了,接触当地的事物达到熟同自己的程度时,肯定会把当
: 地的事物无意识转移认同到自己身上,如果你对孩子否定他的认同,就如同你否认他的
: 经历和感受存在,孩子们会以自己有限的力量,来维护自己的感受,他们就会出现对抗
: 和厌恶的表现。
: 不知你们孩子和中国接触时间的频率多少,,这个奠定了他们对中国的认识多少,
: 他们自己的认识,不是家长强加于他们,要他们知道的认知,
: 孩子自己的认知,和家长的认知,是2回事,,孩子自己的认知,是通过他们自己接触
: 人,接触事物,吃饭,玩耍,学习,听周围人的事,感受周围人的情感,,,感受周围
: 人对他们关注的程度,,,
: 一方面,你们应该高兴,如果孩子认为自己是美国人,说明他们认为美国好,也说明爸

i**e
发帖数: 19242
347
看上去 婆婆并没对你的生活造成很大的影响嘛

【在 g***3 的大作中提到】
: 你说到要害处了,所以啊所以,我很希望,他们能够在决定自己终身大事前,多观察人
: ,多有一点接触人,待人的经验,多和将来可能的那个家庭未来成员接触,,,
: 我自己的痛,就是对婆家婚前不了解,尤其婆婆,孩子们知道的,,,

i**e
发帖数: 19242
348
回国,越小越好,语言发展的黄金期

【在 h*****m 的大作中提到】
: 说来惭愧,我督促他们说中文,学中文的一大动力就是感觉自己的英文不够好,现在简
: 单的日常话题还能应付,但怕他们长大以后与他们的交流受到阻碍。所以从小开始,就
: 一直坚持在家里必须说中文,他们不会说的中文词汇用英文说出来,我翻译给他们,再
: 让他们重复说一次。平时给他们讲故事时遇到估计他们不懂得词儿,也停下来解释一下
: 。也让他们看中文动画片,在网上听中文故事。至少现在还在坚持,希望能坚持到象你
: 家孩子那样认识到学中文重要性的年龄,我就放心了。
: 还是有效果的,昨天老二叽叽喳喳跟我讲他的一个同学是如何淘气的时候,很顺畅地说
: “他竟敢在老师耳朵边上大喊“。我都不记得什么时候给他讲过“竟敢”这个词儿了,
: 也可能是从那个动画片里学的吧。
: 等他们大些了,也准备每年在暑假把他们放回国内待些日子,据说对提高中文水平大有

i**e
发帖数: 19242
349
我会问孩子,你认为他们为什么会这么做?他们有没有意识到这么做不妥当?你觉着你想
怎么解决这个问题?根子在哪儿?治表治里有啥不同?
可以展开谈的太多了

们。

【在 g***3 的大作中提到】
: 抱歉啊,你是个好爸爸,爸爸对孩子的影响相比妈妈,对孩子,可能更让他们珍惜,因
: 为一般爸爸相比妈妈,和孩子相处时间较少。 孩子们往往会更重视,喜爱的人,讲的
: 话,说的故事。
: 孩子们由于他们的外表,他们源生族这个事实,肯定是抹不掉的。
: 我发现,我的孩子对自己的身份认同,有一个换漫的过程,偶然和他们说起来,老大说
: ,以前在家附近公立初中的时候,偶然有些小黑小阿会嘲笑他,但是儿子人源好,从小
: 一起朋友多,说他的人,都是后来搬过来,他们在学校根本没有影响,儿子不在意他们。
: 后来高中去了凡尔赛最好的私立高中,绝大部同学层次高,这种事就不存在了,更多是
: 羡慕他,能够经常到处旅行,他们学习中文,老大经常帮助同学。还有,他学习一般还
: 可以,给学校,给老师的影响,都是好学生的形象,,对中国的看法,更由于他现在自

i**e
发帖数: 19242
350
我经常跟孩子一起笑我的语法错误 呵呵
整个外婆语录应该挺有意思的
其实 口语不需要那么好的
有思想有智慧善于观察
用简单的词语完全可以言简意赅的

过,

【在 g***3 的大作中提到】
: 问题提得好,
: 我建议上面那位爸爸,还是需要在自己孩子还没觉察前,化功夫抓一下,自己的英文。
: 如果有一天,孩子觉察到自己父母的居住国语言不行的时候,他们也许会失落,会难过,
: 我有一些周围的例子,孩子们到十几岁,就不和父母说话了,无意识里,他们觉得父母
: 居住国语言不好,就不了解居住国文化,人文,可能会看不起自己的父母,可能会不再
: 说父母源生国的语言,有的更对抗父母,,,
: 谨慎,,
: 我的原则,,孩子说什么话,我都接受,,
: 只要他们一回家,就问妈妈在哪里,就和我们滔滔不绝,我们一起做饭,娱乐,看电影
: ,逛街,评论衣着,

相关主题
大家怎么推孩子的弱项?二年级女儿的report
少年智力开发觉得美帝数学进度慢的进来看一下
[转载] 为什么说usamo 简单分数应用题
进入Parenting版参与讨论
g***3
发帖数: 471
351
看来,我们好像啊,,

【在 i**e 的大作中提到】
: 我经常跟孩子一起笑我的语法错误 呵呵
: 整个外婆语录应该挺有意思的
: 其实 口语不需要那么好的
: 有思想有智慧善于观察
: 用简单的词语完全可以言简意赅的
:
: 过,

g***3
发帖数: 471
352
婆婆的冷漠,对我的孩子们来说,是一种无形的冷暴力。
我们居住的地方,大家庭概念强,孩子的同学朋友们,都有祖父母捧,法国的假期又多
,每7个学周,就有2个周假期,不可能老往中国跑,孩子们很多同学朋友们,都可以去
海边,或山上爷爷奶奶,外祖父母家度假,,可以有表兄弟姐妹玩,学校外学到的亲情
日常生活知识丰富。
哎,我婆家很特别的,婆婆是东部大人家的长女,自己的父亲是法国工业家,法国巴黎
有名的综合理工大学的高才生,,有祖传的产业,一百多年,然后在2战后,发展扩大
,,
婆婆看人高瞻远视,她生了6个孩子,老公是老4,我认识老公时,并不知道他们家族的
情况,,老公一直瞒着我,我那时年轻,在法国身边又没有父母姐姐,那时和国内电话
也很贵的,我们认识了很多年,
分分合和,,等到老公坚决和我结婚的时候,婆婆才不得不接受,,
婆婆是那种和人保持远距离的人,,从观察老公的性格,和他兄弟姐妹来看,,
婆婆是一个没有情感的人。我怀疑她是否抱过她的每一个孩子,,她生了这么多孩子,
完全是他们的传统,最后一个孩子,是生出来就去世的那种,那是在60年代末的法国,
,,
这些年,我们一直在分析我们自己,,我老公是一个纯理智,以前很少能够表达自己的
人。
我们的生活,完全靠我们自己的双手,近2年,婆婆去世,才发现,这些年,她的财产
被那些银行家,中间人,玩笼得完全没了,,老公其他兄弟姐妹,都是性格非常弱的,
,我常常思考这样的例子,,

【在 i**e 的大作中提到】
: 看上去 婆婆并没对你的生活造成很大的影响嘛
g***3
发帖数: 471
353
这个故事很长,,
老大4岁,在幼儿园中班的时候,遇到了一个从巴黎市区搬过来的男孩B,,他们渐渐成
了非常好的朋友,B的妈妈,非常热情,常常请我们,我们后来成了比较投机的朋友。
她是电讯局一个部门不大不小的经理,工作很忙,,我是自由职业,法国公立学校,老
师罢工啥的,我常帮助照看孩子们,,,男孩是老2,上面有一个大2岁的哥哥。有天生
坏血病,需要每2周,到医院换血,,他们的爸爸也是电脑工程师,很喜欢游戏,爸爸
很胖,他18岁,母亲就去世,他对2个男孩的宠爱,比如让他们吃糖,没有约束,他们
可以玩游戏一直到深夜,,,
我家孩子第一个gameboy,就是在他们的影响下,犹豫了好久,才给孩子玩的,,
开始的时候,我们还一起渡过假,,B的妈妈,自己的父母,是巴黎开咖啡店的,有好
几个,她是独生女,从小被父母送到乡下祖父母那里,到8岁,才回到父母身边,然后
父母要她继承咖啡店,但是她不喜欢,自己读书,后来成了工程师,,,和父母关系一
直很差。
孩子们渐渐长着,她常说,孩子要轮其自然,自己的父母不懂读书,她说她自己成了工
程师,,,
问题是在,老3出生后,发生的,那时老大9岁多了,我们发现,他一直想和那个男孩在
一起,,甚至晚上,周末也要在一起。这个完全影响了我们的日常生活,有时家里有客
人,那个男孩会来电话,
我开始注意老大在学校的情况,,老大一直是个朋友很多,整天开心的孩子,那阵子,
他老说那个哪个不好,同学们都是笨蛋,他们学校演戏剧,本来老大和另2个男孩老早
就准备好的,,但是演戏的那个星期2,老大突然跟我说,他不想演自己的戏了,他要
到B那组,,,

【在 i**e 的大作中提到】
: 能再说一下是怎么回事吗?
: 我越想越觉得搬家好不靠谱的建议 汗
: 也不过几年 想法这么大的差别呢

g***3
发帖数: 471
354
然后,我就开始天天推着婴儿车,上老大学校暗地侦探了,,发现,在课堂休息时,大
部分孩子在操场一头玩,老大和B,还有另一个男孩,在一个角落里,有一次,老大的
女同学看到我,很喜欢老3,我和她们聊天,才知道,那个B,找不到和他一组的孩子演
,,,所以他要求老大,放弃别人,
就是在那个阶段,我在本坛发帖子,求建议的。。
老大还邀请那个B 和另个男孩来家玩,,另个男孩是个非洲混血,我不喜欢他的举止,
,他的生日,老大一定要去,他们住在离我们比较远的地方,,那天,我们坐电梯到了
后,他们开一条缝,让儿子进去,妈妈也不出来见一下,我觉得太奇怪了,,,
这样的生活维持了很久,期间,我们和老大谈,设法了解其原因,,妹妹也在学校看,
她有时会说哥哥的事,
比如有一次,他们学校组织到巴黎科学馆,在选择走路拉手的朋友对时,一位也是从小
和老大一起的男孩,要拉老大的手,那个B 就对老大说,别人以后都会抛弃他,但是他
永远不会,这个话给妹妹听见了,妹妹说哥哥不好拒绝,,,
g***3
发帖数: 471
355
B完全操纵了老大,,令我担心的事,老大一边为难,不能和其他孩子一起玩,一边又
不忍心让B失望,,我还找老师希望把他们分开,,这个老师还说我多管,,,
老大由于这个,还不想去凡尔赛的私立初中,,他以后一直后悔,,
上初中的时候,那个男孩,还想和老大在一起,,,
幸好,我找了初中校长,谈心,,她帮忙没有把2男孩分在一起,,,
后来,儿子又重新认识了新的伙伴,,,初3时,考进了欧洲班,,学习上渐渐与B拉开
了距离,,
高中后,更是到了2个皆然不同的高中,,有时,偶然车上遇到,老大说,他现在在读
技术高中,,又听说,他爸受不了工作上的压力,辞职了,
后来,我在凡尔赛宫旁我的一个出租房附近,看到了B 的爸爸,在卖冰激凌,,没有客
,我很难过,,我觉得他可怜,每次走过那里,我会很该尬,,
g***3
发帖数: 471
356
人啊人,命运,轨迹,和父母的,,一整连,上3代,下3代,我相信代代相传,,心理
,,
B的父母,一直没有结婚。我想,都是2个自己没有安全感的人。
很多年后,老大跟我们说,B 曾经跟他说,妈妈生了小弟弟了,就不要他了。。
t******l
发帖数: 10908
357
我后来想了一下,我觉得做卡片的这点时间本身不应该是问题。
因为我觉得大部分普通娃都是 20% 时间干掉 80% 的事儿,然后 80% 的时间瞎玩浪费
掉。
如果拿初中阶段的 AMC 8 / AMC 10 做例子的话,其实普通娃一天做 10 道适合难度的
题也差不多了。做更多的话,普通娃一般也记不住。做后面太难的题目的话,普通娃更
记不住。
这样 10 道适合难度的 AMC 计算选择题,再拖拉,咋也拖拉不过一小时。这一大整天
的,咋也不会缺这一小时不是?
我觉得深层次的问题可能是在 “上心 vs 不上心”。而具体可能表现成 80% 的 “瞎
玩” 到底是 “普通型瞎玩” 还是 “逃避型瞎玩” 的差别。

【在 f**********g 的大作中提到】
: 你们怎么对卡片的反应都这么负面?
: 我看到了只觉得新奇和欣赏,这么一个做事情细心又认真的女孩子,以后不管做文字还
: 是管理工作都很不错。而且高中枯燥的学习过程都能添加一些花样,那么以后自己的生
: 活也会过的很有情趣
: 至于因此浪费了时间,稍微引导一下,让她学会统筹安排就好了,实在没必要打击扼杀

t******l
发帖数: 10908
358
我觉得这本质上可能是不同教育思路的差别。
美帝的中小学特别强调 spiral 螺旋式上升。但 “螺旋式上升” 这枚硬币的另一面,
也就是装了又拆、拆了再装的 “折腾式教育”。
或者换个比方,学校 spiral 教育思想,好比沿着盘山公路一个一个的拔地堡。看似容
易,但其实也很慢很折腾也多流血。
但如果军事长官的计划是呼唤空军把山头炸个稀巴烂以后,用直升机直接把 101st air
lift 到山顶,架榴弹炮/轻重机枪/狙击枪/手榴弹/空地协同激光引导器/等等等等五
毒俱全,然后从制高点往下远距离精确活力定点清除公路两旁的碉堡工事,M1A1 坦克师
确认后立马从山下公路以 55 mph 冲入撕开的缺口后,直插后方切断敌人补给,顺手把
敌军指挥所直接一窝端了。。。但军事演习的裁判员同学就死看规则,说你丫盘山公路
上的地堡一个都没拔掉,我只能给你零蛋。那其实也不能算不靠谱,只是死板一点。
不过话说回来,中小学的一战型保守步兵堑壕战打法,也不是没有其考虑的。因为 “
大手笔” 这枚硬币的另一面也就是 “大傻鼻”。所以在巴黎跪舔曼斯坦因以前,马奇
诺防线按照其一战用血积累的经验,担心曼斯坦因这嘴上没毛/乳臭未干的臭小子的出
格想法也太不靠谱了,那也算是人之常情其实。。。

system
Roll

【在 c***x 的大作中提到】
:
: sporadic-
: 这个与潮水曾经
: 介绍的大脑灰质剪枝理论,和提到的大脑neural-plasticity理论,
: 存在内在的不一致性。

g***3
发帖数: 471
359
一句到位,女儿说到底,还是没有紧迫感。

【在 t******l 的大作中提到】
: 我后来想了一下,我觉得做卡片的这点时间本身不应该是问题。
: 因为我觉得大部分普通娃都是 20% 时间干掉 80% 的事儿,然后 80% 的时间瞎玩浪费
: 掉。
: 如果拿初中阶段的 AMC 8 / AMC 10 做例子的话,其实普通娃一天做 10 道适合难度的
: 题也差不多了。做更多的话,普通娃一般也记不住。做后面太难的题目的话,普通娃更
: 记不住。
: 这样 10 道适合难度的 AMC 计算选择题,再拖拉,咋也拖拉不过一小时。这一大整天
: 的,咋也不会缺这一小时不是?
: 我觉得深层次的问题可能是在 “上心 vs 不上心”。而具体可能表现成 80% 的 “瞎
: 玩” 到底是 “普通型瞎玩” 还是 “逃避型瞎玩” 的差别。

d**********h
发帖数: 2795
360
潮水兄在现实中肯定是“抉择困难”症患者
做不得司令,最多参谋长

【在 t******l 的大作中提到】
: 我后来想了一下,我觉得做卡片的这点时间本身不应该是问题。
: 因为我觉得大部分普通娃都是 20% 时间干掉 80% 的事儿,然后 80% 的时间瞎玩浪费
: 掉。
: 如果拿初中阶段的 AMC 8 / AMC 10 做例子的话,其实普通娃一天做 10 道适合难度的
: 题也差不多了。做更多的话,普通娃一般也记不住。做后面太难的题目的话,普通娃更
: 记不住。
: 这样 10 道适合难度的 AMC 计算选择题,再拖拉,咋也拖拉不过一小时。这一大整天
: 的,咋也不会缺这一小时不是?
: 我觉得深层次的问题可能是在 “上心 vs 不上心”。而具体可能表现成 80% 的 “瞎
: 玩” 到底是 “普通型瞎玩” 还是 “逃避型瞎玩” 的差别。

相关主题
我招,我是猪大学录取率
请教:怎样说服孩儿他爸多关心孩子的学习?别鸡兔同笼了,来喝啤酒吧
娃又在推算他的质数公式。。。学校早晚要教的东西,早早学会了又如何?
进入Parenting版参与讨论
C********e
发帖数: 2327
361
法国真是这样?还流传下几代?

【在 g***3 的大作中提到】
: 美国读大学,对于我们来说,是太贵了。
: 法国是一个很追求文凭的社会,尤其是第一流的精英学校,有一个好文凭,是一辈子的
: 事,甚至是流传到下面几代人的事,,

i**e
发帖数: 19242
362
你这算是嫁入豪门了?:) 呵呵 至少半个
我看当时即使你对婆婆有一定的了解 你还是会嫁的:)
懂了
性情如此 不必太纠结了
女性一般生而有nurture的能力,正态分布,总是有人落在边界上的
你们夫妻挺互补的
孩子们传承了父亲的智商又传承了你的情商和为人处事

【在 g***3 的大作中提到】
: 婆婆的冷漠,对我的孩子们来说,是一种无形的冷暴力。
: 我们居住的地方,大家庭概念强,孩子的同学朋友们,都有祖父母捧,法国的假期又多
: ,每7个学周,就有2个周假期,不可能老往中国跑,孩子们很多同学朋友们,都可以去
: 海边,或山上爷爷奶奶,外祖父母家度假,,可以有表兄弟姐妹玩,学校外学到的亲情
: 日常生活知识丰富。
: 哎,我婆家很特别的,婆婆是东部大人家的长女,自己的父亲是法国工业家,法国巴黎
: 有名的综合理工大学的高才生,,有祖传的产业,一百多年,然后在2战后,发展扩大
: ,,
: 婆婆看人高瞻远视,她生了6个孩子,老公是老4,我认识老公时,并不知道他们家族的
: 情况,,老公一直瞒着我,我那时年轻,在法国身边又没有父母姐姐,那时和国内电话

g***3
发帖数: 471
363
比喻生动。

潮水兄在现实中肯定是“抉择困难”症患者

【在 d**********h 的大作中提到】
: 潮水兄在现实中肯定是“抉择困难”症患者
: 做不得司令,最多参谋长

C********e
发帖数: 2327
364
同意这个观点

【在 f**********g 的大作中提到】
: 那只能说你们高中的考试水平不够难
: 另外,你确定那个女孩不是看上了你儿子?/run

g***3
发帖数: 471
365
法国到了高层,没有那几个有名传统学校的文凭,还真的很难做到最高的位置。
如果是那几个学校出来的学生,子孙都会觉得光荣,

【在 C********e 的大作中提到】
: 法国真是这样?还流传下几代?
t******l
发帖数: 10908
366
现实中的曼斯坦因,多半更担心突防深入的 AMC 10 直升机群的冲击进度。。。因为这
决定下一步是继续直升机群蛙跳式前进呢;还是直升机群不够,要改成坦克群地面冲击
;还是说 AMC12 的 V22 鱼鹰群到货了不用蛙跳了,直接从平顶船一步到位 air lift
到柏林市郊。。。所以的确有一定程度的 “抉择困难症”。。。
而现实中的马奇诺防线,显然更担心是不是能进 Honor Roll 三八红旗手。。。这倒也
确实没有 “抉择困难症”,因为学校把抉择的活了也给包了,不用抉择了。

【在 d**********h 的大作中提到】
: 潮水兄在现实中肯定是“抉择困难”症患者
: 做不得司令,最多参谋长

g***3
发帖数: 471
367
他们家,到了婆婆兄弟姐妹这一代,工厂都给他们的兄弟荒光了,还一钱不值卖给了美
国基金,
婆婆这一支孩子最不争气,,她的其他兄弟,因为娶了其他女子,我看子女都发展得比
老公的兄妹好。

【在 i**e 的大作中提到】
: 你这算是嫁入豪门了?:) 呵呵 至少半个
: 我看当时即使你对婆婆有一定的了解 你还是会嫁的:)
: 懂了
: 性情如此 不必太纠结了
: 女性一般生而有nurture的能力,正态分布,总是有人落在边界上的
: 你们夫妻挺互补的
: 孩子们传承了父亲的智商又传承了你的情商和为人处事

g***3
发帖数: 471
368
我当时还以为老公是孤儿,真的,他从来不提起他的家庭,,

【在 i**e 的大作中提到】
: 你这算是嫁入豪门了?:) 呵呵 至少半个
: 我看当时即使你对婆婆有一定的了解 你还是会嫁的:)
: 懂了
: 性情如此 不必太纠结了
: 女性一般生而有nurture的能力,正态分布,总是有人落在边界上的
: 你们夫妻挺互补的
: 孩子们传承了父亲的智商又传承了你的情商和为人处事

i**e
发帖数: 19242
369
脸红ing
7,8年前的事,搬家不是靠谱的做法,呵呵
当时我孩子太小我太没经验了
现在问同样的问题,答案是你做法,也比较知道怎么去跟孩子谈这种问题了
好戏剧啊
比电视剧好看:)
我小孩3年级的时候也遇到过一个很厉害的小朋友,呵呵
情商高特别善于manipulate peers
后来转走了 成了没有结果的结局:)

【在 g***3 的大作中提到】
: B完全操纵了老大,,令我担心的事,老大一边为难,不能和其他孩子一起玩,一边又
: 不忍心让B失望,,我还找老师希望把他们分开,,这个老师还说我多管,,,
: 老大由于这个,还不想去凡尔赛的私立初中,,他以后一直后悔,,
: 上初中的时候,那个男孩,还想和老大在一起,,,
: 幸好,我找了初中校长,谈心,,她帮忙没有把2男孩分在一起,,,
: 后来,儿子又重新认识了新的伙伴,,,初3时,考进了欧洲班,,学习上渐渐与B拉开
: 了距离,,
: 高中后,更是到了2个皆然不同的高中,,有时,偶然车上遇到,老大说,他现在在读
: 技术高中,,又听说,他爸受不了工作上的压力,辞职了,
: 后来,我在凡尔赛宫旁我的一个出租房附近,看到了B 的爸爸,在卖冰激凌,,没有客

i**e
发帖数: 19242
370
娘牛牛一窝的节奏?:)

【在 g***3 的大作中提到】
: 他们家,到了婆婆兄弟姐妹这一代,工厂都给他们的兄弟荒光了,还一钱不值卖给了美
: 国基金,
: 婆婆这一支孩子最不争气,,她的其他兄弟,因为娶了其他女子,我看子女都发展得比
: 老公的兄妹好。

相关主题
学校早晚要教的东西,早早学会了又如何?是不是任我儿子继续迷象棋
构建式数学在中国9岁, 如何报名考amc10
关于刷还是不刷求科普: 奥数 vs. Math Olympiad vs. Math Count vs. Math circle vs. 超前学数学
进入Parenting版参与讨论
d**********h
发帖数: 2795
371
有的人擅长打“呆仗”,而有的人则擅长“浪仗”
司令官只要以己之不可胜待敌之可胜就可以了,临阵了,把paper里最前沿最fancy的战
法拿出来耍宝,也不管儿郎们的实力,后勤保障力度,这基本就是“作死”
好在潮水兄家里有太座把关,否则。。。

lift

【在 t******l 的大作中提到】
: 现实中的曼斯坦因,多半更担心突防深入的 AMC 10 直升机群的冲击进度。。。因为这
: 决定下一步是继续直升机群蛙跳式前进呢;还是直升机群不够,要改成坦克群地面冲击
: ;还是说 AMC12 的 V22 鱼鹰群到货了不用蛙跳了,直接从平顶船一步到位 air lift
: 到柏林市郊。。。所以的确有一定程度的 “抉择困难症”。。。
: 而现实中的马奇诺防线,显然更担心是不是能进 Honor Roll 三八红旗手。。。这倒也
: 确实没有 “抉择困难症”,因为学校把抉择的活了也给包了,不用抉择了。

i**e
发帖数: 19242
372
时塌的文,你都看懂了?
写个总结文科普一下吧:)

有的人擅长打“呆仗”,而有的人则擅长“浪仗”

【在 d**********h 的大作中提到】
: 有的人擅长打“呆仗”,而有的人则擅长“浪仗”
: 司令官只要以己之不可胜待敌之可胜就可以了,临阵了,把paper里最前沿最fancy的战
: 法拿出来耍宝,也不管儿郎们的实力,后勤保障力度,这基本就是“作死”
: 好在潮水兄家里有太座把关,否则。。。
:
: lift

t******l
发帖数: 10908
373
不跟你争论这个,这玩意儿有 National Head Counts 整套信息数据支持。当然本版的
主流是鸡汤,你螳臂就不用挡车了。
另外这种动态策略也像阿发狗围棋一样,没有 “神之一手”。如果你是想火力侦查是
不是有 “神之一手” 的话,那看看阿发狗就不用浪费带宽了。

击;

【在 d**********h 的大作中提到】
: 有的人擅长打“呆仗”,而有的人则擅长“浪仗”
: 司令官只要以己之不可胜待敌之可胜就可以了,临阵了,把paper里最前沿最fancy的战
: 法拿出来耍宝,也不管儿郎们的实力,后勤保障力度,这基本就是“作死”
: 好在潮水兄家里有太座把关,否则。。。
:
: lift

g***3
发帖数: 471
374
妹妹的孩子,是女孩,还是男孩 ?
这几天,这个帖子的引深,中文吃饭时,和老大讨论,
他说,作为亚洲外表的男孩,他觉得是挺难的,因为大多数人,看到他们,会给他们贴
标签,要么功夫,要么读书好,他不希望别人给他贴标签,还有,我们在这里没有大家
庭,婆家没有频繁来往,这个他有时是比较痛苦的,,还有,昨天,他带弟弟坐车,去
凡尔赛宫花园和同学朋友夜餐,他说,弟弟啊,完全站在法国人的视野上,看到那里很
多中国旅游客,就象法国人那样说,中国人真多,,他说,要好好教育弟弟,别忘了自
己是半个中国人,,弟弟马上回击,说,自己说不来中文,老大说,妈妈失职,弟弟说
,他就是说事实,他看到的,,老大又为中国旅游客的形象抓急,,
女儿不常为这些问题困绕。也许女孩比较容易被所居住国人接受,,男孩,可能给人有
威胁的感觉,
老大一直在练身体,高高大大的,体育很棒,他多么希望中国强大,来法国的中国旅客
礼貌,文明。
他的高中,就在凡尔赛宫旁边的路上。

【在 i**e 的大作中提到】
: 我对祖国有归属感,米国住再久也没有这个感觉
: 顺推 生长在米国的孩子的归属感认同感在米国
: 我很幸运 跟孩子能交心情感上非常近
: 孩子爱屋及乌,中国是妈妈的母国,中国对她来说就具有非常非常的特殊的意义
: 中华血脉是孩子identity的一部分 很自然而然的接受
: 很幸运 这方面没有太多struggle

t******l
发帖数: 10908
375
我就是低成本低烈度 USMC 抢个滩。
本版牛蛙是一小时打扁全球直接上 AIME 的那种。
这个够精简了吧。。。

【在 i**e 的大作中提到】
: 时塌的文,你都看懂了?
: 写个总结文科普一下吧:)
:
: 有的人擅长打“呆仗”,而有的人则擅长“浪仗”

g***3
发帖数: 471
376
你们幸好啊,
不过,生活中,这种事还是挺多的,,早遇到,早体验,以后,就会懂得如何处理了,
通过这件事,妹妹也受益的。

【在 i**e 的大作中提到】
: 脸红ing
: 7,8年前的事,搬家不是靠谱的做法,呵呵
: 当时我孩子太小我太没经验了
: 现在问同样的问题,答案是你做法,也比较知道怎么去跟孩子谈这种问题了
: 好戏剧啊
: 比电视剧好看:)
: 我小孩3年级的时候也遇到过一个很厉害的小朋友,呵呵
: 情商高特别善于manipulate peers
: 后来转走了 成了没有结果的结局:)

t******l
发帖数: 10908
377
另外本版最 fancy 的战法是四年级上 AIME,800 小时跑步进入 USAMO。
我那种 USMC 抢个滩,根本算不上 fancy。

【在 d**********h 的大作中提到】
: 有的人擅长打“呆仗”,而有的人则擅长“浪仗”
: 司令官只要以己之不可胜待敌之可胜就可以了,临阵了,把paper里最前沿最fancy的战
: 法拿出来耍宝,也不管儿郎们的实力,后勤保障力度,这基本就是“作死”
: 好在潮水兄家里有太座把关,否则。。。
:
: lift

h*****m
发帖数: 1034
378
呵呵,我带孩子可能还更多一些。。
同意你的观点,关键还是希望孩子自身强大起来。不仅精神上强大,不受或少受其他人
看法的影响,自身的实力强大以后,即使无法改变别人,也可以争取远离他们。
至于你提到的小事,其实与我最开始提的问题有点关系。我的担心,孩子们天然具有双
重身份,如果平时如果接触到的都是亲切友好又文明的亲属朋友同胞,听到的是中国古
代如何辉煌,现在如何走向强大,自然会对自己的华裔身份乐于认同。但一旦遇到与自
己利益有关的事情,小到爸爸因为他们的华裔身份要求学中文,中到受其他人嘲弄/孤
立/欺负/,或向你说的其他行为不文明的国人或有关中国的负面新闻对自己形象的损害
,大到两国利益直接发生冲突(其实,就是因为看到了你的那篇关于“中国强大以后到
世界上越多的地方占领会造福我们后代“的帖子,才有所感想,提出了关于身份认同的
问题),他们就很有可能对自己华裔身份产生怀疑,甚至想否定。而不管他们自心如何
否定,在别人眼里都是确定无疑的华裔,我是十分担心这种内外之间的矛盾可能对他们
产生的困扰。
希望我只是杞人忧天罢了。

抱歉啊,你是个好爸爸,爸爸对孩子的影响相比妈妈,对孩子,可能更让他们珍惜,因
为一般爸爸相比妈妈,和孩子相处时间较少。 孩子们往往会更重视,喜爱的人,讲的
话,说的故事。
孩子们由于他们的外表,他们源生族这个事实,肯定是抹不掉的。
我发现,我的孩子对自己的身份认同,有一个换漫的过程,偶然和他们说起来,老大说
,以前在家附近公立初中的时候,偶然有些小黑小阿会嘲笑他,但是儿子人源好,从小
一起朋友多,说他的人,都是后来搬过来,他们在学校根本没有影响,儿子不在意他们。
后来高中去了凡尔赛最好的私立高中,绝大部同学层次高,这种事就不存在了,更多是
羡慕他,能够经常到处旅行,他们学习中文,老大经常帮助同学。还有,他学习一般还
可以,给学校,给老师的影响,都是好学生的形象,,对中国的看法,更由于他现在自
己能够从学校学习的经济,历史,哲学等课程中思考中国,他对中国的热爱,是无法抹
去的。
说到这点,我想在目前,我们遇到的一个小事,问问大家的建议。
我家住在凡尔赛宫往巴黎去的最近别墅区。近年来,国内来凡尔赛宫旅游的旅游团成百
上千,我们这里由于是古城,,空间很紧张,,一些商人,买了沿大马路商铺,开了3
个饭店,孩子们上下学乘坐的bus,就在这条大路上停,昨天儿子回家说,,妈妈,我
真难过,他问我怎么办,,他说国人,到处都是,站在马路上,几十个,几十个,鼻子
一哼,喉咙一吼,,,就在路上就地处理,他说,他在公车上看到,很多车上的学生们
,都看到了,他说我们的形象啊,怎么办,,

【在 g***3 的大作中提到】
: 抱歉啊,你是个好爸爸,爸爸对孩子的影响相比妈妈,对孩子,可能更让他们珍惜,因
: 为一般爸爸相比妈妈,和孩子相处时间较少。 孩子们往往会更重视,喜爱的人,讲的
: 话,说的故事。
: 孩子们由于他们的外表,他们源生族这个事实,肯定是抹不掉的。
: 我发现,我的孩子对自己的身份认同,有一个换漫的过程,偶然和他们说起来,老大说
: ,以前在家附近公立初中的时候,偶然有些小黑小阿会嘲笑他,但是儿子人源好,从小
: 一起朋友多,说他的人,都是后来搬过来,他们在学校根本没有影响,儿子不在意他们。
: 后来高中去了凡尔赛最好的私立高中,绝大部同学层次高,这种事就不存在了,更多是
: 羡慕他,能够经常到处旅行,他们学习中文,老大经常帮助同学。还有,他学习一般还
: 可以,给学校,给老师的影响,都是好学生的形象,,对中国的看法,更由于他现在自

h*****m
发帖数: 1034
379
口音确实是我的大问题,很多音发不准。据说小时学说话时就这样,可能是天生语言能
力不强。
拿中文讨论当地的新闻目前还没有大问题,前几天还跟老大聊总统选举的事呢,什么民
主党,共和党,候选人。。一律翻译成中文。 : )
可能将来讨论他懂我不懂的东西的时候就不灵了吧。

【在 g***3 的大作中提到】
: 我没有深展论点,口音,不是问题,孩子大了,会理解,重要的是让孩子明白,父母关
: 注当地的新闻,社会日常大事,这些需要使用当地语言。。。
:
: 文。

g***3
发帖数: 471
380
是的,都是给人欺负,不敢出声的,或者不敢挑战新事物的,
都是小小的,安份守纪的公务员。3个姐妹,一个药剂师,一个工程师,一个电脑师,
都是早早结婚,生了一大堆孩子,都没有继续工作,他们的下一代,除了大姐的老大,
在美国,其他人都胆小,他们还劝我老大,不要去上预科,说很难,很苦,,
大哥,近2年离婚了,还保密,在他们2个孩子,都上大学后离的,他的前老婆很厉害,
我们是婆婆去世,公证人那里每个孩子签字,,才知道大哥离婚了,,
他们妈妈去世近2年,现在遗留下来一大堆事,被公证人脱着,还没有解决尼,,
你看他们这帮后代,都轮人宰割,,
现在是我老公,在出头,,,哎,其他兄弟姐妹还不配合,,
他们虽然6个兄妹,都不来往,,小时家庭没有教育好啊,,

【在 i**e 的大作中提到】
: 娘牛牛一窝的节奏?:)
相关主题
有明天考AMC 8的吗?少年智力开发
算不算有数学天分[转载] 为什么说usamo 简单
大家怎么推孩子的弱项?二年级女儿的report
进入Parenting版参与讨论
g***3
发帖数: 471
381
哈哈,形象

有的人擅长打“呆仗”,而有的人则擅长“浪仗”

【在 d**********h 的大作中提到】
: 有的人擅长打“呆仗”,而有的人则擅长“浪仗”
: 司令官只要以己之不可胜待敌之可胜就可以了,临阵了,把paper里最前沿最fancy的战
: 法拿出来耍宝,也不管儿郎们的实力,后勤保障力度,这基本就是“作死”
: 好在潮水兄家里有太座把关,否则。。。
:
: lift

C********e
发帖数: 2327
382
屠呦呦难道不是因为做对了project?

【在 h**w 的大作中提到】
: 版上讲男女,讲来讲去其实也就不出自己的经验和三观。
: 其实可以想想屠呦呦的例子。要说屠呦呦特别的聪明过人吧,估计这里不少人要从鼻子
: 里哼一下的。 我觉得中国在很长时间内不会再出一个屠呦呦,一个很大的原因就是,
: 中国的女性意识在抗战起因缘际会的空前高涨,在一段时间内不会再现了。 女性在那
: 个时代的思想解放,屠呦呦就是开出的最美的花。 这不是说她比别人更强什么的。在
: 那个“女性能抵半边天的”的时代背景下,女性可以得到类似的资源,没有过多的束缚
: 。比如说,屠呦呦可以把三个月的孩子扔下,不是要赞扬这种行为,但这是典型的“男
: 性优待”,女性在那个年代可以同样毫无心理负担的享受到。
: 其实在中国很多对中国女性的profile,和西方人对中国人的profile是类似的。比如
: 学校里成绩都不错,一出学校全完蛋这种。大家同不同意呢?

g***3
发帖数: 471
383
你的担忧,都是现在和将来,会遇到的现实问题,
老大,现在已经走出了,我是法国人,中国人这个疑问,他关心的是将来哪个利益最大
,先把自己练造成被社会需要的人,,老3,可能外表亚洲人不特出,还不常提,或还
没想到这层,,他有哥哥姐姐经常的解释,会感到自己很强大,他们3个很抱团,这个
我想,很重要。你们孩子间,最好抱团,多和附近的国人抱团,对孩子有好处。我们这
里居住的国人太少了,

【在 h*****m 的大作中提到】
: 呵呵,我带孩子可能还更多一些。。
: 同意你的观点,关键还是希望孩子自身强大起来。不仅精神上强大,不受或少受其他人
: 看法的影响,自身的实力强大以后,即使无法改变别人,也可以争取远离他们。
: 至于你提到的小事,其实与我最开始提的问题有点关系。我的担心,孩子们天然具有双
: 重身份,如果平时如果接触到的都是亲切友好又文明的亲属朋友同胞,听到的是中国古
: 代如何辉煌,现在如何走向强大,自然会对自己的华裔身份乐于认同。但一旦遇到与自
: 己利益有关的事情,小到爸爸因为他们的华裔身份要求学中文,中到受其他人嘲弄/孤
: 立/欺负/,或向你说的其他行为不文明的国人或有关中国的负面新闻对自己形象的损害
: ,大到两国利益直接发生冲突(其实,就是因为看到了你的那篇关于“中国强大以后到
: 世界上越多的地方占领会造福我们后代“的帖子,才有所感想,提出了关于身份认同的

C********e
发帖数: 2327
384
我不觉得语言是一个问题,父母的自信更重要。有些父母把自己不善social的nerd本质
推到语言上,娃们自然会觉得不好,这样的父母如果自豪的说我就是不喜欢social,效
果会好很多。
再说,语言不好也不耽误我们读书看报,哪里会影响社会理解。
我家娃每周从学校带回一本时事杂志,观点很左,我每周都要认真研读,再给娃洗脑

过,

【在 g***3 的大作中提到】
: 问题提得好,
: 我建议上面那位爸爸,还是需要在自己孩子还没觉察前,化功夫抓一下,自己的英文。
: 如果有一天,孩子觉察到自己父母的居住国语言不行的时候,他们也许会失落,会难过,
: 我有一些周围的例子,孩子们到十几岁,就不和父母说话了,无意识里,他们觉得父母
: 居住国语言不好,就不了解居住国文化,人文,可能会看不起自己的父母,可能会不再
: 说父母源生国的语言,有的更对抗父母,,,
: 谨慎,,
: 我的原则,,孩子说什么话,我都接受,,
: 只要他们一回家,就问妈妈在哪里,就和我们滔滔不绝,我们一起做饭,娱乐,看电影
: ,逛街,评论衣着,

t******l
发帖数: 10908
385
我想了想,我觉得这不是 “呆仗” vs “浪仗”。
这个差别,在于 spiral 理念的本质目的是降低坡度,但降低了坡度势必要以 Z-turn
的姿势前进。这样除了进展慢以外,还容易造成战略上防守但战术上进攻的不利态势,
也就是防守一大堆瞎几把傻鼻课程。
而 air-lift 理念的本质是不折腾走直线,但走直线就不仅不可能降低坡度,而且会增
大坡度。但这样首先是要轻装,轻装到 101 空中骑兵师那种猴版火炮轻坦。
但在轻装也不可能太过轻装,否则就成了行 air-lift 的名,但走 spiral 的实(因为
没有足够的火力,守不住/打不下)。这样导致严重依赖于 air-lift 的能力,也就是
严重依赖有多少 黑鹰/支奴干/V22鱼鹰。
这种情况还要具体分析,如果是天才娃那种 V22 鱼鹰要多少有多少,那直接从平顶船
air-lift 到柏林市郊就完事了。但是大部分普通娃没有那种能力,也就是黑鹰和支奴
干的差别。那样普通娃合适的战术就是 101 空中骑兵师的 “蛙跳” 式战术。建立一
个一个地面上不直接连接,但是空中在 air-lift 力所能及的距离的 “跳岛岛链”。
这样在 air-lift 能力和地面占领区域以及攻击速度三者折中一下。
但总而言之,air-lift 的理念就是走直线,不要太管坡度。轻装空中骑兵师先快速推
进建立桥头堡链/蛙跳岛链。这样的好处就是不折腾,同时容易达到战略上进攻但战术
上防守的有利态势。

【在 d**********h 的大作中提到】
: 有的人擅长打“呆仗”,而有的人则擅长“浪仗”
: 司令官只要以己之不可胜待敌之可胜就可以了,临阵了,把paper里最前沿最fancy的战
: 法拿出来耍宝,也不管儿郎们的实力,后勤保障力度,这基本就是“作死”
: 好在潮水兄家里有太座把关,否则。。。
:
: lift

t******l
发帖数: 10908
386
其实俺这么倒腾 air-lifting 的理念,说实话其实是有苦衷。。。我觉得我娃贪玩加
叛逆,一言不合就火花四溅。。。这个娃的小时数实在上不去。。。说白了鸡飞狗跳了
都上不去。。。这种情况下我要去守住 spiral 理念下 Z-turn 弯弯曲曲的漫长盘山公
路补给线,我手上娃可用的小时数兵力根本不行,严重 outnumbered。。。所以合适的
air-lifting 策略,我觉得在不失为一个没有办法的办法。。。

turn


【在 t******l 的大作中提到】
: 我想了想,我觉得这不是 “呆仗” vs “浪仗”。
: 这个差别,在于 spiral 理念的本质目的是降低坡度,但降低了坡度势必要以 Z-turn
: 的姿势前进。这样除了进展慢以外,还容易造成战略上防守但战术上进攻的不利态势,
: 也就是防守一大堆瞎几把傻鼻课程。
: 而 air-lift 理念的本质是不折腾走直线,但走直线就不仅不可能降低坡度,而且会增
: 大坡度。但这样首先是要轻装,轻装到 101 空中骑兵师那种猴版火炮轻坦。
: 但在轻装也不可能太过轻装,否则就成了行 air-lift 的名,但走 spiral 的实(因为
: 没有足够的火力,守不住/打不下)。这样导致严重依赖于 air-lift 的能力,也就是
: 严重依赖有多少 黑鹰/支奴干/V22鱼鹰。
: 这种情况还要具体分析,如果是天才娃那种 V22 鱼鹰要多少有多少,那直接从平顶船

i**e
发帖数: 19242
387
女孩
今天把这事给孩子讲了一下,问她怎么看
她把我想问的问题都答了

【在 g***3 的大作中提到】
: 妹妹的孩子,是女孩,还是男孩 ?
: 这几天,这个帖子的引深,中文吃饭时,和老大讨论,
: 他说,作为亚洲外表的男孩,他觉得是挺难的,因为大多数人,看到他们,会给他们贴
: 标签,要么功夫,要么读书好,他不希望别人给他贴标签,还有,我们在这里没有大家
: 庭,婆家没有频繁来往,这个他有时是比较痛苦的,,还有,昨天,他带弟弟坐车,去
: 凡尔赛宫花园和同学朋友夜餐,他说,弟弟啊,完全站在法国人的视野上,看到那里很
: 多中国旅游客,就象法国人那样说,中国人真多,,他说,要好好教育弟弟,别忘了自
: 己是半个中国人,,弟弟马上回击,说,自己说不来中文,老大说,妈妈失职,弟弟说
: ,他就是说事实,他看到的,,老大又为中国旅游客的形象抓急,,
: 女儿不常为这些问题困绕。也许女孩比较容易被所居住国人接受,,男孩,可能给人有

i**e
发帖数: 19242
388
Château de Versailles,我应该去过
Fontainebleau 漫步的时候,突然意识到,这个就是枫丹白露啊!!!
//对当年译名的人佩服得紧肃然起敬啊,呵呵

【在 g***3 的大作中提到】
: 妹妹的孩子,是女孩,还是男孩 ?
: 这几天,这个帖子的引深,中文吃饭时,和老大讨论,
: 他说,作为亚洲外表的男孩,他觉得是挺难的,因为大多数人,看到他们,会给他们贴
: 标签,要么功夫,要么读书好,他不希望别人给他贴标签,还有,我们在这里没有大家
: 庭,婆家没有频繁来往,这个他有时是比较痛苦的,,还有,昨天,他带弟弟坐车,去
: 凡尔赛宫花园和同学朋友夜餐,他说,弟弟啊,完全站在法国人的视野上,看到那里很
: 多中国旅游客,就象法国人那样说,中国人真多,,他说,要好好教育弟弟,别忘了自
: 己是半个中国人,,弟弟马上回击,说,自己说不来中文,老大说,妈妈失职,弟弟说
: ,他就是说事实,他看到的,,老大又为中国旅游客的形象抓急,,
: 女儿不常为这些问题困绕。也许女孩比较容易被所居住国人接受,,男孩,可能给人有

i**e
发帖数: 19242
389
那只是本版牛娃?
扁豆已走:)

【在 t******l 的大作中提到】
: 我就是低成本低烈度 USMC 抢个滩。
: 本版牛蛙是一小时打扁全球直接上 AIME 的那种。
: 这个够精简了吧。。。

i**e
发帖数: 19242
390
这就是 福祸焉知的节奏
其实,当时我挺想看看如何发展下去地,呵呵
当然有学到东西啊,娃现在交友比较慎重了,也比较懂得it takes time

【在 g***3 的大作中提到】
: 你们幸好啊,
: 不过,生活中,这种事还是挺多的,,早遇到,早体验,以后,就会懂得如何处理了,
: 通过这件事,妹妹也受益的。

相关主题
觉得美帝数学进度慢的进来看一下请教:怎样说服孩儿他爸多关心孩子的学习?
分数应用题娃又在推算他的质数公式。。。
我招,我是猪大学录取率
进入Parenting版参与讨论
i**e
发帖数: 19242
391
豪门家庭亲情理念不同吧
没事,it shall pass,总是有办法解决的虽然过程会辛苦些

【在 g***3 的大作中提到】
: 是的,都是给人欺负,不敢出声的,或者不敢挑战新事物的,
: 都是小小的,安份守纪的公务员。3个姐妹,一个药剂师,一个工程师,一个电脑师,
: 都是早早结婚,生了一大堆孩子,都没有继续工作,他们的下一代,除了大姐的老大,
: 在美国,其他人都胆小,他们还劝我老大,不要去上预科,说很难,很苦,,
: 大哥,近2年离婚了,还保密,在他们2个孩子,都上大学后离的,他的前老婆很厉害,
: 我们是婆婆去世,公证人那里每个孩子签字,,才知道大哥离婚了,,
: 他们妈妈去世近2年,现在遗留下来一大堆事,被公证人脱着,还没有解决尼,,
: 你看他们这帮后代,都轮人宰割,,
: 现在是我老公,在出头,,,哎,其他兄弟姐妹还不配合,,
: 他们虽然6个兄妹,都不来往,,小时家庭没有教育好啊,,

i**e
发帖数: 19242
392
哎呦哎,我东拉西扯地让你讲了不少家事
觉着不妥帖子就清理了吧:)
大家继续探讨如何提高学习效率:)

【在 g***3 的大作中提到】
: 是的,都是给人欺负,不敢出声的,或者不敢挑战新事物的,
: 都是小小的,安份守纪的公务员。3个姐妹,一个药剂师,一个工程师,一个电脑师,
: 都是早早结婚,生了一大堆孩子,都没有继续工作,他们的下一代,除了大姐的老大,
: 在美国,其他人都胆小,他们还劝我老大,不要去上预科,说很难,很苦,,
: 大哥,近2年离婚了,还保密,在他们2个孩子,都上大学后离的,他的前老婆很厉害,
: 我们是婆婆去世,公证人那里每个孩子签字,,才知道大哥离婚了,,
: 他们妈妈去世近2年,现在遗留下来一大堆事,被公证人脱着,还没有解决尼,,
: 你看他们这帮后代,都轮人宰割,,
: 现在是我老公,在出头,,,哎,其他兄弟姐妹还不配合,,
: 他们虽然6个兄妹,都不来往,,小时家庭没有教育好啊,,

C*****d
发帖数: 2253
393
其实随着父母的social-economic status提高,子女可以达到的社会/经济地位相对父
母而言下降的可能性也大大增加了,因为现代社会多少还是有点流动性的。像你说的你
丈夫的兄弟姐妹,其实就是一般中产,并没有特别困窘,只是和父辈比较显得差了。做
父母的总想子女一代比一代强,这个除非父母自己起点低到一定程度,否则概率其实小
于一代比一代弱的可能。

【在 g***3 的大作中提到】
: 是的,都是给人欺负,不敢出声的,或者不敢挑战新事物的,
: 都是小小的,安份守纪的公务员。3个姐妹,一个药剂师,一个工程师,一个电脑师,
: 都是早早结婚,生了一大堆孩子,都没有继续工作,他们的下一代,除了大姐的老大,
: 在美国,其他人都胆小,他们还劝我老大,不要去上预科,说很难,很苦,,
: 大哥,近2年离婚了,还保密,在他们2个孩子,都上大学后离的,他的前老婆很厉害,
: 我们是婆婆去世,公证人那里每个孩子签字,,才知道大哥离婚了,,
: 他们妈妈去世近2年,现在遗留下来一大堆事,被公证人脱着,还没有解决尼,,
: 你看他们这帮后代,都轮人宰割,,
: 现在是我老公,在出头,,,哎,其他兄弟姐妹还不配合,,
: 他们虽然6个兄妹,都不来往,,小时家庭没有教育好啊,,

t******l
发帖数: 10908
394
你的意思。。。那哥们是中药专业转行翻译的?

【在 i**e 的大作中提到】
: 哎呦哎,我东拉西扯地让你讲了不少家事
: 觉着不妥帖子就清理了吧:)
: 大家继续探讨如何提高学习效率:)

i**e
发帖数: 19242
395
哪里,就觉着特文艺范儿,又是枫又是露的:)

【在 t******l 的大作中提到】
: 你的意思。。。那哥们是中药专业转行翻译的?
t******l
发帖数: 10908
396
“丹”总给人一股重要的味道。。。但是 “风饏白露” 的话,是不是看着像厨师专业
人士?

:哪里,就觉着特文艺范儿,又是枫又是露的:)
:【 在 timefall (时光崩塌) 的大作中提到: 】
g***3
发帖数: 471
397
每个家长,都有自己擅长那个领域,如果能够尽量体现给孩子,让他们尝到喜悦,那是
最理想的。
语言是交流的方式,包括在家里,在外面,在家长朋友们之间,和各种各样外人的接触
,应付,,
家长朋友们间的交流,来往,孩子们会从小目睹,,在旁观察,到一定年龄后,参与交
谈,讨论,
还有所居住地,当地的人文,邻居,当地的社会活动,很多很多,都需要语言的娴熟,
和人接触的技巧,孩子们小时,是家长在前,大了,就是他们在前,家长在后支撑,这
种能力,从小耳闻目览,积少成多,信心是一点点来的。
就比如,女儿这周开始课外学习效率课程,,报名时,需要搭另一个课,,因为时间上
安排,她就选了数学。第一天,回家,觉得那个效率课,挺有意思,老师提出的方案,
很新颖,她从没有这样想,
但是数学,课程似乎是她以前就学过的,因为一共有6个学生,要照顾到其他学生,,
第2天,回家,觉得老师没有回答她提的问题,,然后她提了3次,似乎老师没有注意她
的要求,然后,女儿开始怀疑自己的做法,爸爸工作忙,今天早上,女儿越想越不对劲
,,她跟我说了后,,我发现她开始对数学产生抵触,,,
我想了想,决定马上和她一起去找负责人,,到了学校后,我找到了负责人,他要那个
数学老师一起来,,还有10分钟,马上要上课了,我长话短说,他们问女儿,似乎是女
儿的错,没有提问题,问老师,还问女儿几岁了,,
我看女儿,眼泪似乎在眼里打转,我还是让她表达了自己的感受,,女儿的高中很有名
,她是考进去的,说明程度还可以,也许那个课程老师没有及时察觉到女儿的需求,女
儿做的作业,也不知是对错,自己疑惑越来越大,,我请负责人考虑我女儿的状况,他
当时马上建议2条路,要么女儿继续上数学课,老师注意,要么放弃,他们寻其他解决
方法,,,他们给我们2分钟考虑,,最后女儿还是放弃了,
我们回家路上,我帮女儿分析她当时的心理,看到她有委屈,然后那个负责人那种压力
,当老师面要女儿说,有啥不对,这种情况,我希望女儿不惧怕,,要把自己放在与对
方同等的位置,来应答,要镇定,不要给负责人外表是个年长的老师的阵势给吓着了。
事实上,我们一到家,那个负责人就给我来电话,我们希望继续那个效率课,数学课费
用,是否换其他课,或还给我们。。后来他们找到下周的物理课,女儿说可以去试试。。
我对女儿说,生活中总是会出现意想不到的事,如何处理,最好就是当即立断,,不要
犹豫,很多事,就是犹豫,或为难,一拖再拖,最后成了困难。。
生活中,事越做得多,和外界有越多的冲突,都是锻炼的机会。
我老公是理工男,结婚前,长期安逸于自己的现状,在职业上,也没啥要求,对生活物
质,吃喝玩乐,也没啥特别的爱好,所以他以前,很少会遇到问题,,他说他把自己藏
在蜗牛壳里,如果以前,面临同样的问题的话,肯定会说算了,下次不去了之类,,他
虽然是土生土长法国人,以前我们家里大多数事,都是我出面周旋,同时一面锻炼他的
交流技巧,,,所以,后来老公一点点被我锻炼得也应付自如,敢于和外界挑战了,,

【在 C********e 的大作中提到】
: 我不觉得语言是一个问题,父母的自信更重要。有些父母把自己不善social的nerd本质
: 推到语言上,娃们自然会觉得不好,这样的父母如果自豪的说我就是不喜欢social,效
: 果会好很多。
: 再说,语言不好也不耽误我们读书看报,哪里会影响社会理解。
: 我家娃每周从学校带回一本时事杂志,观点很左,我每周都要认真研读,再给娃洗脑
:
: 过,

g***3
发帖数: 471
398
你问了啥,很感兴趣,

【在 i**e 的大作中提到】
: 女孩
: 今天把这事给孩子讲了一下,问她怎么看
: 她把我想问的问题都答了

g***3
发帖数: 471
399
第3代,,我下次有时间来聊,,

【在 C*****d 的大作中提到】
: 其实随着父母的social-economic status提高,子女可以达到的社会/经济地位相对父
: 母而言下降的可能性也大大增加了,因为现代社会多少还是有点流动性的。像你说的你
: 丈夫的兄弟姐妹,其实就是一般中产,并没有特别困窘,只是和父辈比较显得差了。做
: 父母的总想子女一代比一代强,这个除非父母自己起点低到一定程度,否则概率其实小
: 于一代比一代弱的可能。

g***3
发帖数: 471
400
小弟弟,后来也遇到类似的事,我们处理起来,就有余的多了。

【在 i**e 的大作中提到】
: 这就是 福祸焉知的节奏
: 其实,当时我挺想看看如何发展下去地,呵呵
: 当然有学到东西啊,娃现在交友比较慎重了,也比较懂得it takes time

相关主题
别鸡兔同笼了,来喝啤酒吧关于刷还是不刷
学校早晚要教的东西,早早学会了又如何?是不是任我儿子继续迷象棋
构建式数学在中国9岁, 如何报名考amc10
进入Parenting版参与讨论
r*g
发帖数: 3159
401
哈。枫丹白露。。。同没读出啥好来。。。
学习一下:
Fontainebleau在法文中意为“美丽的泉水”,徐志摩曾经译名为“芳丹薄罗”,但是
朱自清译作的“枫丹白露” 会让人不自觉地陷入无尽的美丽遐想。思海里有树影的摇
曳,有清秋的薄露,有季节的转换,有时光的永恒……

【在 t******l 的大作中提到】
: 你的意思。。。那哥们是中药专业转行翻译的?
d**********h
发帖数: 2795
402
潮水兄,丹在这里应该是“红色”的意思
留取丹心照汗青,文丞相是开中药铺子的?
d**********h
发帖数: 2795
403
呵呵,你家女儿几岁,不是都高中了吗?
我接触的孩子好像没有一个“怯”老师的,不论是学校里的,还是补习班的 :)
下次有相似情况,可以教娃唱一首歌:
一只老猫,有啥可怕
老鼠怕猫,那是谣传
壮起鼠胆,把猫打翻
千古偏见,一定推翻
是你女儿觉得数学课太容易了吗?换成物理课也不错!
效率课是啥?咱浅薄,第一次听说,汗-_-||。。。

【在 g***3 的大作中提到】
: 每个家长,都有自己擅长那个领域,如果能够尽量体现给孩子,让他们尝到喜悦,那是
: 最理想的。
: 语言是交流的方式,包括在家里,在外面,在家长朋友们之间,和各种各样外人的接触
: ,应付,,
: 家长朋友们间的交流,来往,孩子们会从小目睹,,在旁观察,到一定年龄后,参与交
: 谈,讨论,
: 还有所居住地,当地的人文,邻居,当地的社会活动,很多很多,都需要语言的娴熟,
: 和人接触的技巧,孩子们小时,是家长在前,大了,就是他们在前,家长在后支撑,这
: 种能力,从小耳闻目览,积少成多,信心是一点点来的。
: 就比如,女儿这周开始课外学习效率课程,,报名时,需要搭另一个课,,因为时间上

t******l
发帖数: 10908
404
她那是 6 个人的小班,抬眉不见低眉见。。。而像我们穷加州一开锅就是 30 人的大
班,1st Quarter 都过去了,老师也不一定知道我是谁!!!。。。
我觉得大班教育也有大班教育的好处,体育老师转业教数学,也能一步步往前走。。。
小班教育对老师要求要高出很多,否则上课可能搞成械斗。。。
当然我觉得她家娃说对数学有抵触,多半是气话。。。因为数学总是那个数学,跟学校
数学老师是不是体育系转业过来的,关系不大。。。

【在 d**********h 的大作中提到】
: 呵呵,你家女儿几岁,不是都高中了吗?
: 我接触的孩子好像没有一个“怯”老师的,不论是学校里的,还是补习班的 :)
: 下次有相似情况,可以教娃唱一首歌:
: 一只老猫,有啥可怕
: 老鼠怕猫,那是谣传
: 壮起鼠胆,把猫打翻
: 千古偏见,一定推翻
: 是你女儿觉得数学课太容易了吗?换成物理课也不错!
: 效率课是啥?咱浅薄,第一次听说,汗-_-||。。。

t******l
发帖数: 10908
405
如果是 “留取心丹照汗青”,那就离开中药铺子不远了。。。

【在 d**********h 的大作中提到】
: 潮水兄,丹在这里应该是“红色”的意思
: 留取丹心照汗青,文丞相是开中药铺子的?

t******l
发帖数: 10908
406
另外他们学校这课程还带搭卖的。。。我看以后大伙儿干脆 track & field 搭卖 AMC
10 好了:跑完 100 米后,大喘气的时候顺便做一道 AMC 10。再跑一个 100 米,顺便
对个答案 review 一下。。。4x100 接力正好讨论难题,接力棒上捆绑一道 AMC 10
Problem [20, 25] 好了。。。

【在 d**********h 的大作中提到】
: 呵呵,你家女儿几岁,不是都高中了吗?
: 我接触的孩子好像没有一个“怯”老师的,不论是学校里的,还是补习班的 :)
: 下次有相似情况,可以教娃唱一首歌:
: 一只老猫,有啥可怕
: 老鼠怕猫,那是谣传
: 壮起鼠胆,把猫打翻
: 千古偏见,一定推翻
: 是你女儿觉得数学课太容易了吗?换成物理课也不错!
: 效率课是啥?咱浅薄,第一次听说,汗-_-||。。。

d**********h
发帖数: 2795
407
根据我的理解是:课外课,不是正统学校里全日制的那种。。。
这种搭配才是“科学的”
比如,练田径的给你搭配力量训练,练芭蕾的给你搭配声乐训练,开卡车的给你搭配机
械原理,买豆浆的搭油条,买锤子手机搭螺丝刀,定旅馆搭租车,来班上问数学的搭“
政治思想三观”教育。。

AMC

【在 t******l 的大作中提到】
: 另外他们学校这课程还带搭卖的。。。我看以后大伙儿干脆 track & field 搭卖 AMC
: 10 好了:跑完 100 米后,大喘气的时候顺便做一道 AMC 10。再跑一个 100 米,顺便
: 对个答案 review 一下。。。4x100 接力正好讨论难题,接力棒上捆绑一道 AMC 10
: Problem [20, 25] 好了。。。

t******l
发帖数: 10908
408
Track & Field 在这里是 after-school sports club。
MOEMS div-M 在这里是 after-school math club。(这边 AMC 纯属个人行为,学校就
帮助组织考场)。
都是课外课。

【在 d**********h 的大作中提到】
: 根据我的理解是:课外课,不是正统学校里全日制的那种。。。
: 这种搭配才是“科学的”
: 比如,练田径的给你搭配力量训练,练芭蕾的给你搭配声乐训练,开卡车的给你搭配机
: 械原理,买豆浆的搭油条,买锤子手机搭螺丝刀,定旅馆搭租车,来班上问数学的搭“
: 政治思想三观”教育。。
:
: AMC

d**********h
发帖数: 2795
409
要滴,要滴
运动员一定要学生理和科学,数学作为智力开发的工具有利于运动脑瓜子的灵活,缺乏
临门一脚意识的一律补数学

【在 t******l 的大作中提到】
: Track & Field 在这里是 after-school sports club。
: MOEMS div-M 在这里是 after-school math club。(这边 AMC 纯属个人行为,学校就
: 帮助组织考场)。
: 都是课外课。

t******l
发帖数: 10908
410
我家大娃体育课倒一直都是 A,相比而言她数学课不仅从来没拿过 A,第一个 quarter
的数学甚至是 B-,差一点就要拿 C 了。
但其实我娃是有速度但无耐力型的,按现在数学课标准的话,我目测她体育课应该拿不
到 A,撑爆了也只能是 A- 。。。当然我目前的理论是,8 billion 的猴子们大概不
contaminate 体育课考评标准。

【在 d**********h 的大作中提到】
: 要滴,要滴
: 运动员一定要学生理和科学,数学作为智力开发的工具有利于运动脑瓜子的灵活,缺乏
: 临门一脚意识的一律补数学

相关主题
9岁, 如何报名考amc10算不算有数学天分
求科普: 奥数 vs. Math Olympiad vs. Math Count vs. Math circle vs. 超前学数学大家怎么推孩子的弱项?
有明天考AMC 8的吗?少年智力开发
进入Parenting版参与讨论
d**********h
发帖数: 2795
411
恭喜,贺喜
把你娃的数学搞到A,她的体育就爆棚了

quarter

【在 t******l 的大作中提到】
: 我家大娃体育课倒一直都是 A,相比而言她数学课不仅从来没拿过 A,第一个 quarter
: 的数学甚至是 B-,差一点就要拿 C 了。
: 但其实我娃是有速度但无耐力型的,按现在数学课标准的话,我目测她体育课应该拿不
: 到 A,撑爆了也只能是 A- 。。。当然我目前的理论是,8 billion 的猴子们大概不
: contaminate 体育课考评标准。

h*****m
发帖数: 1034
412
参见 Chess boxing
https://en.wikipedia.org/wiki/Chess_boxing
还有专门的组织呢。

AMC

【在 t******l 的大作中提到】
: 另外他们学校这课程还带搭卖的。。。我看以后大伙儿干脆 track & field 搭卖 AMC
: 10 好了:跑完 100 米后,大喘气的时候顺便做一道 AMC 10。再跑一个 100 米,顺便
: 对个答案 review 一下。。。4x100 接力正好讨论难题,接力棒上捆绑一道 AMC 10
: Problem [20, 25] 好了。。。

t******l
发帖数: 10908
413
你的意思是不是。。。我们的体育课培养了 USA Junior Math Olympiads?。。。而我
们的数学课培养了 USATF's Junior Olympics?。。。

【在 d**********h 的大作中提到】
: 恭喜,贺喜
: 把你娃的数学搞到A,她的体育就爆棚了
:
: quarter

g***3
发帖数: 471
414
我们这里现在是放假,这个是补习班学校。
今天早上,是找的负责人,他们的头头,比较年老的一个老师,从来没遇到过。
这个补习班,还是挺有名,全法有很多连锁部。 这种事,我们真没有想到会遇上,课
程费用也不菲啊,
因为由于女儿近来发现,这个学习效率问题,我找到他家,他们有Methodologie;;;
Methodology的课。女儿上下来,通过老师的模拟练习,学习做题试验,对自己以前的
学习习惯,有了新的认识,还是发现了自己以前做法的错误。
至于老师,他们就是不想退钱吗,,
如果数学课有效,我们也愿意,但是现在帮倒忙,我们就想设法马上制止。
所以,今天早上,有经济利益的关系,,
他们那种架势,是想让我们承担责任吗,

呵呵,你家女儿几岁,不是都高中了吗?

【在 d**********h 的大作中提到】
: 呵呵,你家女儿几岁,不是都高中了吗?
: 我接触的孩子好像没有一个“怯”老师的,不论是学校里的,还是补习班的 :)
: 下次有相似情况,可以教娃唱一首歌:
: 一只老猫,有啥可怕
: 老鼠怕猫,那是谣传
: 壮起鼠胆,把猫打翻
: 千古偏见,一定推翻
: 是你女儿觉得数学课太容易了吗?换成物理课也不错!
: 效率课是啥?咱浅薄,第一次听说,汗-_-||。。。

h*****m
发帖数: 1034
415
纯抬杠啊,从逻辑上,得是“你的父母从第三国到中国,然后你在中国长大,对中国有
归属感认同感”顺推出“你从中国来到美国,你的孩子在美国长大,对美国有归属感认
同感”。
抬杠完毕,表示认同你的结论。我父亲一代从山东搬到现在所在地,我在那出生长大,
归属感不在山东。但听到“山东好汉武二郎”还是有种自豪亲切感,对“山西白眉大侠
”就没有这种感觉,虽然据说几百年前先人也是从山西大槐树下走过来的。

【在 i**e 的大作中提到】
: 我对祖国有归属感,米国住再久也没有这个感觉
: 顺推 生长在米国的孩子的归属感认同感在米国
: 我很幸运 跟孩子能交心情感上非常近
: 孩子爱屋及乌,中国是妈妈的母国,中国对她来说就具有非常非常的特殊的意义
: 中华血脉是孩子identity的一部分 很自然而然的接受
: 很幸运 这方面没有太多struggle

t******l
发帖数: 10908
416
我觉得比 4x100 接力 IMO 差远了。。。因为 chess 属于棋类运动,而棋类运动也算
是体育类不是?

【在 h*****m 的大作中提到】
: 参见 Chess boxing
: https://en.wikipedia.org/wiki/Chess_boxing
: 还有专门的组织呢。
:
: AMC

h*****m
发帖数: 1034
417
我觉得你的做法非常正确,孩子对数学抵触很有可能跟老师有关。
对于某些孩子来说,老师的影响是巨大的,不只是老师水平的问题,老师的态度,对学
生的反应,孩子喜不喜欢这个老师,都会影响孩子对课程本身的兴趣。很多上中学的孩
子都是特别的情绪化。
我高中的同学不只一个都曾经因为不喜欢某个老师连带着不喜欢那个老师教的课,故意
不去好好学。
多年以后,一起聊天谈到当时的情况,后悔当初的幼稚。我当时也有类似的幼稚,幸好
没有表现在学习上。

【在 g***3 的大作中提到】
: 我们这里现在是放假,这个是补习班学校。
: 今天早上,是找的负责人,他们的头头,比较年老的一个老师,从来没遇到过。
: 这个补习班,还是挺有名,全法有很多连锁部。 这种事,我们真没有想到会遇上,课
: 程费用也不菲啊,
: 因为由于女儿近来发现,这个学习效率问题,我找到他家,他们有Methodologie;;;
: Methodology的课。女儿上下来,通过老师的模拟练习,学习做题试验,对自己以前的
: 学习习惯,有了新的认识,还是发现了自己以前做法的错误。
: 至于老师,他们就是不想退钱吗,,
: 如果数学课有效,我们也愿意,但是现在帮倒忙,我们就想设法马上制止。
: 所以,今天早上,有经济利益的关系,,

g***3
发帖数: 471
418
是假期补习班,当时报名时,说是要一门专业课,加methodology的课,
因为methodology是11点15到13点15,,物理也是同一时间,所以才选了数学。
当时我和他们说清楚,女儿需要的是空间几何,他们说给老师加注了。
到上课的那天,因为其他学生是其他学校,相比女儿,还浅了点,,
到上课时,老师出题,然后大家做,女儿做题方式和老师不同,然后连问老师,老师要
她自己想,
就去照顾其他学生了,,然后,然后,半个小时过去了,女儿越听别的,越对自己的做
法,不知所错了,疑惑多了,,第2天,去,还是老样子,,没劲了。
话外,女儿现在就读的法国德国联合高中,是在法唯一的一个德法联合的国家级公立高
中,在德国还有2家,法国菲尔茨数学奖获得者,就是从小这个学校来的,,这个学校
的理工科,在法国是很有名的,学生都是进过国家竟考,挑选的。
补习班的老师,我今天早上,见到了,是个年轻人,,,也许,经验不足吧,,
女儿说,她高中的数学老师,解释的很清楚,这个老师,他越说,越给他搞糊涂了。。

【在 t******l 的大作中提到】
: 她那是 6 个人的小班,抬眉不见低眉见。。。而像我们穷加州一开锅就是 30 人的大
: 班,1st Quarter 都过去了,老师也不一定知道我是谁!!!。。。
: 我觉得大班教育也有大班教育的好处,体育老师转业教数学,也能一步步往前走。。。
: 小班教育对老师要求要高出很多,否则上课可能搞成械斗。。。
: 当然我觉得她家娃说对数学有抵触,多半是气话。。。因为数学总是那个数学,跟学校
: 数学老师是不是体育系转业过来的,关系不大。。。

h*****m
发帖数: 1034
419
那倒是。不过你有没有说过,将来阿发狗横行天下,IMO也要被归到体育类里面去了?

【在 t******l 的大作中提到】
: 我觉得比 4x100 接力 IMO 差远了。。。因为 chess 属于棋类运动,而棋类运动也算
: 是体育类不是?

t******l
发帖数: 10908
420
倒也是。。。看来最强的应该是:
USATF Junior Olympics 4x100 relay USA Junior Computer Olympiads
还有更强的么?

【在 h*****m 的大作中提到】
: 那倒是。不过你有没有说过,将来阿发狗横行天下,IMO也要被归到体育类里面去了?
相关主题
[转载] 为什么说usamo 简单分数应用题
二年级女儿的report我招,我是猪
觉得美帝数学进度慢的进来看一下请教:怎样说服孩儿他爸多关心孩子的学习?
进入Parenting版参与讨论
g***3
发帖数: 471
421
是的,我们自己都是曾经的学生,我也是初中最后一年,数学老师一次调我上黑板,慌
张,没做出,就把我当差生,我们那时是分好中差组吗,整个学期,抬不起头来,,心
理阴影一直很深,,
幸好后来上高中,遇到了好老师,,
我老大,去年也是得,
所以我现在对这种问题特敏感,,心理愉快,学习记忆效果成倍,

【在 h*****m 的大作中提到】
: 我觉得你的做法非常正确,孩子对数学抵触很有可能跟老师有关。
: 对于某些孩子来说,老师的影响是巨大的,不只是老师水平的问题,老师的态度,对学
: 生的反应,孩子喜不喜欢这个老师,都会影响孩子对课程本身的兴趣。很多上中学的孩
: 子都是特别的情绪化。
: 我高中的同学不只一个都曾经因为不喜欢某个老师连带着不喜欢那个老师教的课,故意
: 不去好好学。
: 多年以后,一起聊天谈到当时的情况,后悔当初的幼稚。我当时也有类似的幼稚,幸好
: 没有表现在学习上。

t******l
发帖数: 10908
422
航模组的,书包里装着长短不等的航模刀。。。短跑队的,书包里有一双共 16 只钉子
的钉鞋。。。我觉得问题的关键是要有合法武器携带证,那心理就一定会强大。。。
// super fast run

【在 g***3 的大作中提到】
: 是的,我们自己都是曾经的学生,我也是初中最后一年,数学老师一次调我上黑板,慌
: 张,没做出,就把我当差生,我们那时是分好中差组吗,整个学期,抬不起头来,,心
: 理阴影一直很深,,
: 幸好后来上高中,遇到了好老师,,
: 我老大,去年也是得,
: 所以我现在对这种问题特敏感,,心理愉快,学习记忆效果成倍,

g***3
发帖数: 471
423
今天通过这件事,女儿还是很高兴的,她回家后,说methodologie让她突然意识到以前
,,每次考试,都是到最后一时,才去复习,做卡片,时间化的太多,来不及了,就开
始慌张,就连卡片上的内容,考试时,也忘了,,,
下午,我们一起去逛街买衣服了,,
有时,家长,老师的话,说的时候,孩子可能并不真正理解,他们需要自己碰上了,才
会真正意识到。这个methodologie对她来说,还是有效果的。我为她高兴。
g***3
发帖数: 471
424
女儿这个时段的孩子,还有一个很重要点,不能被忽视了,就是不能在其他学生面前出
丑,,
她即不想让别人知道,她是那个高中的,不想让别人认为,她高不可攀,,她也不想让
别人认为,她笨,,所以,很难找到适合点,,哥哥,也怪我,这个课程,不是他们的
水平,出钱买烦恼。

【在 t******l 的大作中提到】
: 航模组的,书包里装着长短不等的航模刀。。。短跑队的,书包里有一双共 16 只钉子
: 的钉鞋。。。我觉得问题的关键是要有合法武器携带证,那心理就一定会强大。。。
: // super fast run

t******l
发帖数: 10908
425
我觉得这个除了个性以外,跟中小学的小学求满分、高中比竞争的 “Z-turn 型教学法
“ 有一定的关系。。。一个比较明显的现象就是,怕出丑的更多在 honor roll 三好
学生里;而各类竞赛组的主流是 “快糙猛” 的不怕出丑型,不管是学科竞赛还是体育
竞赛还是棋类航模车模大赛。。。当然当年的车模现在怎么变选美了,这玩意儿俺也看
不懂。。。

:女儿这个时段的孩子,还有一个很重要点,不能被忽视了,就是不能在其他学生面前
出丑,,
i**e
发帖数: 19242
426
就是我回你贴里的那些问题啊:)
当然,她还小,所以我没问根表之别,只是问了,如果想改变这个状况,你会怎么做?

【在 g***3 的大作中提到】
: 你问了啥,很感兴趣,
i**e
发帖数: 19242
427
扑哧
原来是朱自清
记得《荷塘月色》考试的时候要求默写的,当然是片段啦
不过我个人倒是更喜欢他的《背影》,为啥课文选了荷而没有选影呢?
大概荷忧国忧民大情怀,背影太小情调了,呵呵

【在 r*g 的大作中提到】
: 哈。枫丹白露。。。同没读出啥好来。。。
: 学习一下:
: Fontainebleau在法文中意为“美丽的泉水”,徐志摩曾经译名为“芳丹薄罗”,但是
: 朱自清译作的“枫丹白露” 会让人不自觉地陷入无尽的美丽遐想。思海里有树影的摇
: 曳,有清秋的薄露,有季节的转换,有时光的永恒……

i**e
发帖数: 19242
428
是一个意思吧
父母从哪儿来的,都敌不过一个人是在哪儿成长的

【在 h*****m 的大作中提到】
: 纯抬杠啊,从逻辑上,得是“你的父母从第三国到中国,然后你在中国长大,对中国有
: 归属感认同感”顺推出“你从中国来到美国,你的孩子在美国长大,对美国有归属感认
: 同感”。
: 抬杠完毕,表示认同你的结论。我父亲一代从山东搬到现在所在地,我在那出生长大,
: 归属感不在山东。但听到“山东好汉武二郎”还是有种自豪亲切感,对“山西白眉大侠
: ”就没有这种感觉,虽然据说几百年前先人也是从山西大槐树下走过来的。

h*****m
发帖数: 1034
429
后面这个是不是相当于遛阿发狗?赛狗也是体育项目啊。
要不上哲学吧。这个不大懂,听起来有点高大上。

【在 t******l 的大作中提到】
: 倒也是。。。看来最强的应该是:
: USATF Junior Olympics 4x100 relay USA Junior Computer Olympiads
: 还有更强的么?

h*****m
发帖数: 1034
430
你准备每个老师桌子上插几把?: )

【在 t******l 的大作中提到】
: 航模组的,书包里装着长短不等的航模刀。。。短跑队的,书包里有一双共 16 只钉子
: 的钉鞋。。。我觉得问题的关键是要有合法武器携带证,那心理就一定会强大。。。
: // super fast run

相关主题
娃又在推算他的质数公式。。。学校早晚要教的东西,早早学会了又如何?
大学录取率构建式数学在中国
别鸡兔同笼了,来喝啤酒吧关于刷还是不刷
进入Parenting版参与讨论
h*****m
发帖数: 1034
431
我就是觉得需要把父母的影响排除掉,才有可比性啊。
不用理我,我这是理科思维强迫症犯了。

【在 i**e 的大作中提到】
: 是一个意思吧
: 父母从哪儿来的,都敌不过一个人是在哪儿成长的

t******l
发帖数: 10908
432
我想了想,我觉得这可能不是 “怕不怕” 出丑的问题,而是 “能不能” 出丑的问题
。或者这么比方:
推着独轮车沿 spiral 的盘山公路 Z-turn 的 Teacher's pet,当然要在独轮车里装个
罗马大理石的抽水马桶。。。这根本就不是啥 “愿意不愿意”(或者说,“怕不怕”
)的问题。不信的可以去黄石国家公园的盘山公路上拉个大号,同时跟 Park Ranger
聊个天气啥的。。。
而在上下两顶帐篷之间直上直下的,蛙跳式背水背食物的 “快糙猛”型登山蛙,要是
背个抽水马桶上去,那会不会被人笑死才是个问题。。。那种大伙儿都直接 expect 背
个工兵铲,去小树林里挖个坑。。。或者说,这种是属于被大众认可的 “出丑”,跟
在黄石国家公园的盘山公路上用工兵铲挖个坑拉大号,那根本就不是一码事。。。虽然
两者的共同点都是挖个坑。

【在 t******l 的大作中提到】
: 我觉得这个除了个性以外,跟中小学的小学求满分、高中比竞争的 “Z-turn 型教学法
: “ 有一定的关系。。。一个比较明显的现象就是,怕出丑的更多在 honor roll 三好
: 学生里;而各类竞赛组的主流是 “快糙猛” 的不怕出丑型,不管是学科竞赛还是体育
: 竞赛还是棋类航模车模大赛。。。当然当年的车模现在怎么变选美了,这玩意儿俺也看
: 不懂。。。
:
: :女儿这个时段的孩子,还有一个很重要点,不能被忽视了,就是不能在其他学生面前
: 出丑,,
: :

C********e
发帖数: 2327
433
如果怕出错,总是会出错。但是这种性格上的问题也比较难搞。

【在 g***3 的大作中提到】
: 女儿这个时段的孩子,还有一个很重要点,不能被忽视了,就是不能在其他学生面前出
: 丑,,
: 她即不想让别人知道,她是那个高中的,不想让别人认为,她高不可攀,,她也不想让
: 别人认为,她笨,,所以,很难找到适合点,,哥哥,也怪我,这个课程,不是他们的
: 水平,出钱买烦恼。

d**********h
发帖数: 2795
434
你们那里课外补课的多吗?
一般是什么人会去补习班(提高进阶班)?
孩子有没有抱怨过为啥xxx不需要在周末或者课余补习?或者说,妈妈你注意了吗,只
有中国孩子需要周末去中文学校 :)

【在 g***3 的大作中提到】
: 是假期补习班,当时报名时,说是要一门专业课,加methodology的课,
: 因为methodology是11点15到13点15,,物理也是同一时间,所以才选了数学。
: 当时我和他们说清楚,女儿需要的是空间几何,他们说给老师加注了。
: 到上课的那天,因为其他学生是其他学校,相比女儿,还浅了点,,
: 到上课时,老师出题,然后大家做,女儿做题方式和老师不同,然后连问老师,老师要
: 她自己想,
: 就去照顾其他学生了,,然后,然后,半个小时过去了,女儿越听别的,越对自己的做
: 法,不知所错了,疑惑多了,,第2天,去,还是老样子,,没劲了。
: 话外,女儿现在就读的法国德国联合高中,是在法唯一的一个德法联合的国家级公立高
: 中,在德国还有2家,法国菲尔茨数学奖获得者,就是从小这个学校来的,,这个学校

d**********h
发帖数: 2795
435
明白了,潮水兄的保命绝招就是rocket run,俗称潮跑跑

【在 t******l 的大作中提到】
: 航模组的,书包里装着长短不等的航模刀。。。短跑队的,书包里有一双共 16 只钉子
: 的钉鞋。。。我觉得问题的关键是要有合法武器携带证,那心理就一定会强大。。。
: // super fast run

d**********h
发帖数: 2795
436
我觉得lz闺女是超前,不是忌讳“出丑”
她只是还没有强大到去主动challenge老师(出彩),或者被老师challenge(出丑),虽然
意识到和老师之间有问题。
经过这个事情之后,尤其是她老妈带她去challenge一次老师和领导之后,她就不会
再怵这种事情了

【在 t******l 的大作中提到】
: 我想了想,我觉得这可能不是 “怕不怕” 出丑的问题,而是 “能不能” 出丑的问题
: 。或者这么比方:
: 推着独轮车沿 spiral 的盘山公路 Z-turn 的 Teacher's pet,当然要在独轮车里装个
: 罗马大理石的抽水马桶。。。这根本就不是啥 “愿意不愿意”(或者说,“怕不怕”
: )的问题。不信的可以去黄石国家公园的盘山公路上拉个大号,同时跟 Park Ranger
: 聊个天气啥的。。。
: 而在上下两顶帐篷之间直上直下的,蛙跳式背水背食物的 “快糙猛”型登山蛙,要是
: 背个抽水马桶上去,那会不会被人笑死才是个问题。。。那种大伙儿都直接 expect 背
: 个工兵铲,去小树林里挖个坑。。。或者说,这种是属于被大众认可的 “出丑”,跟
: 在黄石国家公园的盘山公路上用工兵铲挖个坑拉大号,那根本就不是一码事。。。虽然

t******l
发帖数: 10908
437
如果是在没有把握老师会不会恼羞成怒的情况下,选择不去 challenge 老师,或者被
老师小的 challenge 就 eat loss 算了,那我觉得不仅没啥不好,反而是一个好的
trait。。。说白了将来上班也不能没事去 challenge 老板不是?。。。而且人都趋向
于自信,工作时的 peer 水平常常更高,或者旁观者更清,更不该没事去 challenge。
。。就算后来发现是老板错了,那帮老板一起混过去,老板也不会开替罪羊。。。公司
的事儿更是 shareholder / board 担心的事儿,小兵张嘎也犯不着沟命海心。公司垮
了换一家就是了。
当然我承认这样很不高尚,但北鸟也说了,“高尚是高尚者的墓志铭”。为了自己生存
又不害别人,我觉得完全可以理直气壮地不去 challenge。

【在 d**********h 的大作中提到】
: 我觉得lz闺女是超前,不是忌讳“出丑”
: 她只是还没有强大到去主动challenge老师(出彩),或者被老师challenge(出丑),虽然
: 意识到和老师之间有问题。
: 经过这个事情之后,尤其是她老妈带她去challenge一次老师和领导之后,她就不会
: 再怵这种事情了

c***x
发帖数: 1826
438

仔细想了想,这首英文打油诗其实有点深意。
要不要去参加诗歌月活动?
http://www.mitbbs.com/article_t2/LeisureTime/2033539.html

【在 t******l 的大作中提到】
: 看完上面这些 video,就会发现,其实所谓的那些 “感性”,不过就是一些 space-
: time-pattern,用一种 educated sporadic-chaotic 的方式,比如就这么整:
: Spontaneous Symmetry Breaking
: -- by timefall, 2016.04.18
: You play major,
: You play minor;
: You sound augmented,
: You sound diminished;
: Then, ... (You gave me perfect-fifth?)
: You syncopate left,

c***x
发帖数: 1826
439

说的好,点个赞。
有人曾开玩笑说,要是把家长老师的经验,都浓缩成精华,给孩子打一针就全掌握了,
该多好啊。
后来发现,再深的道理,再好的智慧,也必须由孩子自己去经历才能真正明白和掌握。
我们的间接经验,最多也就是打个预防针吧,当然也很重要就是了。

【在 g***3 的大作中提到】
: 女儿这个时段的孩子,还有一个很重要点,不能被忽视了,就是不能在其他学生面前出
: 丑,,
: 她即不想让别人知道,她是那个高中的,不想让别人认为,她高不可攀,,她也不想让
: 别人认为,她笨,,所以,很难找到适合点,,哥哥,也怪我,这个课程,不是他们的
: 水平,出钱买烦恼。

c***x
发帖数: 1826
440

烦劳你们三位继续逗潮水玩,玩着玩着,他或许就能蹦出一个有趣的理论。。。
理论贡献的归属问题,到时候可以商榷。

【在 i**e 的大作中提到】
: 那只是本版牛娃?
: 扁豆已走:)

相关主题
关于刷还是不刷求科普: 奥数 vs. Math Olympiad vs. Math Count vs. Math circle vs. 超前学数学
是不是任我儿子继续迷象棋有明天考AMC 8的吗?
9岁, 如何报名考amc10算不算有数学天分
进入Parenting版参与讨论
t******l
发帖数: 10908
441
俺 anti-social 啊。。。

【在 c***x 的大作中提到】
:
: 烦劳你们三位继续逗潮水玩,玩着玩着,他或许就能蹦出一个有趣的理论。。。
: 理论贡献的归属问题,到时候可以商榷。

g***3
发帖数: 471
442
女儿在学业上,以前很少参加竞赛,即使有,都是因为课目老师参加全国的比赛,然后
要求全班学生都报名。成绩也不是很在乎的,也没特别赛前训练。
体育比赛,她倒是以前代表学校,到外地去参加全国性青少年乒乓球竞标赛,和赛艇,
我们都没跟着去,重在他们的参与。
女儿现在这个阶段,是青少年摸索自己的形象,确认自己在社会中强落位置的阶段,我
观察到,女孩,和男孩,表现又很不同,

【在 t******l 的大作中提到】
: 我觉得这个除了个性以外,跟中小学的小学求满分、高中比竞争的 “Z-turn 型教学法
: “ 有一定的关系。。。一个比较明显的现象就是,怕出丑的更多在 honor roll 三好
: 学生里;而各类竞赛组的主流是 “快糙猛” 的不怕出丑型,不管是学科竞赛还是体育
: 竞赛还是棋类航模车模大赛。。。当然当年的车模现在怎么变选美了,这玩意儿俺也看
: 不懂。。。
:
: :女儿这个时段的孩子,还有一个很重要点,不能被忽视了,就是不能在其他学生面前
: 出丑,,
: :

g***3
发帖数: 471
443
她现在几岁啦?

【在 i**e 的大作中提到】
: 就是我回你贴里的那些问题啊:)
: 当然,她还小,所以我没问根表之别,只是问了,如果想改变这个状况,你会怎么做?

g***3
发帖数: 471
444
我觉得是,在10岁前,父母影响大与一半,10岁后,环境影响大与父母影响一半,
环境也是父母潜意识,创造的吧。

【在 h*****m 的大作中提到】
: 我就是觉得需要把父母的影响排除掉,才有可比性啊。
: 不用理我,我这是理科思维强迫症犯了。

g***3
发帖数: 471
445
孩子的性格,从小宝宝最初的大线条基本性格定型,,到随年龄增长,每个阶段不同年
龄,不同外界接触,不断的雕塑,女孩,按个性不同,在14,岁到17岁,这段,我认为
应该是最难搞的,但不说明,她小时候,是这样,也不能确定,她以后一直会是这样。
女孩和男孩,又很多地方不同,,男孩,会还晚一些出现,
我小时候,常被认为难搞,我到15岁,以后,自己主意就很大了,和父母的交流只停留
在吃喝上了。
我记得,我很小的时候,食物还是稀少的年代,比如有好吃的,别人问你,要不要吃,
礼貌上要回答,不要,但是内心里,希望,别人能给你,,,如果别人,尤其父母,不
给你,那么会难过,,会觉得父母不爱自己,,,
现在的孩子也是一样的,但比当时那种食物情况,更复杂,我觉得,如果我能平时猜透
孩子们的心思,他们的情绪状态,会一直保持很好,,和我的话,会一直很多,,事实
上,大孩已平安的渡过了反叛期,,他更是很明确自己所想,以后生活的打算,我们日
常话很多,,比如,昨天,他去一个同学晚会,今天中午,就想跟我说话,讨论,,,
女儿,每天出门前,会和我讨论她的穿着,问我的意见,有时,自己主意也很大,把好
好的裤子,中间剪一缝,,我们经常一起逛街,,她现在比我高了,,然后,她穿小的
衣服,都给了我穿,,呵呵,,我以前喜欢的书,她也喜欢,,他们在寻找,自己的形
象,他们现在这种状态,如果我们能够一直继续下去的话,,到他们成人,成事,我的
生活会丰富无穷,,
这个就是我所希望,理想的生活,,

【在 C********e 的大作中提到】
: 如果怕出错,总是会出错。但是这种性格上的问题也比较难搞。
g***3
发帖数: 471
446
女儿,现在是第一次,去外面上课。
介绍下,法国的教育背景,,每年大概有60多万到70万高中毕业生,,
拿破仑制定的预科精英制学校,理工商文,全包,每年大概招2,到3万人,,
这个2,到3万学生,还要通过2年的苦读,然后再参加全国竟考,,根据排名先后,再
被分配到不同名次的精英学校。。比如儿子想上的巴黎3个高商之一,每年招300到400
个学生,所以,考试时,在5千多人里,要进入前300名等。。
这个2,到3万名学生的挑选,是根据他们高中的学习成绩。预科学校也有排名,越前的
学校,进入顶级的精英学校人数越多。
我们孩子小时,去周日中文学校,2年多后,放弃。那时他们也有抱怨。
但是他们人大了后,特别是到高中后,发现,很多同学,都是从小家里就有私教的,
他们现在的高中,根据不同年份,在全国2600多个综合高中,排前15名。。高中根据成
绩,和各方课外活动,挑选学生成分越多,政治正确,学校排名越后,他们的高中,传
统上,都是很好的高中。
老大高中也有商预科,排名全国第7.每年3分之一学生,能够进入巴黎3个高商。。
最好的商预科高中在巴黎市区,所以他想去巴黎市区就读,,但是实际上,2年的苦读
,类似我们当时的高考,需要住在学校附近,全部学习时间用来学习,,我们家去巴黎
市区,根据他要去的学校,每天要40分钟单程公交,不然要在学校附近租房,,,增加
家庭费用,,,
老大,在我们全量下,还是同意继续在本高中读预科,优点,离家近,15分钟,然后我
们能照顾他日常生活。
现在的补习班,是女儿自己要求的,,
法国这里,我发现,父母自己从小精英系统读上来的,对自己的孩子的教育,从小会越
小心,
巴黎有2个私立的高中,他们从孩子3岁,进幼儿园时,就有严格的测试,和赛选了,还
要看父母,祖父母的背景,所以,我前面说过,法国的文凭,是很重要的,尤其是那几
个传统精英学校。

你们那里课外补课的多吗?

【在 d**********h 的大作中提到】
: 你们那里课外补课的多吗?
: 一般是什么人会去补习班(提高进阶班)?
: 孩子有没有抱怨过为啥xxx不需要在周末或者课余补习?或者说,妈妈你注意了吗,只
: 有中国孩子需要周末去中文学校 :)

d**********h
发帖数: 2795
447
nice,
学习了,本楼绝对值得收藏!
60~70万应届高中毕业生,两三万去精英学校,其他人呢?
法国的预科是怎么回事?college之前的,在高中完成的?有多少预科结束不继续
college的?不知道法国的门槛高矮,至少层数够多,我猜每层都能阻拦一部分人。。。
孩子一般六岁入学,十八岁预科结束,还是高中结束?college毕业多少岁?我看了一
个法国牛人23岁就phd毕业了,这是一个什么节奏?
法国高中大学筛选的时候,课外活动有多重要?法国有没有对少数民族的照顾之类?比
如公司政府中的女性,穆斯林不得低于一个比例之类。。。
四十分钟的单程不算远,呵呵 :)

400

【在 g***3 的大作中提到】
: 女儿,现在是第一次,去外面上课。
: 介绍下,法国的教育背景,,每年大概有60多万到70万高中毕业生,,
: 拿破仑制定的预科精英制学校,理工商文,全包,每年大概招2,到3万人,,
: 这个2,到3万学生,还要通过2年的苦读,然后再参加全国竟考,,根据排名先后,再
: 被分配到不同名次的精英学校。。比如儿子想上的巴黎3个高商之一,每年招300到400
: 个学生,所以,考试时,在5千多人里,要进入前300名等。。
: 这个2,到3万名学生的挑选,是根据他们高中的学习成绩。预科学校也有排名,越前的
: 学校,进入顶级的精英学校人数越多。
: 我们孩子小时,去周日中文学校,2年多后,放弃。那时他们也有抱怨。
: 但是他们人大了后,特别是到高中后,发现,很多同学,都是从小家里就有私教的,

d**********h
发帖数: 2795
448
哈哈
我发个宏愿:待得此楼不长之时,做个干净的合集,剔除一切,只留楼主真言。。。
:)

【在 c***x 的大作中提到】
:
: 烦劳你们三位继续逗潮水玩,玩着玩着,他或许就能蹦出一个有趣的理论。。。
: 理论贡献的归属问题,到时候可以商榷。

c***x
发帖数: 1826
449

人人都只发宏愿不行动,最后宏愿也就是泡影。
请认真考虑一下去申请做版主;到时候该合集的合集,该mark的mark,该活动的活动。
于你,“人生也就是一个作字”嘛。
于版友,龙吸言之有物,论之有情,中正平和,有理有礼有节,支持的会很多,反对的
人应该没有。

【在 d**********h 的大作中提到】
: 哈哈
: 我发个宏愿:待得此楼不长之时,做个干净的合集,剔除一切,只留楼主真言。。。
: :)

d**********h
发帖数: 2795
450
呵呵,合集是为我自己以后参考的,你如果是想要我可以发给你哦。(lz既然发在公共
板块,想必是不介意我们收藏的)
至于其他,野火即使不收了草原,草原自己一岁也会枯荣交替一次;即使枯荣往复,随
春风勃勃而兴的生命也已经不是去年的那些老帮菜了
咱努力做好草木灰吧 :),别拿春风这么高大上的角色吓唬我了,弄得咱心里很慌张

【在 c***x 的大作中提到】
:
: 人人都只发宏愿不行动,最后宏愿也就是泡影。
: 请认真考虑一下去申请做版主;到时候该合集的合集,该mark的mark,该活动的活动。
: 于你,“人生也就是一个作字”嘛。
: 于版友,龙吸言之有物,论之有情,中正平和,有理有礼有节,支持的会很多,反对的
: 人应该没有。

相关主题
大家怎么推孩子的弱项?二年级女儿的report
少年智力开发觉得美帝数学进度慢的进来看一下
[转载] 为什么说usamo 简单分数应用题
进入Parenting版参与讨论
r*g
发帖数: 3159
451
法国这么早分流,那么那些被分流的不会不甘心?还有机会跳上去吗?

400

【在 g***3 的大作中提到】
: 女儿,现在是第一次,去外面上课。
: 介绍下,法国的教育背景,,每年大概有60多万到70万高中毕业生,,
: 拿破仑制定的预科精英制学校,理工商文,全包,每年大概招2,到3万人,,
: 这个2,到3万学生,还要通过2年的苦读,然后再参加全国竟考,,根据排名先后,再
: 被分配到不同名次的精英学校。。比如儿子想上的巴黎3个高商之一,每年招300到400
: 个学生,所以,考试时,在5千多人里,要进入前300名等。。
: 这个2,到3万名学生的挑选,是根据他们高中的学习成绩。预科学校也有排名,越前的
: 学校,进入顶级的精英学校人数越多。
: 我们孩子小时,去周日中文学校,2年多后,放弃。那时他们也有抱怨。
: 但是他们人大了后,特别是到高中后,发现,很多同学,都是从小家里就有私教的,

t******l
发帖数: 10908
452
做野火难道不是更爽?。。。一高兴就烧特么一把。。。

【在 d**********h 的大作中提到】
: 呵呵,合集是为我自己以后参考的,你如果是想要我可以发给你哦。(lz既然发在公共
: 板块,想必是不介意我们收藏的)
: 至于其他,野火即使不收了草原,草原自己一岁也会枯荣交替一次;即使枯荣往复,随
: 春风勃勃而兴的生命也已经不是去年的那些老帮菜了
: 咱努力做好草木灰吧 :),别拿春风这么高大上的角色吓唬我了,弄得咱心里很慌张

i**e
发帖数: 19242
453
呵呵
我一般对这种比较敏感的话题几乎不参与:)
很多事情没有那么多的对错,每个人都有自己的信仰和理念
只是分享没有说服的目的:)
这次是lz真诚坦荡分享,我多说了三四句,所谓言多必失:)

【在 h*****m 的大作中提到】
: 我就是觉得需要把父母的影响排除掉,才有可比性啊。
: 不用理我,我这是理科思维强迫症犯了。

i**e
发帖数: 19242
454
6年级
跟你大儿子比小多啦

【在 g***3 的大作中提到】
: 她现在几岁啦?
i**e
发帖数: 19242
455
啧啧
玻璃心碎了一地哦:)
原来我算不上污言秽语至少也是虚言假语呢,呵呵

【在 d**********h 的大作中提到】
: 哈哈
: 我发个宏愿:待得此楼不长之时,做个干净的合集,剔除一切,只留楼主真言。。。
: :)

d**********h
发帖数: 2795
456
真言人人有
此番楼主家
玻璃碎一地
当心小脚丫
:)

【在 i**e 的大作中提到】
: 啧啧
: 玻璃心碎了一地哦:)
: 原来我算不上污言秽语至少也是虚言假语呢,呵呵

g***3
发帖数: 471
457
法国从小学6岁开始,小学5年,初中4年,高中3年,,我说的法国预科,是法国特殊的
体制,是对高中毕业,相当于大学2年的课程,但是这个课程,可以说,是法国特色,
是特别难的,,是拿破仑当时强国振兴,从平民,小资本家庭挑选人才的一种学习,竟
考形式。学生通过一般2年的强化学习,训练,去竟考,根据名次不同,分别再进相应
学校grande ecole;,再读3年,出来文凭相当于Master,你说的牛人,他跳过几级,,
15岁可能就高中毕业了。
其他还有政治学院,和学医,学管理,读法律,读艺术等甩选严格的特殊大学,巴黎索
邦大学管理,法律学科也很难进入,这样,大概每年也可接纳一万人左右,余下的普通
大学就很多了,只要通过高中会考Bac 都可以读的大学,法国每年高中会考合格率达百
分90以上。
近十几年来,法国也开始流行如英美之类的bachelor 文凭的。这个是高中毕业前夕,
就可以去考的,我老大认识的好几个同学,现在都在这个阶段,,这个考试,相对预科
,容易些,,考进后,学制有4,年或5年的,每年学费读商的话,也要1万多欧,不包
住宿,食物,交通,书杂费等,读完,包各种费用,也要十万欧以上,毕业后文凭价值
比读普通大学的好,但大大低于从预科那段进入竟考的文凭有价值。
读预科的话,相对成本便宜,如果选离家近的,每年最多3000欧, 读2年,,考入商学
院后,才交每年一万多欧的学费,很多学院都和英美大学有联合,商学院第2年,就可
以去英美相应大学学习,学费按法国这里的算,,文凭是双大学的。
预科在法国很特殊,老师会鼓励学习好,有远大志向的学生,去学习,去考。我们上次
去参加老大学校的家长会,老师就说,在法国读预科,对一生的事业都很重要,以后大
公司选人,就喜欢选这些久经百炼的人。读预科,首先锻炼意志力,尤其数学,会遇到
很多困难,,拿破仑从小就对数学很感兴趣,是他把数学当成最重要的科目,测试每个
学生。学会思考解决问题的方法,还有学习各种不同的文化,交流方式。
预科期间,学习节奏很强,每天早上6点半,就开始自修了,每周有很多考试,口试,
每天需要6,7小时老师授课,然后晚自习到22点30分结束,周6,也学习,所以很多学
生,都是住宿,,读商的在巴黎的,很少有住宿的,外地预科住宿的多,,所以每天单
程40分钟,早上和晚上,来回,会很累的。学校食堂从早开到晚供应,和我们国内大学
类似,就是为了提供一切条件,让学生竟考出最好成绩。每个学生,有权竟考2次,很
多学生,为了某某大学,第一年,名次不够,再读预科,预科最多可以读3年。但是,
一般所有读预科的学生,都会有大学的位置,只是很多读预科的学生,都是为了最前例
的大学而奋斗。
课外活动,特别有特色的,比如体育成绩,国家级水平的,上面说的那些比较难进的大
学,会特殊接受的。
原则上,公司招人,一视同人,不能因为种族信仰,而拒绝,但是比例这个,好像,还
没听说。

。。

【在 d**********h 的大作中提到】
: 真言人人有
: 此番楼主家
: 玻璃碎一地
: 当心小脚丫
: :)

g***3
发帖数: 471
458
是啊,法国这点,就英美系统,不太人性,,
高中生15,6岁,就要决定以后学什么了,做什么了,
当然以后,还是有机会的,,比如那些精英学校,近年来,也会分别在第2,3年,可虑
那些读大学,平行程度的学生,也有一些类似的考试。。
但是,法国各大公司,招人,最公认的,还是那些通过预科,竟考上来的人。

【在 r*g 的大作中提到】
: 法国这么早分流,那么那些被分流的不会不甘心?还有机会跳上去吗?
:
: 400

t******l
发帖数: 10908
459
属实,数学的确很重要。拿破仑其实就是因为理论数学系转业的,所以对打仗的时间空
间感觉特别好。而且拿破仑也对时间轴的 causality 的理解超出凡人,所以在打仗时
对时间特别重视。

:法国从小学6岁开始,小学5年,初中4年,高中3年,,我说的法国预科,是法国特殊
的体制,是对高中毕业,相当于大学2年的课程,但是这个课程,可以说,是法国特色,
:是特别难的,,是拿破仑当时强国振兴,从平民,小资本家庭挑选人才的一种学习,
竟考形式。学生通过一般2年的强化学习,训练,去竟考,根据名次不同,分别再进相应
g***3
发帖数: 471
460
赞,没有绝对的标准,网络的平台,让不同背景的父母,能够交结,产生共鸣,触通启
发,大家一起
集思广意。你们从不同角度,对我提出的问题,也许以前,我还没有注意,通过此贴,
也可以让我重视起来。
妹妹一直是个很感性,聪明的人,我很喜欢。

【在 i**e 的大作中提到】
: 呵呵
: 我一般对这种比较敏感的话题几乎不参与:)
: 很多事情没有那么多的对错,每个人都有自己的信仰和理念
: 只是分享没有说服的目的:)
: 这次是lz真诚坦荡分享,我多说了三四句,所谓言多必失:)

相关主题
我招,我是猪大学录取率
请教:怎样说服孩儿他爸多关心孩子的学习?别鸡兔同笼了,来喝啤酒吧
娃又在推算他的质数公式。。。学校早晚要教的东西,早早学会了又如何?
进入Parenting版参与讨论
g***3
发帖数: 471
461
初中了吧,我女儿这里老师说是相当于美国10年,11年级,

【在 i**e 的大作中提到】
: 6年级
: 跟你大儿子比小多啦

f**********g
发帖数: 4709
462
这个预科,跟国内高考大省的高二高三差不多
那进入预科容易吗?

【在 g***3 的大作中提到】
: 法国从小学6岁开始,小学5年,初中4年,高中3年,,我说的法国预科,是法国特殊的
: 体制,是对高中毕业,相当于大学2年的课程,但是这个课程,可以说,是法国特色,
: 是特别难的,,是拿破仑当时强国振兴,从平民,小资本家庭挑选人才的一种学习,竟
: 考形式。学生通过一般2年的强化学习,训练,去竟考,根据名次不同,分别再进相应
: 学校grande ecole;,再读3年,出来文凭相当于Master,你说的牛人,他跳过几级,,
: 15岁可能就高中毕业了。
: 其他还有政治学院,和学医,学管理,读法律,读艺术等甩选严格的特殊大学,巴黎索
: 邦大学管理,法律学科也很难进入,这样,大概每年也可接纳一万人左右,余下的普通
: 大学就很多了,只要通过高中会考Bac 都可以读的大学,法国每年高中会考合格率达百
: 分90以上。

C********e
发帖数: 2327
463
法国也是最后一考定学校喽,这样挺好,也不用各种aa

400

【在 g***3 的大作中提到】
: 女儿,现在是第一次,去外面上课。
: 介绍下,法国的教育背景,,每年大概有60多万到70万高中毕业生,,
: 拿破仑制定的预科精英制学校,理工商文,全包,每年大概招2,到3万人,,
: 这个2,到3万学生,还要通过2年的苦读,然后再参加全国竟考,,根据排名先后,再
: 被分配到不同名次的精英学校。。比如儿子想上的巴黎3个高商之一,每年招300到400
: 个学生,所以,考试时,在5千多人里,要进入前300名等。。
: 这个2,到3万名学生的挑选,是根据他们高中的学习成绩。预科学校也有排名,越前的
: 学校,进入顶级的精英学校人数越多。
: 我们孩子小时,去周日中文学校,2年多后,放弃。那时他们也有抱怨。
: 但是他们人大了后,特别是到高中后,发现,很多同学,都是从小家里就有私教的,

g***3
发帖数: 471
464
法国全国有理工文商一百多个预科学校。
招生面向全国2600多所综合高中的学生,根据原高中的档次,分别有不同的成绩档次要
求。
最好高中的中档学生,和外省不知名高中最好学生,被招收时的待遇会有差别。
预科学校,每年有竞争激烈的排名,竞争进top精英学校学生的人数,,所以,他们在
招生时,对全法每个最好高中的教育水平是很清楚的,哪些学校的学生,以后能进哪些
精英学校,都有长年的经验,所以,如果想以后进入最好的预科学校,原高中也是很重
要的,然后,是看高中最后2年的成绩,在全校排名次,有些最好的预科,要看高中3年
的成绩,,还有各种课外活动成就,动机信等。
比如老大高中的预科商,每年招80个学生,,他们会受到全国1千500多候选人申请,每
年他们留给自己高中的学生有10个名额左右。儿子这届商有100多名学生,只要能够进
入前10名,就可以进入自己学校预科。

【在 f**********g 的大作中提到】
: 这个预科,跟国内高考大省的高二高三差不多
: 那进入预科容易吗?

g***3
发帖数: 471
465
拿破仑的数学老师,都是法国有名的数学家。拿波仑对教育很重视,好多学校,都是他
那时建立的。

色,
相应

【在 t******l 的大作中提到】
: 属实,数学的确很重要。拿破仑其实就是因为理论数学系转业的,所以对打仗的时间空
: 间感觉特别好。而且拿破仑也对时间轴的 causality 的理解超出凡人,所以在打仗时
: 对时间特别重视。
:
: :法国从小学6岁开始,小学5年,初中4年,高中3年,,我说的法国预科,是法国特殊
: 的体制,是对高中毕业,相当于大学2年的课程,但是这个课程,可以说,是法国特色,
: :是特别难的,,是拿破仑当时强国振兴,从平民,小资本家庭挑选人才的一种学习,
: 竟考形式。学生通过一般2年的强化学习,训练,去竟考,根据名次不同,分别再进相应

g***3
发帖数: 471
466
是的,我看有些名人的履历,有些也是平民出身,就是因为在中学成绩特好,被老师推
荐去读预科,然后考入精英大学。
预科学校对家庭收入低的学生,有特别的经济支援。

【在 C********e 的大作中提到】
: 法国也是最后一考定学校喽,这样挺好,也不用各种aa
:
: 400

t******l
发帖数: 10908
467
这个其实是有道理的,这么说:
过于一人一票的民主的系统,说白了就是让 8 billion 猴子们随时随地都可以
contaminate 智人 intellectual 的系统。。。其本质原因,是如果 “臭皮匠” 和
“猪哥亮” 之间存在质变的差别,那 8 billion 臭皮匠也抵不过 one single 猪哥亮
。。。(老军医鸡汤成语就不驳了,不浪费带宽)。。。
当然,过于不民主的系统也不行。。。目前智人的最佳系统是宪政平衡,但这说起来容
易,具体实现上差别很大。。。就好比 Deepmind 实现出一只阿发狗,但做猴子聊天器
的 facebook 搞出来的就是个 joke。。。

【在 g***3 的大作中提到】
: 拿破仑的数学老师,都是法国有名的数学家。拿波仑对教育很重视,好多学校,都是他
: 那时建立的。
:
: 色,
: 相应

c***x
发帖数: 1826
468

赞慌张。根据高晓松的“四十不惑”理论,慌张就是年轻嘛 :-)
整理好了,请给我的gmail邮箱发一份,谢谢。
还是觉得树比草好。
这世上春风和野火其实都不常有;阳光,泥土和水分倒是必须的。
惟愿那些没有被野火烧死的树,每年都还能长高长大一点吧。
别的,也不知道说什么好了,就不说了。。。。。。

【在 d**********h 的大作中提到】
: 呵呵,合集是为我自己以后参考的,你如果是想要我可以发给你哦。(lz既然发在公共
: 板块,想必是不介意我们收藏的)
: 至于其他,野火即使不收了草原,草原自己一岁也会枯荣交替一次;即使枯荣往复,随
: 春风勃勃而兴的生命也已经不是去年的那些老帮菜了
: 咱努力做好草木灰吧 :),别拿春风这么高大上的角色吓唬我了,弄得咱心里很慌张

i**e
发帖数: 19242
469
没事
碎片拾起来粘吧粘吧还要凑合着接着用呢:)

真言人人有

【在 d**********h 的大作中提到】
: 真言人人有
: 此番楼主家
: 玻璃碎一地
: 当心小脚丫
: :)

i**e
发帖数: 19242
470
你这妹妹一声一声地叫得亲切
我都不知道是你在占我便宜还是我在占你便宜了
赶紧,生成八字私信火速报将上来
真比我大,就认了:)

【在 g***3 的大作中提到】
: 赞,没有绝对的标准,网络的平台,让不同背景的父母,能够交结,产生共鸣,触通启
: 发,大家一起
: 集思广意。你们从不同角度,对我提出的问题,也许以前,我还没有注意,通过此贴,
: 也可以让我重视起来。
: 妹妹一直是个很感性,聪明的人,我很喜欢。

相关主题
学校早晚要教的东西,早早学会了又如何?是不是任我儿子继续迷象棋
构建式数学在中国9岁, 如何报名考amc10
关于刷还是不刷求科普: 奥数 vs. Math Olympiad vs. Math Count vs. Math circle vs. 超前学数学
进入Parenting版参与讨论
i**e
发帖数: 19242
471
米国义务教育 K-12
Kindergarten 5岁开始
有的地方6年级还在小学,有的地方是初中第一年
5岁入小学,18岁入大学:)

【在 g***3 的大作中提到】
: 初中了吧,我女儿这里老师说是相当于美国10年,11年级,
t******l
发帖数: 10908
472
按楼主说的 “法国从小学6岁开始,小学5年,初中4年,高中3年”,换算下来法国高
一相当于美国 10 年级,而不是 11 年级。
至于 小学 vs 初中,只是个虚词差别而已。。。我们也可以把 PHD 说成小学,然后没
学微分几何的统统说成小学肄业好了。

【在 i**e 的大作中提到】
: 米国义务教育 K-12
: Kindergarten 5岁开始
: 有的地方6年级还在小学,有的地方是初中第一年
: 5岁入小学,18岁入大学:)

h*****m
发帖数: 1034
473
没失没失,是我的问题,有时候太较真了。
你说的对,大家各抒己见,每家的情况都不同嘛。

【在 i**e 的大作中提到】
: 呵呵
: 我一般对这种比较敏感的话题几乎不参与:)
: 很多事情没有那么多的对错,每个人都有自己的信仰和理念
: 只是分享没有说服的目的:)
: 这次是lz真诚坦荡分享,我多说了三四句,所谓言多必失:)

i**e
发帖数: 19242
474
奥......
我这理解理欠费,你谅解吧,呵呵

【在 t******l 的大作中提到】
: 按楼主说的 “法国从小学6岁开始,小学5年,初中4年,高中3年”,换算下来法国高
: 一相当于美国 10 年级,而不是 11 年级。
: 至于 小学 vs 初中,只是个虚词差别而已。。。我们也可以把 PHD 说成小学,然后没
: 学微分几何的统统说成小学肄业好了。

i**e
发帖数: 19242
475
莫紧张:) 没事儿

【在 h*****m 的大作中提到】
: 没失没失,是我的问题,有时候太较真了。
: 你说的对,大家各抒己见,每家的情况都不同嘛。

t******l
发帖数: 10908
476
其实我还是在想 air-lifting-快糙猛 vs 烤肉拉-开天窗。。。我觉得其中的一个重大
差别,是 “战略上进攻,战术上防守” vs “战术上进攻,战略上防守”。。。这么
说。。。
“air-lifting-快糙猛” 的基本想法是,在敌人到达之前快速抢位,抢位后防守敌人
反扑。所以强调快和猛,不强调精细进攻技术(因为先敌抢位占领,抢位时反正山头上
也没啥敌人,抢位后主要是防守敌人反扑)。。。这样容易达到 “战略上进攻,战术
上防守” 。。。
而 “烤肉拉-开天窗” 的基本想法,是先搞精细进攻战术。但精细了就慢(有无限量
V22 鱼鹰的天才娃另说,但大部分都没有),不容易先敌抢位。。。这样容易陷入 “
战术上进攻,战略上防守” 的不利态势。。。
g***3
发帖数: 471
477
法国的选举制度有一个很长的变化过程,
从1791年,到1848,期间政权有几次变化,但只有男的,超过25岁,至少交一定数目的
税收,才有权投票。投票人岁数,,交税数目,有几次变化,在25岁到30岁间,交税数
目从至少3天工资以上,到300多法郎,,,
1848 到1944,是男子有权投票。。1944年后至从,女子也能投票了。投票人岁数降至
18岁。不交税的公民都有权了。

【在 t******l 的大作中提到】
: 这个其实是有道理的,这么说:
: 过于一人一票的民主的系统,说白了就是让 8 billion 猴子们随时随地都可以
: contaminate 智人 intellectual 的系统。。。其本质原因,是如果 “臭皮匠” 和
: “猪哥亮” 之间存在质变的差别,那 8 billion 臭皮匠也抵不过 one single 猪哥亮
: 。。。(老军医鸡汤成语就不驳了,不浪费带宽)。。。
: 当然,过于不民主的系统也不行。。。目前智人的最佳系统是宪政平衡,但这说起来容
: 易,具体实现上差别很大。。。就好比 Deepmind 实现出一只阿发狗,但做猴子聊天器
: 的 facebook 搞出来的就是个 joke。。。

g***3
发帖数: 471
478
妹妹啊,我是69年的。

【在 i**e 的大作中提到】
: 你这妹妹一声一声地叫得亲切
: 我都不知道是你在占我便宜还是我在占你便宜了
: 赶紧,生成八字私信火速报将上来
: 真比我大,就认了:)

g***3
发帖数: 471
479
法国这里,孩子入学,是受从1月一日到12月31日的孩子,其中学生差距挺大的。

【在 t******l 的大作中提到】
: 按楼主说的 “法国从小学6岁开始,小学5年,初中4年,高中3年”,换算下来法国高
: 一相当于美国 10 年级,而不是 11 年级。
: 至于 小学 vs 初中,只是个虚词差别而已。。。我们也可以把 PHD 说成小学,然后没
: 学微分几何的统统说成小学肄业好了。

d**********h
发帖数: 2795
480
第一个典型的浪仗,第二个典型的呆仗
兵法讲究:以正合以奇胜
一个统帅的特点偏向正合,就是呆仗,偏向以奇胜就是浪仗
无农不稳无商不富,农就是正就是呆,商*就是奇就是浪
那个谁用三角形比喻物理学家的时候,说爱因斯坦是基础扎实头脑敏锐,就是典型的正
合奇胜,最差的是基础薄弱头脑迟钝,最常见的是基础扎实头脑迟钝,也就是我又堂堂
之阵却无破敌之法,这就只能老老实实步步为营的打呆仗了
真正正奇相得的,那是老爱的水平,大家过过嘴瘾就行了,现实里不用考虑了
你海空直降,占了先机,你也要能守得住啊!拿破仑和元首当年就是够猛,结果是夹生
饭,把自己给坑了。
互联网时代的呆浪总结是:跟在最浪的那个身后“半步”之遥。跟得太近太浪,很容易
成炮灰,成为被拍死在沙滩的前浪;落后太远就不容易追赶,就真的是“呆子”了
当然如果家里有条件让娃打浪仗,那也别浪费了资源不是



【在 t******l 的大作中提到】
: 其实我还是在想 air-lifting-快糙猛 vs 烤肉拉-开天窗。。。我觉得其中的一个重大
: 差别,是 “战略上进攻,战术上防守” vs “战术上进攻,战略上防守”。。。这么
: 说。。。
: “air-lifting-快糙猛” 的基本想法是,在敌人到达之前快速抢位,抢位后防守敌人
: 反扑。所以强调快和猛,不强调精细进攻技术(因为先敌抢位占领,抢位时反正山头上
: 也没啥敌人,抢位后主要是防守敌人反扑)。。。这样容易达到 “战略上进攻,战术
: 上防守” 。。。
: 而 “烤肉拉-开天窗” 的基本想法,是先搞精细进攻战术。但精细了就慢(有无限量
: V22 鱼鹰的天才娃另说,但大部分都没有),不容易先敌抢位。。。这样容易陷入 “
: 战术上进攻,战略上防守” 的不利态势。。。

相关主题
有明天考AMC 8的吗?少年智力开发
算不算有数学天分[转载] 为什么说usamo 简单
大家怎么推孩子的弱项?二年级女儿的report
进入Parenting版参与讨论
i**e
发帖数: 19242
481
你跟我家妹同年哦:)
我大学同学有去年升级做爷爷的,呵呵

【在 g***3 的大作中提到】
: 妹妹啊,我是69年的。
t******l
发帖数: 10908
482
哥们我觉得您搞反了。在基础教育方面,先搞 “快糙猛”(不讲究旁门左道的偏门技
巧)才是 “正合”;而先搞 “烤肉拉天窗扔水果刀” 这种偏门技巧,那才是 “奇胜
”。
举个例子,好比记忆二次方程球根公式而不理解其 “complete a square” 内在思想
,你要是觉得记忆公式是 “正合”,那您显然是被猴子 contaminate 的数学教科书给
卖了。。。记忆二次方程的球根公式是典型的 “烤肉拉天窗扔水果刀” 的偏门技巧行
为。。。(当然三次及以上方程另说,那个不属于基础教育的范畴。一般只有铣工技校
才学,车工刨工学校根本不学三次方程球根。)
当然我不是一味反对 “奇胜”,但 “奇胜” 应该是技校阶段的事儿。。。基础教育
阶段搞 “Z-turn 型奇胜教育法”,我个人觉得很大概率会造成 short term 考试高分
,但实质是自欺欺人。这不仅没锻炼基本素质,连 where we are 的信息这种 long
term decision 所必须的信息,都是喝完老军医鸡汤后的幻觉。。。这样将来 long
term 是不是会 decay,堪忧。。。



【在 d**********h 的大作中提到】
: 第一个典型的浪仗,第二个典型的呆仗
: 兵法讲究:以正合以奇胜
: 一个统帅的特点偏向正合,就是呆仗,偏向以奇胜就是浪仗
: 无农不稳无商不富,农就是正就是呆,商*就是奇就是浪
: 那个谁用三角形比喻物理学家的时候,说爱因斯坦是基础扎实头脑敏锐,就是典型的正
: 合奇胜,最差的是基础薄弱头脑迟钝,最常见的是基础扎实头脑迟钝,也就是我又堂堂
: 之阵却无破敌之法,这就只能老老实实步步为营的打呆仗了
: 真正正奇相得的,那是老爱的水平,大家过过嘴瘾就行了,现实里不用考虑了
: 你海空直降,占了先机,你也要能守得住啊!拿破仑和元首当年就是够猛,结果是夹生
: 饭,把自己给坑了。

t******l
发帖数: 10908
483
当然其实这个问题不怪 dragonbreath,主要因为数学教科书要保证 No Child Left
Behind。而记忆住二次方程球根公式,至少这学期不会 Left Behind 是刚刚的。至于
下学期,那也不归我这个老师这本教材管不是?
当然也有时代发展因子,肉算时代记忆个球根公式,至少可以当人肉计算器。。。这就
相当于一战挖堑壕,算是 “正合”。而一战时的 5mph 柜子坦克,也确实是 “奇胜”
,而且事实上也就 “奇胜” 了那么一次,第二次被发现弱点就很翘很翘了。。。但二
战在开阔平原挖战壕,反而变成了 “奇胜”,当然情急时也可以使这招 。二战时在开
阔地坦克集群冲击,就摇身一变成为 “正合” 了。。。

【在 t******l 的大作中提到】
: 哥们我觉得您搞反了。在基础教育方面,先搞 “快糙猛”(不讲究旁门左道的偏门技
: 巧)才是 “正合”;而先搞 “烤肉拉天窗扔水果刀” 这种偏门技巧,那才是 “奇胜
: ”。
: 举个例子,好比记忆二次方程球根公式而不理解其 “complete a square” 内在思想
: ,你要是觉得记忆公式是 “正合”,那您显然是被猴子 contaminate 的数学教科书给
: 卖了。。。记忆二次方程的球根公式是典型的 “烤肉拉天窗扔水果刀” 的偏门技巧行
: 为。。。(当然三次及以上方程另说,那个不属于基础教育的范畴。一般只有铣工技校
: 才学,车工刨工学校根本不学三次方程球根。)
: 当然我不是一味反对 “奇胜”,但 “奇胜” 应该是技校阶段的事儿。。。基础教育
: 阶段搞 “Z-turn 型奇胜教育法”,我个人觉得很大概率会造成 short term 考试高分

t******l
发帖数: 10908
484
当然现在 8 billion 猴子已经进化到游行示威把考试给统统给取消掉咔嚓掉。。。当
然这倒确实是也不用记忆二次球根公式了。。。也是给黄小木的 “人类的 mean /
medium 在不断进化” 的一个生动的证明。
t******l
发帖数: 10908
485
高中毕业时其实也就是 +- 1/36 的差别,不算太大。。。不过基本所有的国家系统,
都是按年级而不是按年龄划线,所以实际上是年龄大点的占小小的优势。。。而跳级后
但水平不到能跨级竞赛的,纠结了。。。当然跳级确实给纳税人省钱,减轻社会负担。
。。

【在 g***3 的大作中提到】
: 法国这里,孩子入学,是受从1月一日到12月31日的孩子,其中学生差距挺大的。
k**n
发帖数: 6198
486
林彪数学怎样?

色,
相应

【在 t******l 的大作中提到】
: 属实,数学的确很重要。拿破仑其实就是因为理论数学系转业的,所以对打仗的时间空
: 间感觉特别好。而且拿破仑也对时间轴的 causality 的理解超出凡人,所以在打仗时
: 对时间特别重视。
:
: :法国从小学6岁开始,小学5年,初中4年,高中3年,,我说的法国预科,是法国特殊
: 的体制,是对高中毕业,相当于大学2年的课程,但是这个课程,可以说,是法国特色,
: :是特别难的,,是拿破仑当时强国振兴,从平民,小资本家庭挑选人才的一种学习,
: 竟考形式。学生通过一般2年的强化学习,训练,去竟考,根据名次不同,分别再进相应

k**n
发帖数: 6198
487
拿破仑“从平民,小资本家庭挑选人才的一种学习竟考形式”
很好奇法国的教育系统目前是如何照顾socioeconomic status的?

【在 g***3 的大作中提到】
: 法国从小学6岁开始,小学5年,初中4年,高中3年,,我说的法国预科,是法国特殊的
: 体制,是对高中毕业,相当于大学2年的课程,但是这个课程,可以说,是法国特色,
: 是特别难的,,是拿破仑当时强国振兴,从平民,小资本家庭挑选人才的一种学习,竟
: 考形式。学生通过一般2年的强化学习,训练,去竟考,根据名次不同,分别再进相应
: 学校grande ecole;,再读3年,出来文凭相当于Master,你说的牛人,他跳过几级,,
: 15岁可能就高中毕业了。
: 其他还有政治学院,和学医,学管理,读法律,读艺术等甩选严格的特殊大学,巴黎索
: 邦大学管理,法律学科也很难进入,这样,大概每年也可接纳一万人左右,余下的普通
: 大学就很多了,只要通过高中会考Bac 都可以读的大学,法国每年高中会考合格率达百
: 分90以上。

t******l
发帖数: 10908
488
当然这个讨论有很大的价值就是,俺在美国一直看不懂花国家巨资搞出来的正统教科书
,为啥总是让人有一种 “玉石和 bullshit 齐飞” 的喜剧效果。。。而 internet 地
摊上免费的玩意儿,从 Khan Academy 到 IMO 免费习题集不等,至少都简洁明了而自
洽不是?(当然具体也要具体看,你总不能拿 Khan Academy 去学 IMO 就是了)。。
。这世界为啥这么奇妙。。。
现在我明白了,看任何公立学校系统的教科书,一定要从 “是不是被猴子
contaminate 过?” 这个角度 critical thinking 一下,就能恍然大悟。。。并且这
样就容易以 “去 bullshit 而留玉石” 的姿势,在公立学校教科书上游刃有余。。。
h*****m
发帖数: 1034
489
双方辨友:在进入下一回合之前,请先澄清一下:
"complete a square"和“记忆二次方程球根公式“到底哪个是“正合”,哪个是“奇
胜“?
”complete a repeated addition"和“背乘法口诀“哪个是“正合”,哪个是“奇胜
”?
谢谢!
***********************************************
发信人: timefall (时光崩塌), 信区: Parenting
标 题: Re: 求建议,高中生如何提高学习效率?
发信站: BBS 未名空间站 (Fri Apr 22 12:20:15 2016, 美东)
哥们我觉得您搞反了。在基础教育方面,先搞 “快糙猛”(不讲究旁门左道的偏门技
巧)才是 “正合”;而先搞 “烤肉拉天窗扔水果刀” 这种偏门技巧,那才是 “奇胜
”。
举个例子,好比记忆二次方程球根公式而不理解其 “complete a square” 内在思想
,你要是觉得记忆公式是 “正合”,那您显然是被猴子 contaminate 的数学教科书给
卖了。。。记忆二次方程的球根公式是典型的 “烤肉拉天窗扔水果刀” 的偏门技巧行
为。。。(当然三次及以上方程另说,那个不属于基础教育的范畴。一般只有铣工技校
才学,车工刨工学校根本不学三次方程球根。)
当然我不是一味反对 “奇胜”,但 “奇胜” 应该是技校阶段的事儿。。。基础教育
阶段搞 “Z-turn 型奇胜教育法”,我个人觉得很大概率会造成 short term 考试高分
,但实质是自欺欺人。这不仅没锻炼基本素质,连 where we are 的信息这种 long
term decision 所必须的信息,都是喝完老军医鸡汤后的幻觉。。。这样将来 long
term 是不是会 decay,堪忧。。。
***********************************************
发信人: dragonbreath (龙吸), 信区: Parenting
标 题: Re: 求建议,高中生如何提高学习效率?
发信站: BBS 未名空间站 (Fri Apr 22 09:25:34 2016, 美东)
第一个典型的浪仗,第二个典型的呆仗
兵法讲究:以正合以奇胜
一个统帅的特点偏向正合,就是呆仗,偏向以奇胜就是浪仗
无农不稳无商不富,农就是正就是呆,商*就是奇就是浪
那个谁用三角形比喻物理学家的时候,说爱因斯坦是基础扎实头脑敏锐,就是典型的正
合奇胜,最差的是基础薄弱头脑迟钝,最常见的是基础扎实头脑迟钝,也就是我又堂堂
之阵却无破敌之法,这就只能老老实实步步为营的打呆仗了
真正正奇相得的,那是老爱的水平,大家过过嘴瘾就行了,现实里不用考虑了
你海空直降,占了先机,你也要能守得住啊!拿破仑和元首当年就是够猛,结果是夹生
饭,把自己给坑了。
互联网时代的呆浪总结是:跟在最浪的那个身后“半步”之遥。跟得太近太浪,很容易
成炮灰,成为被拍死在沙滩的前浪;落后太远就不容易追赶,就真的是“呆子”了
当然如果家里有条件让娃打浪仗,那也别浪费了资源不是
t******l
发帖数: 10908
490
哥们,“complete a square” 和 “记忆二次方程球根公式”,对二次方程求解,是
两个等价的玩意儿,而且是有一定意义完备性的玩意儿,都是能完备地解出二次方程。
你这个 “complete a repeated addition” vs “背乘法口诀”,这根本不是一个等
价的玩意儿,而且根本没有完备性的意义。。。哥们您背 size 是 100000 × 100000
乘法口诀表,不用 place value 竖式?。。。或者哥们您做 addition 是基本靠扳手
指头,不记忆 7 + 8 = 15?。。。而且就算是 mathematical properties,那乘法也
是 distributive property 在空间上切 2D array 蛋糕,而不能仅仅停留在 repeated
addition 这种 naive intuition 的层次。否则你分数咋 repeated addition,负负
得正咋 repeated addition。。。
还有哥们能不能不拿三年级以下的算术做例子行不行?。。。打个比方,算术就好比
20 米跑步。。。你要跑 100 米还至少有个起跑、中途、冲刺之类的战术,6 个钉子还
是 8 个钉子。。。你跑个 20 米也要搞一套战略战术出来的话,这也太 over-
engineering 了不是?。。。

【在 h*****m 的大作中提到】
: 双方辨友:在进入下一回合之前,请先澄清一下:
: "complete a square"和“记忆二次方程球根公式“到底哪个是“正合”,哪个是“奇
: 胜“?
: ”complete a repeated addition"和“背乘法口诀“哪个是“正合”,哪个是“奇胜
: ”?
: 谢谢!
: ***********************************************
: 发信人: timefall (时光崩塌), 信区: Parenting
: 标 题: Re: 求建议,高中生如何提高学习效率?
: 发信站: BBS 未名空间站 (Fri Apr 22 12:20:15 2016, 美东)

相关主题
觉得美帝数学进度慢的进来看一下请教:怎样说服孩儿他爸多关心孩子的学习?
分数应用题娃又在推算他的质数公式。。。
我招,我是猪大学录取率
进入Parenting版参与讨论
t******l
发帖数: 10908
491
或者这么说,跑个 20 米就得来一套起跑中途冲刺的战略战术出来。。。那将来 100
米别人直接就上训练战略战术了,但那悲催的哥们先得把 20 米战略战术的坏习惯改了
才能学 100 米战略战术不是?。。。我看那哥们多半扔下一句 “娘的 Z-turn 教学法
,这日子我没法过”,然后直接走人。。。

100000
repeated

【在 t******l 的大作中提到】
: 哥们,“complete a square” 和 “记忆二次方程球根公式”,对二次方程求解,是
: 两个等价的玩意儿,而且是有一定意义完备性的玩意儿,都是能完备地解出二次方程。
: 你这个 “complete a repeated addition” vs “背乘法口诀”,这根本不是一个等
: 价的玩意儿,而且根本没有完备性的意义。。。哥们您背 size 是 100000 × 100000
: 乘法口诀表,不用 place value 竖式?。。。或者哥们您做 addition 是基本靠扳手
: 指头,不记忆 7 + 8 = 15?。。。而且就算是 mathematical properties,那乘法也
: 是 distributive property 在空间上切 2D array 蛋糕,而不能仅仅停留在 repeated
: addition 这种 naive intuition 的层次。否则你分数咋 repeated addition,负负
: 得正咋 repeated addition。。。
: 还有哥们能不能不拿三年级以下的算术做例子行不行?。。。打个比方,算术就好比

t******l
发帖数: 10908
492
或者这么比方,国象小娃酱油班的老师,跟小娃们说。。。初学酱油国象的时候,不要
过于热心记忆开局棋谱了。。。然后后边一位家长立马质疑曰:那老师您说这匹拐角马
的跳法要不要记忆?。。。
我估计狠点的老师会这么反应:。。。后边那位家长,旁边 2 号窗口,退学费。不送
,谢谢。。。

【在 t******l 的大作中提到】
: 或者这么说,跑个 20 米就得来一套起跑中途冲刺的战略战术出来。。。那将来 100
: 米别人直接就上训练战略战术了,但那悲催的哥们先得把 20 米战略战术的坏习惯改了
: 才能学 100 米战略战术不是?。。。我看那哥们多半扔下一句 “娘的 Z-turn 教学法
: ,这日子我没法过”,然后直接走人。。。
:
: 100000
: repeated

h*****m
发帖数: 1034
493
呵呵,我只观战,看到不清楚的问两句。休想用一波波的潮水把我冲下来。: )
龙吸呢?快来救我!

【在 t******l 的大作中提到】
: 或者这么比方,国象小娃酱油班的老师,跟小娃们说。。。初学酱油国象的时候,不要
: 过于热心记忆开局棋谱了。。。然后后边一位家长立马质疑曰:那老师您说这匹拐角马
: 的跳法要不要记忆?。。。
: 我估计狠点的老师会这么反应:。。。后边那位家长,旁边 2 号窗口,退学费。不送
: ,谢谢。。。

k**n
发帖数: 6198
494
你是野火烧不尽,还是潮水淹又生?

【在 h*****m 的大作中提到】
: 呵呵,我只观战,看到不清楚的问两句。休想用一波波的潮水把我冲下来。: )
: 龙吸呢?快来救我!

h*****m
发帖数: 1034
495
大潮过后,被冲昏过去的观众醒了过来,嘴里哼哼唧唧地嘟囔着:“不讲道理嘛,哪有
对着观众冲得!?另一名选手哪去了?“
我对小孩学习过程的理解:对每一个新知识,都需要从道理上明白为什么会这样,然后
总结出一个能够快速解决问题的方法(公式,或步骤),记熟。再往后,这已经记熟的
方法就作为理解更高级的知识的基础。理解,掌握,记忆,如此往复循环。乘法是这样
,二次方程求根也是这样。
不要小瞧个位数乘法,对于小小孩来说,难度不一定小于方程求根对于中学生的难度。
用连加计算乘法,是最天然的方式,我家老二现在上K,昨天问我8分钟是多少秒,我让
他自己想,人家就是60,60,60。。加出来了,还挺让我觉得高兴的,这是“正”,现
在让他背口诀,是“奇“。可是如果再过两三年,他还是这么算,我就该头疼了,因为
那时口诀才是“正”,那时的“奇”是对于更高级的比如说二位数乘法而言了。二位数
乘法,开始时,列竖式按步骤算是“奇“,画格子分块算再加起来才是“正“,因为这
样才能理解。再往后,列竖式又变成“正”了,那时我要看他见个乘法题就开始画格子
,头又要疼了。
中学生学解二次方程,当然应该从"complete the square"开始理解解题原理,但前人
总结出个求根公式放教科书里不是吃饱了撑的。首先,用公式解方程确实更快;其次,
很多时候不需要真正求出方程的根,拿b^2-4ac快速判断一下方程有没有实根,一个根
还是两个根,心算就可以了;更重要的是,后面的很多内容好像是需要从求根公式推导
出来的,比如说韦达定理。年代太久远了记不太清楚了,求根公式应该有好多应用的,
比如说方程的常数项里放个新变量m的表达式,问你m为何值时,方程有唯一根。等等。
对这些问题,你不能都来一个"complete the square"吧?(也许可以?但想想效率)
总之,我的观点是:初学一个知识时,理解是“正”,背公式是“奇”。等掌握了这个
知识以后,背公式就转化为“正”了。没有这个“正”,后面的学习会遇到困难。该背
的公式,是一定要背的。
所有这些,可能和你说的并不矛盾,也可能就是一回事。可是看你一会儿air-lifting
了,一会儿“烤肉拉开天窗“了,一会儿又跑去“挖战壕“了,迷糊!就来问个问题,
至于用那么多的水来冲吗?: )

【在 h*****m 的大作中提到】
: 呵呵,我只观战,看到不清楚的问两句。休想用一波波的潮水把我冲下来。: )
: 龙吸呢?快来救我!

h*****m
发帖数: 1034
496
我是潮水冲不断,又是一条好汉!: )

【在 k**n 的大作中提到】
: 你是野火烧不尽,还是潮水淹又生?
t******l
发帖数: 10908
497
三年级以下的数学我不讨论。。。对于 complete a square vs 球根公式而言,我实战
教娃 AMC 10 的经验,对于纯粹二次方程求个跟的 Kumon 型 routine 题,complete a
square 比 球根公式 慢 10%。。。但 AMC 10 型的计算题(不是简单按照 routine
解方程,否则是 Kumon),complete a square 通常比求根公式快 10% 到 50% 不等。
快更多的例子的题目也有。。至于 AIME,我娃现在做不了 AIME, 目前对 AIME 没有
实战经验。。。
之所以球根公式不会太快,因为你要把 a b c 都给对上,然后默写公式 apply 上去。
。。有这点时间,除掉二次项系数,然后 x 减去一次项系数的一半,括号平方,用一
次项系数一半的平方平衡掉,两边开根号,也差不多搞完了。。。其实球根公式不会省
多少时间。。。唯一的例外是备考前把球根公式练上五百遍到手熟,不过这种其实考完
就忘的,属于临考突击,也不能用作日常教学的搞法。。。个人看法。。。

【在 h*****m 的大作中提到】
: 大潮过后,被冲昏过去的观众醒了过来,嘴里哼哼唧唧地嘟囔着:“不讲道理嘛,哪有
: 对着观众冲得!?另一名选手哪去了?“
: 我对小孩学习过程的理解:对每一个新知识,都需要从道理上明白为什么会这样,然后
: 总结出一个能够快速解决问题的方法(公式,或步骤),记熟。再往后,这已经记熟的
: 方法就作为理解更高级的知识的基础。理解,掌握,记忆,如此往复循环。乘法是这样
: ,二次方程求根也是这样。
: 不要小瞧个位数乘法,对于小小孩来说,难度不一定小于方程求根对于中学生的难度。
: 用连加计算乘法,是最天然的方式,我家老二现在上K,昨天问我8分钟是多少秒,我让
: 他自己想,人家就是60,60,60。。加出来了,还挺让我觉得高兴的,这是“正”,现
: 在让他背口诀,是“奇“。可是如果再过两三年,他还是这么算,我就该头疼了,因为

t******l
发帖数: 10908
498
:比如说方程的常数项里放个新变量m的表达式,问你m为何值时,方程有唯一根。等等。
:对这些问题,你不能都来一个"complete the square"吧?(也许可以?但想想效率)
这个问题 complete a square 不会比球根公式慢,唯一根就是 complete a square 以
后剩下的常数项为零。或者用 spatial 的表述就是,整个表达式被 complete 成一个
完整的 square,不多不少。
当然 AMC 10 往后边一些的题目,不会停留在问个何时有唯一根。再往后的情况下,
complete a square 往往比球根公式更快。

【在 h*****m 的大作中提到】
: 大潮过后,被冲昏过去的观众醒了过来,嘴里哼哼唧唧地嘟囔着:“不讲道理嘛,哪有
: 对着观众冲得!?另一名选手哪去了?“
: 我对小孩学习过程的理解:对每一个新知识,都需要从道理上明白为什么会这样,然后
: 总结出一个能够快速解决问题的方法(公式,或步骤),记熟。再往后,这已经记熟的
: 方法就作为理解更高级的知识的基础。理解,掌握,记忆,如此往复循环。乘法是这样
: ,二次方程求根也是这样。
: 不要小瞧个位数乘法,对于小小孩来说,难度不一定小于方程求根对于中学生的难度。
: 用连加计算乘法,是最天然的方式,我家老二现在上K,昨天问我8分钟是多少秒,我让
: 他自己想,人家就是60,60,60。。加出来了,还挺让我觉得高兴的,这是“正”,现
: 在让他背口诀,是“奇“。可是如果再过两三年,他还是这么算,我就该头疼了,因为

t******l
发帖数: 10908
499
不过也不用老是死磕 complete a square。。。随便举一个其他的例子,比如解析几何
里,各种直线方程都是多余。。。因为万变不离其宗就是直线的 point-slope 的表述
,其本质就是 point-slope 就是 “直” 的数学语言,因为 slope 是 rise over run
。。。如果死背解析几何啥子式啥子式直线方程,那将来的三角函数一定更加死记硬背
。。。
俺有五百种不该死背公式的例子。。。可以冲五百遍没有问题。。。当然就是可能比较
费带宽。。。哈哈哈哈哈哈哈

【在 h*****m 的大作中提到】
: 大潮过后,被冲昏过去的观众醒了过来,嘴里哼哼唧唧地嘟囔着:“不讲道理嘛,哪有
: 对着观众冲得!?另一名选手哪去了?“
: 我对小孩学习过程的理解:对每一个新知识,都需要从道理上明白为什么会这样,然后
: 总结出一个能够快速解决问题的方法(公式,或步骤),记熟。再往后,这已经记熟的
: 方法就作为理解更高级的知识的基础。理解,掌握,记忆,如此往复循环。乘法是这样
: ,二次方程求根也是这样。
: 不要小瞧个位数乘法,对于小小孩来说,难度不一定小于方程求根对于中学生的难度。
: 用连加计算乘法,是最天然的方式,我家老二现在上K,昨天问我8分钟是多少秒,我让
: 他自己想,人家就是60,60,60。。加出来了,还挺让我觉得高兴的,这是“正”,现
: 在让他背口诀,是“奇“。可是如果再过两三年,他还是这么算,我就该头疼了,因为

t******l
发帖数: 10908
500
这个如果用 complete a square 做 10 道题来判断。第一练习了代数变换凑 square
的能力和速度。第二理解了 additive identity 的 additive inverse 就是 additive
identity 的自己,以及 additive identity 的唯一性。第三从 complete square 以
后的表达式形式(vertex form),为将来二次函数的解析几何画图移轴,以及二次方
程求根的解析几何二次函数空间表述,打好 algebra 1 的基础。。。我觉得是基础教
育 “快糙猛” 和 “正合” 生动事例。。。
那我们看一下 “拿b^2-4ac快速判断一下方程有没有实根” 做 10 道题后学到了啥。
。。不说了,烤肉拉天窗里飞出 10 把水果刀。。。当然这种 “奇胜” 的办法对付考
试真的是刚刚的 “奇胜”。。。古人云:烤肉拉天窗飞出水果刀,刀刀应考。。。当
然考完以后干嘛,不知道,有阿发狗以后谁会需要 (b^2-4ac)?: 的人肉计算器,还是
先还给老师再说,天天背着这用不着的球根公式也怪沉的。。。
1 2 下页 末页 (共2页)
进入Parenting版参与讨论
相关主题
我招,我是猪是不是任我儿子继续迷象棋
请教:怎样说服孩儿他爸多关心孩子的学习?9岁, 如何报名考amc10
娃又在推算他的质数公式。。。求科普: 奥数 vs. Math Olympiad vs. Math Count vs. Math circle vs. 超前学数学
大学录取率有明天考AMC 8的吗?
别鸡兔同笼了,来喝啤酒吧算不算有数学天分
学校早晚要教的东西,早早学会了又如何?大家怎么推孩子的弱项?
构建式数学在中国少年智力开发
关于刷还是不刷[转载] 为什么说usamo 简单
相关话题的讨论汇总
话题: 孩子话题: 女儿话题: 学习话题: 老大话题: 法国